0% found this document useful (0 votes)
257 views

Bdtmathlinks PDF

1. The problem involves proving an inequality relating positive numbers x, y, and z. 2. The left hand side of the inequality can be rewritten in a form involving products of x, y, and z. 3. Applying the AM-GM inequality and Schur's inequality shows that the left hand side is greater than or equal to the right hand side, proving the original inequality.

Uploaded by

Nguyen Huu Quan
Copyright
© © All Rights Reserved
We take content rights seriously. If you suspect this is your content, claim it here.
Available Formats
Download as PDF, TXT or read online on Scribd
0% found this document useful (0 votes)
257 views

Bdtmathlinks PDF

1. The problem involves proving an inequality relating positive numbers x, y, and z. 2. The left hand side of the inequality can be rewritten in a form involving products of x, y, and z. 3. Applying the AM-GM inequality and Schur's inequality shows that the left hand side is greater than or equal to the right hand side, proving the original inequality.

Uploaded by

Nguyen Huu Quan
Copyright
© © All Rights Reserved
We take content rights seriously. If you suspect this is your content, claim it here.
Available Formats
Download as PDF, TXT or read online on Scribd
You are on page 1/ 134

1, Let a, b and c are positive numbers.

Prove that:
a2
a+b+c
b2
c2

+
+
a2 + bc b2 + ca c2 + ab
2 3 abc
2, Let a, b and c are positive numbers. Prove that:
5 ab + ac + bc
a2
b2
c2

2
+
+
2 a + b2 + c2
a2 + bc b2 + ca c2 + ab
3, Given a, b, c 0. Prove that:
a2
b2
c2
(a + b + c)2
2
+ 2
+ 2
2(ab + bc + ca)
a + bc b + ca c + ab
HD
C1
X a2
X
2a2
a2 (a b)(a c)

=
0
2
(a + b)(a + c)
a + bc
(a + b)(a + c)(a2 + bc)
P
X
(a + b + c)2
2 ab(a + b)
2a2

=
2(ab + bc + ca)
(a + b)(a + c)
(a + b)(b + c)(c + a)

Assume that a + b + c = 1 and put q = ab + bc + ca, r = abc, then the inequality


becomes
1
q 3r

4q
qr
qr

4q
q 3r

2r
4q 1
q 3r

By Schurs Inequality for third degree, we have r

4q1
9 ,

then

2r
6r
2r

=
q 3r
1
q
q 4q1
3
It suffices to show that
6r (4q 1)(1 q)
But this is just Schurs Inequality for fourth degree
X
X
X
a4 + abc
a
ab(a2 + b2 )
We have done.
C2
My proof: Suppose a + b + c = 3. We need to prove:
f (r) = 4q 4 9q 3 + 24qr2 54q 2 r 72r2 243r + 216qr 0
1

f 0 (r) = 48qr 54q 2 144r 243 + 216q


f 00 (r) = 48(q 3) 0, sof 0 (r) f 0 (0) = 54q 2 144 + 216q 0
So, with q

9
4 , f (r)

f (0) = q 3 (4q 9) 0 With q

f (r) f (

4q 9
) 0(trues with q
3

9
4,

we have:

9
4)

4, Let a + b + c = 1; a, b, c 0. Prove that:


1
1
1
3
+
+
2
2a 5b2
2b 5c2
2c 5a2
(a + b2 + c2 )2
HD
LHS

3
9
3
P 2 2
P 2
P 2 = P 2 2
2
2
2
2(a + b + c) 5 a
(a + b + c )(6 a 5 a )
( a )

5, Let x, y, z 0 and x + y + z = 1. Prove that:


27(x3 + yz)(y 3 + xz)(z 3 + xy) 64x2 y 2 z 2
HD: Its the following ineq of a well-know ineq :
(a2 + 3)(b2 + 3)(c2 + 3) 64 a + b + c = 3
Y 3x4
(
+ 3) 64
xyz
Setting :3x = a, 3y = b, 3z = c By am-gm , we have :
LHS(1) (a4 + 3)(b4 + 3)(c4 + 3)

1
((a2 + 3)(b2 + 3)(c2 + 3))2 64
64

Note : Its better if you think more about classical ineq before use modern tech 6, Find
the best value of k to this ineq is truefor all a, b, c 0, abc = 1
X a2 + b2
X ab 3 X c2 X a + b
k(
+
)
+

c2
c2
4
ab
2c
I think it can be killed by pqr tech but it is not a nice proof.
with a = b = c we find k = 58 Let a + b + c = p, ab + bc + ca = q, abc = r. We have:
Ineq

5 X 2 2 2 2 X 3 3 3a2 b2 c2
1X
a b (a +b )+
a b
abc(a3 +b3 +c3 )+
ab(a+b)
8
4
2

5 2 2 2 2 2 2 2 2 2 X 3 3 3a2 b2 c2
1X
15
(a +b +c )(a b +b c +c a )+
a b
abc(a3 +b3 +c3 )
ab(a+b)
0
8
4
2
8
5
1
21
(p2 2q)(q 2 2p) + (q 3 3pq + 3) (p3 3pq + 3) (pq 3)
0
8
2
8
5p2 q 2
q3
9p3
9

+ 2pq 0
8
4
4
8

Follow Schur:
r
C2

(4q p2 )(p2 q)
(4q p2 )(p2 q)
1
6p
6p

X ab 3 X c2 X a + b
5 X a2 + b2
)
+

(
+
8
c2
c2
4
ab
2c
2 2
3
3
5p q 10p 10q + 16pq 9 0

setting
f (p) = 5p2 q 2 10p3 10q 3 + 16pq 9
f 0 (p) = 10pq 2 30p2 + 16q 0
p

f (p) f ( 3q) = ( q 3)(5q 2 q + 5 3q 2 + 15q q + 3q + 3q + 3 3)


by Am-Gm
p
q 3 f (p) f ( 3q) 0
"="
(a, b, c) = (1, 1, 1)
Perhaps, it is the proof which is used "pqr tech" that i said thank your proof My ineq
X

c2 (a2 b2 2ac 2bc)2 0


It is not natural, i know that.
7, Let x, y, z 0 and x + y + z = 1. Prove that
27(x3 + yz)(y 3 + xz)(z 3 + xy) 64x2 y 2 z 2
HD Its the following ineq of a well-know ineq :
(a2 + 3)(b2 + 3)(c2 + 3) 64 a + b + c = 3
Tranvanluans ineq is equivalent to :
Y 3x4
(
+ 3) 64(1)
xyz
Setting : 3x = a, 3y = b, 3z = c By am-gm, we have :
LHS(1) (a4 + 3)(b4 + 3)(c4 + 3)

1
((a2 + 3)(b2 + 3)(c2 + 3))2 64
64

Note : Its better if you think more about classical ineq before use modern tech.
C2 I proven the problem of tranvanluan but I didnt use your the problem Wink this
my proof.
Lemma:
q 2 (1 q)
r
2(2 3q)
3

and
r

(4q 1)(1 q)
6

27(x3 + yz)(y 3 + xz)(z 3 + xy) 64x2 y 2 z 2


27r3 + 27q 4 54q 2 r + 125r2 + 108qr2 + 27r 108rq 0
setting
f (r) = 27r3 + 27q 4 54q 2 r + 125r2 + 108qr2 + 27r 108rq
the first case:
81r2 54q 2 + 250r + 216rq + 27 108q 0
f 0 (r) = 81r2 54q 2 + 250r + 216rq + 27 108q 0
(4q 1)(1 q)
1
)=
(3q1)(192q 5 1808q 4 +476q 3 267q 2 +518q83) 0
6
72
the second case:
f (r) f (

81r2 54q 2 + 250r + 216rq + 27 108q 0


f 0 (r) = 81r2 54q 2 + 250r + 216rq + 27 108q 0
f (r) f (

q 2 (3q 1)(9q 6 + 192q 5 + 1061q 4 3490q 3 + 4064q 2 2160q + 432)


q 2 (1 q)
)=
0
2(2 3q)
8(2 + 3q)3

we have done Wink


8, Let a, b, c > 0 such that a2 + b2 + c2 = 3. Prove that
ab + bc + ca abc + 2
HD Put
f (a; b; c) = ab + bc + ca abc.T osupposec = mina; b; c
We have

r
a2 + b2
a2 + b2
f (a; b; c) f (
;
; c)
2
2
p
a2 + b2
c(a2 + b2 )
) + [c(a + b) c 2(a2 + b2 )] [abc
]
= (ab
2
2
(a b)2
c(a b)2
c(a b)2
p
=
+

2
2
a + b + 2(a2 + b2 )
r

c 1
c
p
= (a b)2 (
)0
2 2 a + b + 2(a2 + b2 )
r
r
a2 + b2
a2 + b2
;
; c)
f (a; b; c) f (
2
2
f (a; b; c) f (t; t; t) = 2
4

C2 Note:

3
2(abc + 2) = a2 + b2 + c2 + 2abc + 1 a2 + b2 + c2 + 3 a2 b2 c2
9, Let a1 , a2 , a3 , ..., an be n non-negative real numbers, such that a1 +a2 +.....an = 1.
Prove that
1
a1 a2 + a2 a3 + a3 a4 + ...........an1 an
4
C1 It does not work when n = 1. You need n 2. When n = 2, and a1 + a2 = 1, we
do have a1 a2 14 . The proof is easy and Ill omit it here.
Assume that for some n 2, a1 a2 + +an1 an 14 whenever a1 + +an = 1. Let
a1 +a2 + +an +an+1 = 1. WLOG, assume that an an1 . Then if A = an +an+1
we have a1 + + an1 + A = 1; hencea1 a2 + + an1 A 14 .
a1 a2 + + an1 an + an an+1 a1 a2 + + an1 (an + an+1 )

1
.
4

Hence the result is true by induction.


C2 Assume :a1 = max(a1 , ..., an )
LHS a1 (1 a1 )

1
4

10, Let x, y, z > 0 and xyz = 1. Prove that


y3
z3
3
x3
+ 4
+ 4

+1 y +1 z +1
2

x4
Let

f (x, y, z) =
Then

x3
y3
z3
3
+
+

x4 + 1 y 4 + 1 z 4 + 1 2


f (x, y, z) f (x, yz, yz) 0

But
f (x,

3t12 4t9 + 3t8 2t6 + 3t4 4t3 2t3 + 3


1

0
yz, yz) = f ( 2 , t, t) =
t
2(t8 + 1)(t4 + 1)

and inequality is prove.


here is my solution: setting : x = ab , y = ... the inequality becomes:
X
cyc

3
a3 b

a4 + b4
2

we have:
2LHS

X
cyc

X
X 2a8
X
2a3 b
a3 b
q
rq

=
( 8
)1/4
8
8
8
a +b
a8 +b8
cyc
cyc
cyc
a2 b2 + a +b
2
ab
2

2a4
3 = 2RHS
+ b4

a4

the last inequality is true since its Vascs Mr. Green ,and I saw somewhere on this forum , two nice proofs with cauchy and Jensen Smile
11, Let x, y, z are non-negative numbers which not two of them equal to 0. Prove that:
r
r
r
r
x
y
z
xyz
+
+
2 1+
y+z
x+z
y+x
(x + y)(x + z)(y + z)
<=>

x(x + y)(x + z) + 2

X
X
p
(x + y) xy(x + z)(y + z) 4(
x)(
xy)

By am-gm +schur, we hvae : +


X
X
LHS
x(x + y)(x + z) + 12xyz + 2
xy(x + y)
+x(x+y)(x+z)+12xyz+2

X
X
X
X
xy(x+y)4(
x)(
xy) =
x3 +3xyz
xy(x+y) 0

With this problem we have 2 way to solved it


The way 1: it is similar to mitdac123 sproof
The way2 (me)
X
(
a2 (b + c)).LHS 2 (a + b + c)3
let
a + b + c = 1 = p,

ab =

1 q2
, abc = r
3

We will prove
<=>

4(1 q)2 3(1 q 2 )


r
36(1 q 2 ) 9

Use
r

1 + q)2 (1 2q)
27

we will prove this ineq


12(1 q)2 (1 + q) 9(1 q) (1 + q)(1 2q)(4(1 q 2 ) 1)
<=> q 2 (4q 1)2 0
12, Prove If a, b, c > 0 then
X
(ab+bc+ca)
a3 b3 (a+b+c)(ab+c)(a+bc)(a3 +b3 +c3 )(a2 +b2 +c2 )
Assume that: a + b + c = 1
<=> f (r) = 45qr2 + 24r2 + 69q 2 r + 11r 58qr + q 4 9q + 26q 2 + 1 24q 3 0
f 0 (r) = 90qr + 48r + 69q 2 + 11 58q 0
=> f (r) f (

4q 1
1
)=
(q 1)(3q 1)(9q 2 8q + 2) 0
9
27
6

Done Smile
13, Let a, b, c > 0 Prove that:
27(a3 b3 +b3 c3 +c3 a3 )2 (a+b+c)(ab+c)(a+bc)(a3 +b3 +c3 )2 (a+b+c)3
14, Let a, b, c > 0. Prove that
(ab + bc + ca)3 (a + b + c)(a b + c)(a + b c)(a + b + c)3
Assume p = 1

4q 1
9
<=> (3q 1)(3q 2 + q 1) 0
r

15, Let a, b, c > 0. Prove that


(ab + bc + ca)4 (a + b + c)(a b + c)(a + b c)(a2 + b2 + c2 )(a + b + c)3
Assume p = 1
5q 4q 2 1
6
3
2
<=> q + 11q 7q + 1 0
r

16, Let a, b, c > 0. Prove that


a4 b2 + b4 c2 + c4 a2 (a + b + c)(a b + c)(a + b c)(a3 + b3 + c3 )
17, Let a, b, c > 0. Prove that:
9(ab + bc + ca)3 (a + b + c)4 [2(ab + bc + ca) (a2 + b2 + c2 )]
18, Let x, y, z are positive numbers. Prove that:
y2

x5
y5
z5
3(x6 + y 6 + z 6 )
+ 2
+ 2

2
2
2
+z
x +z
y +x
2(x3 + y 3 + z 3 )

Yes Smile. But its too easy Wink


By am-gm ,we have
x5
x6
q

2
2
2
y2 + z2
2 ( x +y3 +z )3
Nice! Very Happy. I have proved it by the following idea:
X
cyc

y2

x5
(x6 + y 6 + z 6 )2
3(x6 + y 6 + z 6 )
P 7 2

2
7
2
+z
(x y + x z )
2(x3 + y 3 + z 3 )

where the last inequality is true by AM-GM too. Wink


19, Let x, y, z, t R and x + y + z + t = x7 + y 7 + z 7 + t7 = 0. Find
S = t(t + x)(t + y)(t + z)
7

Setting : x + y = z t = k
=> x7 + (k x)7 = (k + t)7 t7
<=> k(x + t)(t + x k) = 0
<=> (t + z)(t + y)(t + x) = 0
=> S = 0
20, For a, b, c > 0. Prove that
a3
b3
c3
3.(ab + bc + ca)
+
+

2
2
2
2
2
2
b + c bc c + a ac a + b bc
a+b+c
P 2 2
( a )
LHS P
ab(a + b) 3abc
Assume:

4q 1
9
<=> (3q 1)2 0
p = 1; r

21, Let a, b, c > 0. We have


a3

abc
a2 + b2 + c2
4
+

3
3
+b +c
ac + ab + bc
3

Assume p = 1
<=> (3q 1)(10q 9r 3) 0
By schur we have:
9r 4q 1
(3q 1)(10q 9r 3) 2(3q 1)2 0
Done Smile
X

(a b)2 (

1
a+b+c

)0
ab + bc + ca 3(a3 + b3 + c3 )

wich is true because


3(a3 + b3 + c3 ) (a + b + c)(ab + bc + ca)
22, Prove that if a, b, c 0 then
p
X p
a a2 ab + b2 a2 ac + c2 a3 + b3 + c3
LHS 2 (a3 + b3 + c3 )2 (a b)2 (b c)2 (c a)2 RHS 2
My solution is

2
2
p
p
X p
X p

a a2 ab + b2 a2 ac + c2 =
a3 a2 b + b2 a a3 a2 c + c2 a (a3 +b3 +c3 )2
cyclic

cyclic

If are you mean that


Xp

(a3 a2 b + b2 a)(a3 a2 c + c2 a)

cyc

1X 3 2
(a a b+b2 a+a3 a2 c+c2 a) == a3 +b3 +c3
2 cyc

Then its also my proof.


23, Let a, b, c > 0. Prove that
X

(a + b)2
3
c(2c + a + b)

Let a + b + c = 3. Then
X
cyc

X
(a + b)2
3
c(2c + a + b)
cyc


(3 c)2
1 0
c(3 + c)


X 1 c
X (c 1)2 (4 + c)
X 1c
c1
0
+
0
0
c(3 + c)
c(3 + c)
4
c(3 + c)
cyc
cyc
cyc

24, Let a, b, c > 0 Prove that:


[(a + b)(b + c)(c + a)]2 abc(2a + b + c)(a + 2b + c)(a + b + 2c)
<=>
Let x =

a
b+c ; ....

a(a + b + c)
.(b c)2 0obvious
bc(a + b)(a + c)

we have
X

1
=2
1+x

or
xy + yz + zx + 2xyz = 1

1 = xy(1 + 2z) + z(x + y) xy(1 + 2z) + 2z xy

[(1 + 2z) xy 1]( xy + 1) 0


>

xy

1
1 + 2z

xyz(2x + 1)(2y + 1)(2z + 1) 1


this is Done
Put x = a1 ; y = 1b ; z =

1
c

the inequality becomes:

[(x + y)(y + z)(z + x)]2 (2xy + yz + xz)(xy + 2yz + xz)(xy + yz + 2xz)


Put m = xy + yz + xz we can rewrite this ineq as:
(x2 + m)(y 2 + m)(z 2 + m) (xy + m)(yz + m)(xz + m)
9

By AM-GM we have:
(x2 + m)(y 2 + m) = (xy)2 + m(x2 + y 2 ) + m2 (xy + m)2
Done! 25, Let a, b, c > 0. Prove that
X a
(a + b)2
2
c(2c + a + b)
b+c

Setting :x =

a+b
c ;y

c+a
b ;z

b+c
a

By cauchy-schwarz, we can prove :

P
X x2
(x + y + z)2
x
1
P

2( ) (The last ineq is easy


2+x
6+ x
2
x
<>
Setting :x =

b+c
2a ; y

c+a
2b ; z

Xa+b
c

a+b
a + b + 2c

x
x+1

a+b
2c

<=>

We have xyz 1
=> LHS

x2

X 2x
x+1

Done 26, Let a, b, c > 0. Prove that


X

Xa+b
(a + b)2

c(2c + a + b)
2c

27, Let a, b, c > 0. Prove that


Xa+b
2c

a+b

X
p

(a + b + 2c)c

28, Let a, b, c > 0. Prove that


Xa+b
4c

Xr

a
2a + b + c

29, If a, b, c > 0 then


X a(b + c)2
p
3abc(a + b + c)
2a + b + c
30, Put p = a + b + c, q = ab + bc + ca, r = abc. Prove that Setting :q =
We have :
(1 t)2 (1 + 2t)
r
27

10

1t2
3 (t

0)

So we change
(

1 t2 (1 t)2 (1 + 2t)
,
) instead of (q, r)
3
27
<=> t2 (t 1)2 0

Done 31, Let a, b, c R+ . Prove that


X p
4
a 7a2 + 9b2 (a + b + c)2
3

cyclic

By the Cauchy Schwarz Inequality, we have


X p
X
1X
1 X 2
a 7a2 + 9b2
a(7a + 9b) = (7
a +9
ab)
4
4
It suffices to prove that
X
4 X 2
1 X 2
(7
a +9
ab) (
a)
4
3
X
X
X
X
21
a2 + 27
ab 16
a2 + 32
ab
X
X
5
a2 5
ab
which is true
note that
16(7a2 + 9b2 ) (7a + 9b)2
Using Cauchy Schwarz
p
a2 + a2 + ... + a2 + b2 + b2 + ... + b2 (a+a+..+a+b+..+b) = (7a+9b) ( 7a and 9b)
I think
a 1 + a2 + .. + an

q
n(a21 + a22 + ... + a2n )

32, Let a, b, c R+ and ab + bc + ca = abc. Prove that


X

cyclic

Setting a :=
becomes

ab, b :=

bc, c :=

1
3

2
ab 1

ca then a2 + b2 + c2 = abc And the inequality

1
1
1
3
+
+

a1 b1 c1
2
P
X 1
(b 1) (c 1)
(ab + bc + ca) 2 (a + b + c) + 3
=
=
a1
(a 1) (b 1) (c 1)
abc (ab + bc + ca) + a + b + c 1

Setting p = a + b + c, q = ab + bc + ca, r = abc then we need to show that


3p2 + 7p 11q 9 0
11

By Schur ineq we have


q

p3 + 9p2
18 + 4p

It suffices us to show that




3p + 7p 11

p3 + 9p2
18 + 4p


90

p3 17p2 + 90p 162 0



(p 9) p2 8p + 18 0
Which is true by
p2 8p + 18 0andp 9 0
We have done
33, Let a, b, c > 0 such at abc = 1. Prove that:

X a b+c

2
b+c+1
cyc
Another solution. Applying AM-GM Inequality, we have:

a(b + c)
.
1
2
Thus, we only need to prove that:

a
2

2
b + c( a + 2)
sym
X

using CS inequality, we have:


X
sym

(a + b + c)2
(a + b + c)2
a
pP P
P
p P P

P p
b + c( a + 2)
2 a b + c + a a(b + c)
2 2( a)( ab) + ( a)( ab(a + b))

Setting t2 = a + b + c; u2 = ab + bc + ca. Rewrite inequality, we need to prove:


p
3

2t 2 2u t2 u2 3
By AM-GM ineq, u2 <= t2 /3. Thus,

3
2
5 6 8 3 5 8 4
2 2
( 2t 2 2u) t u + 3 t
t + t +30
3
3
3
Using AM-GM:
r

122

5 6 8 4
8 3 5
24 t
t + t + 3 24
t (t 3)

37
3
3
3
3
12

34, Let a, b, c > 0 and a3 + b3 + c3 = 3. Prove that


a2
b2
c2
3
+
+

2
2
2
3a
3b
3c
2
Are you sure that this inequality is very hard. Huh?
Using AM-GM inequality we have
a3
a3
a3
a3
a2
=
= q
=p

2
2
2 3a2
3a
a(3 a )
2
a2 (3 a2 )2
2. a2 . 3a
2 . 2
35, Let a, b, c > 0 such that a + b + c + 1 = 4abc. Prove that
1
1
1
3
+
+

a4 + b + c b4 + c + a c4 + a + b
a+b+c
The inequality comes from: If a, b, c > 0 such that a + b + c 3 then
1
1
1
3
+
+

a4 + b + c b4 + c + a c4 + a + b
a+b+c
This is a very known result.
We have :
a4 + b + c =

(b2 )2
(a2 )2
(c2 )2
(a2 + b2 + c2 )2
+ 3 + 3
1
b
c
1 + b3 + c3
3 + 2(a3 + b3 + c3 )
1

a4 + b + c
(a2 + b2 + c2 )2

We have to prove that :


3
3 + 2(a3 + b3 + c3 )

()
(a2 + b2 + c2 )2
a+b+c
with : a, b, c > 0 and a + b + c + 1 = 4abc a + b + c = p; ab + bc + ca = q; abc = r
We have :

4
4abc = a + b + c + 1 4 abc abc = r 1
() (p2 3q)2 + (q 2 3p) + 2(q 2 3pr) 0
We have:
(p2 3q)2 + (q 2 3p) + 2(q 2 3pr) (p2 3q)2 + 3(q 2 3pr) 0
We have done Mr. Green 36, Let a, b, c be nonnegative real numbers, no two of which
are zero. Prove that
s
s
s
8b(c + a) + 9ca
8c(a + b) + 9ab
8a(b + c) + 9bc
+
+
5
(2b + c)(b + 2c)
(2c + a)(c + 2a)
(2a + b)(a + 2b)

13

37, Let a, b > 0. Prove that

b
a
+
1
2
2
+ 3b
b + 3a2

a2

Well, well. The problem is rather good although it only contains 2 variables a, b. Interesting enough. Here is my solution. We set that
x=

a2

a
b
and y =
2
2
+ 3b
b + 3a2

that lead to the condition:


8(xy)2 + x2 + y 2 = 1
Assume that
4xy (x + y)2 1
then we have
4(xy)2 1
( from the above condition, you can check it properly). As a result, we have the contradiction.
By Holder
2

b
a

+
(a(a2 +3b2 )+b(b2 +3a2 )) (a+b)3 == a(a2 +3b2 )+b(b2 +3a2 ).
a2 + 3b2
b2 + 3a2
38, Let a, b, c be positive real numbers. Prove that

3
bc + 4ab + 4ac
ca + 4bc + 4ba
ab + 4ca + 4cb
+
+

b+c
c+a
a+b
2 2

b+c
+
a

c+a
+
b

Here is my solution for it


Since
s
r
p
4(bc + 4ab + 4ac)
16a
4bc
16a + b + c
=
+

b+c
b + c (b + c)2
b+c
It suffices to prove that
3 X

b+c X

16a + b + c
b+c

Squaring both sides, we have


!2
r
r

X b+c
Xb+c
X (a + b)(a + c) X b + c
X a + bc

=
+2

+2
a
a
bc
a
bc
=

Xb+c
a

+2

X a
Xb+c
X a
+6
+4
+6
a
b+c
bc
14

a+b
c

X 1 1
X a
X a
a
+
+4
+68
+6
b
c
b+c
b+c

and
X

16a + b + c
b+c

!2
=

X 16a + b + c

X 16a + b + c
b+c

b+c
+

s
+2

X  16a + b + c

= 18

a+c
a
+ 54
b+c

(16a + b + c)(16b + c + a)

(a + c)(b + c)

16b + c + a
b+c

Hence, it suffices to prove that


 X

X a
9
a
8
+ 6 18
+ 54
2
b+c
b+c
3
a

b+c
2
This is Nesbitts Inequality. Equality holds if and only if a = b = c. We have done.

39, Let a, b, c 0; ab + bc + ca = 1. Prove that


X

1
5

a+b
2

I have 3 nice solution: 1 use Cauchy-Schwarz Smile


I knew thems. I find a solution that uses Cauchy Schwarz. Solution: If
c = 0 LHS =
If a, b, c 6= 0. Setting x =

ab
c ;y

ca
b ;z

LHS

1 1
1
5
+ +

a b a+b
2

bc
a

(ab + bc + ca)2
5

2abc
2

40, Let xyz = 1; x, y, z > 0. Prove that:


3
1
1
1

+p
+
2
2
1 + x2
1
+ z2
1+y
Using Cauchy-Schwarz Inequality,we have
s
Xr a
Xp
X
X
X
1
1
(
)2 = (
a(a + c).
)2 (
a2 +
ab)(
)
a+b
(a + b)(a + c)
(a + b)(a + c)

15

then applying p, q, r.
41, Let a, b, c be nonegative real numbers such that a2 + b2 + c2 = 1. Prove that

a
c
b
3
+
+

2
1 + ca
1 + bc
1 + ab

Have you got a nice solution for it?


Here is my solution, MrThuan!
Applying Cauchy Inequality, we have
X

a
1 + bc
We have the following result:
X a

2

1 + bc

 X  X
a

a
1 + bc

a
(a + b)(a + c)

Actually, we have
LHSRHS =
X

X a((a + b)(a + c) a2 b2 c2 bc)


(1 + bc)(a + b)(a + c)

X ab(a b) ca(c a)
=
(1 + bc)(a + b)(a + c)

X
X
ab(a b)
ab(a b)
ab(a b)2 (1 c2 )

=
0
(1 + bc)(a + b)(a + c)
(1 + ac)(b + a)(b + c)
(a + b)(b + c)(c + a)(1 + ac)(1 + bc)

Hence, it is sufficient to prove that


 X  X
a

a
(a + b)(a + c)

(a + b)(b + c)(c + a)

9
4

8
(ab + bc + ca)(a + b + c)
9

By AM - GM Inequality, it is true.
42, Let a, b, c > 0. Prove that

(a + b)

a b
+
b
c

1
+ 2
b

a
+
b

r !
b
c

Divide by b to get
a
a b
1
2 a
( + 1)( + ) + 2 ( +
b
b
c
b
b b
Let

b
)
c

a
b
1
= x, = y, and = z
b
c
b

Then we have to show


(x + 1)(x + y) + z 2 2z(x +
16

y)

x2 + x + y + xy + (z x y)2 (x + y)2

x + xy + (z x y)2 2x y

x(1 y)2 + (z x y)2 0


Which is obviously true. Equality holds when b = c and a + b = 1.
43, Let x, y, z, k > 0. Prove that
x2 + y 2
y2 + z2
z 2 + x2
3
+
+
(x + y + z k)
z+k
x+k
y+k
2
2(x + y + z + 3k)(
= [2(z +k)+2(x+k)+2(y +k)](

x2 + y 2
y2 + z2
z 2 + x2
+
+
)
z+k
x+k
y+k

x2
y2
y2
z2
z2
x2
+
+
+
+
+
)
z+k z+k x+k x+k y+k y+k

4(x + y + z)2
by Cauchy-Schwarz. Now let x + y + z = a then we only need to prove
4a2 3(a + 3k)(a k)
(a 3k)2 0, which is obviously true.
using AM-GM :
k+z
x2 + y 2
+
x+y
z+k
2
44, Let a, b, c > 0. Prove that
3
ab + bc + ca a4 + b4 + c4 + .
4
a4 +

1
a2
4

and cyclic. So
3
a2 + b2 + c2 ab + bc + ca.
4
p
(a4 + b4 + 1/8 + 1/8)/4 4 a4 b4 /64

a4 + b4 + c4 +

The result follows


45, Let a, b, c > 0 and a + b + c = 3. Prove that

a2 + 1 + 2b2
b2 + 1 + 2c2
c2 + 1 + 2a2
+
+
2( ab + bc + ca)
a+1
b+1
c+1
p
p
p
a2 + 1 + 2b2 b2 + 1 + 2c2 c2 + 1 + 2a2
+
+
2( a2 + b2 + b2 + c2 + c2 + a2 )
a+1
b+1
c+1
17

a2 + b2 2ab
and
b2 + 1 2b
so
a2 + 1 + 2b2 2ab + 2b = 2b(a + 1)
So then
LHS 2(a + b + c)
So we have to prove that:
a+b+c

ab +

bc +

ca, Which is trivial

46, Let a, b, c > 0. Prove that




1
1
1
1
1
1
1
+ 3
+ 3
<
+ 2
+ 2
(a3 1)2
(b 1)2
(c 1)2
9 a2 (a 1)2
b (b 1)2
c (c 1)2
Nice!
All we need to do is to prove
9a2 (a 1)2 (a3 1)2
It deduces to prove that
9a2 (a2 + a + 1)2 ()
Moreover, we use the identity
A2 B 2 = (A + B)(A B)
() becomes (a 1)2 (a2 + 4a + 1) 0 which is trivial. Finally, the equality cannot
hold obviously so we have the strict inequality. Done!
47, Given a, b, c 0. Prove that:
r
r
r
b2 bc + c2
a2 ab + b2
c2 ca + a2
2(ab + bc + ca)
+
+
+
4
2
2
a + bc
c + ab
b2 + ac
a2 + b2 + c2
If its trues, itll be a strong ineq Smile
This inequality is valid, nguoivn and it follows from applying AM-GM as follow:
r
b2 bc + c2
2(b2 bc + c2 )
2(b2 bc + c2 )
2
=
2
2
2
a + bc
(a + bc) + (b bc + c )
a2 + b2 + c2
48, Given that a, b, c 0 and a + b + c = 3. Prove that
a + ab + 2abc

18

9
2

Youre right, shaam. We can show that


a + ab + 2abc

9
2

as follow: Replacing b = 3 a c, then we have to prove


a + a(3 a c) + 2ac(3 a c)

9
,
2

or equivalently,
f (a) = (2c + 1)a2 + (2c2 5c 4)a +

9
0.
2

We see that f(a) is a quadratic polynomial of a with the highest coefficient is positive.
Moreover, its disciminant is
= (2c2 5c 4)2 18(2c + 1) = (2c 1)2 (c2 4c 2) 0, as0 c 3.
Therefore, f (a) 0 and our proof is completed. Equality holds if and only if a =
1
3
2 , b = 1, c = 2 .

2a + ab + abc = a(2 + b(c + 1)) a(2 + (

b+c+1 2
) )9
2

a(8 + (b + c + 1)2 ) = a(8 + (5 a)2 ) = 33a 10a2 + a3 36


a3 + 10a2 33a + 36 = (4 a)(3 a)2 0
which is obviously true. Equality iff a = 3, b = 1, c = 0.
Dear Can, thank you for your proof Smile
Recently, I have found a proof based on AM-GM Inequality to this inequality. Here it
is:
9
a + ab + 2abc
2
9
ab(1 + 2c) a
2


1
9
2ab
+c a
2
2


2
b + 21 + c
9
2a
a
2
2

2
a
1
9
3a+
a
2
2
2
which is equivalent of proving:
a(7 2a)2 36 8a
19

But
36 8a a(7 2a)2 = (4 a)(2a 3)2 0
Since a 3, proof is complete.
49, Given a, b, c > 0 and abc = 1. Prove that
(a2 + 1)(b2 + 1)(c2 + 1)

(a + 1)(b + 1)(c + 1)

r
3

(a + b)(b + c)(c + a)
8

now and future, I will try to solve problem,i dont creat more ineq.Thank you to all
people.
r
3
x2 + 1
3 x + 1

x+1
2
(x 1)4 (x2 + x + 1) 0
and
(a2 + bc)(1 +

bc
) (a + b)2
c

so
(a2 + bc)(b2 + ca)(c2 + ab) abc(a + b)(b + c)(c + a)
(a3 + 1)(b3 + 1)(c3 + 1) (a + b)(b + c)(c + a)
The stronger is trues and very easy:
(1 + a)3 (1 + b)3 (1 + c)3 64(a + b)(b + c)(c + a)
Y
(1 + a)16 =

!8
Y
(1 + a)(1 + b)
=

!8
Y
(1 + ab + a + b)

cyc

cyc

cyc

!4
Y
(1 + ab)(a + b)

88

cyc

Y
= 88 (a + b)4
cyc

!2
Y
(1 + ab)(1 + ac)
=
cyc

!2
= 88

Y
(a + b)4

Y
(1 + a + a(b + c))

cyc

cyc

810

Y
Y
(a + b)5 (1 + a),
cyc

which gives your inequality.


My full proof:
(a + b)(1 + ab)

(1 + a)2 (1 + b)2
4

Because of:
(1 + ab)(1 + bc)(1 + ca) = (1 + a)(1 + b)(1 + c)

20

cyc

(with abc = 1)we have q.e.d.


50, Given a, b, c 0. Prove that:
1
1
1
5
+ 2
+ 2

a2 + b2
b + c2
c + a2
2(ab + bc + ca)
We have :
b2

1
4
1
+ 2
2
2
2
+c
c +a
2c + (a + b)2 2ab

a b c; assumea + b = 1. We have : c(1 c) x = ab


We have to prove that:
(x + c)(

1
4

1
4
5
+
)
1 2x 2c2 2x + 1
2

f (x) = 40x2 + x(24c2 20c + 30) + 4c3 10c2 + 10c 5 0


We have :

f (x) = 80x + 24c2 20c + 30


00

f (x) = 80 < 0
0
0 1
f (x) f ( ) = 24c2 20c + 10 > 0
4

We have :

1
f (x) f ( ) = c((2c 1)2 + 4) 0
4

We have done
Assume : c = min{a; b; c} We have :
a2 + b2 x2 + y 2
b2 + c2 y 2
c2 + a2 x2
ab + bc + ca xy
With :

c
c
x = a + ;y = b +
2
2

We have to prove that :


xy
x2 + y 2
5
+

x2 + y 2
xy
2
By AM-GM we have :

x2

xy
xy
3(x2 + y 2 )
5
x2 + y 2
x2 + y 2
=( 2
)+

+
+
2
2
+y
xy
x +y
4xy
4xy
2

21

We have done.
51, Given a, b, c 0. Prove that
(a3 b3 + b3 c3 + c3 a3 )[(a + b)(b + c)(c + a) 8abc] abc(a b)2 (b c)2 (c a)2
[c(ab)2 +b(ac)2 +a(bc)2 ][

1 1 1
ab bc ca 2
(a b)2 (b c)2 (c a)2
+
+
]

(
+
+
)
=
c3 b3 a3
c
a
b
a2 b2 c2

and we get
(a3 b3 + b3 c3 + c3 a3 )[(a + b)(b + c)(c + a) 8abc] abc(a b)2 (b c)2 (c a)2
52,
(a3 b3 + b3 c3 + c3 a3 )[(a + b)(b + c)(c + a) 8abc] kabc(a b)2 (b c)2 (c a)2
kmax = 9.
Replacing a, b, c by

1 1 1
a, b, c

respectively, then our inequality becomes

(a3 + b3 + c3 )[(a + b)(b + c)(c + a) 8abc] 9(a b)2 (b c)2 (c a)2


Now, assume that c = min{a, b, c} then we have
a3 + b3 + c3 a3 + b3
(a+b)(b+c)(c+a)8abc = 2c(ab)2 +(a+b)(ac)(bc) (a+b)(ac)(bc),
and
(a b)2 (a c)2 (b c)2 ab(a b)2 (a c)(b c)
Therefore, we can reduce our inequality to
(a3 + b3 )(a + b)(a c)(b c) 9ab(a b)2 (a c)(b c),
or
(a3 + b3 )(a + b) 9ab(a b)2
which is equivalent to
(a2 4ab + b2 )2 0
which is trivial
53, Let a, b and c are non-negative numbers such that ab + ac + bc 6= 0. Prove that:
a2

1
1
1
9
+ 2
+ 2

+ ab + bc b + bc + ca c + ca + ab
(a + b + c)2

X
(
a2 + ab + bc)(
cyc

1
1
1
+
+
) (1 + 1 + 1)2 = 9
a2 + ab + bc b2 + bc + ca c2 + ca + ab

22

a2

1
1
1
9
+ 2
+ 2

+ ab + bc b + bc + ca c + ca + ab
(a + b + c)2

54, Actually, we can prove by Cauchy-Schwarz that


X
1
3

ka2 + ab + bc
k+2
for any k 0.
55, Given a, b, c 0 and 3(ab + bc + ca) + 2abc = 27. Prove that:
a+b+c

9
2

ab + bc + ca

27
4

9
2
3(a + b + c) 2(ab + bc + ca)
ab +

Nice solution,arqady, and

bc +

ca

9
2
also nice and easy.Try it.Sorry for my bad english. But 3(a + b + c) 2(ab + bc + ca),
we can use schur.
ab +

bc +

ca

56, Given a, b, c > 0 and abc = 1. Prove that:


b2
c2
(a2 c + b2 a + c2 b 3)2
a2
+
+
a+b+c+
b
c
a
ab + bc + ca
It is equivalent to
X

(a3 3a2 b + 6a2 c 4abc) 0, which is obvious.

cyc



1 1 1
(a b)2
(b c)2
(c a)2
+ +
+
+
b
c a
b
c
a

2
ab bc ca

+
+
b
c
a

57, Given a, b, c > 0. Prove that:


 2

a
b2
c2
15(a2 + b2 + c2 )
2
+
+
+ 3(a + b + c)
b
c
a
a+b+c
I had a nice and very short proof by Am-Gm
58, Given a, b, c 0. Prove that:
(a + 2b)2 (b + 2c)2 (c + 2a)2 27abc(a + 2c)(b + 2a)(c + 2b)
23

Thanks you.Here is my proof.

p
(a+2b)(b+2c) = ab+bc+bc+b2 +b2 +bc+bc+ca+ca = b(a+2c)+b(c+2b)+c(b+2a) 3 3 b2 c(a + 2c)(c + 2b)(b
and similar,we have
(a + 2b)2 (b + 2c)2 (c + 2a)2 27abc(a + 2c)(b + 2a)(c + 2b)
59, Its weaker than the well-known:
(a + 2b)2 (b + 2c)2 (c + 2a)2 27(ab + bc + ca)3
60, Given a, b, c > 0. Prove that:
(a + b)(b + c)(c + a)
(a2 + b2 )(b2 + c2 )(c2 + a2 )

8abc
(a2 + bc)(b2 + ca)(c2 + ab)
(a2 + b2 )(b2 + c2 )(c2 + a2 ) (a2 + bc)(b2 + ca)(c2 + ab)
= a2 b2 (a2 +b2 )+b2 c2 (b2 +c2 )+c2 a2 (c2 +a2 )(a3 b3 +b3 c3 +c3 a3 )abc(a3 +b3 +c3 )
a2 b2 (a2 + b2 ) + b2 c2 (b2 + c2 ) + c2 a2 (c2 + a2 ) (a3 b3 + b3 c3 + c3 a3 ) 3a2 b2 c2
P 2
c (a b)2 (a + b)2
a2 b2 (a b)2 + b2 c2 (b c)2 + c2 a2 (c a)2
+
=
2
2
I think this way can prove this problem.
p
(a2 +bc)(b+c) = a2 b+a2 c+b2 c+c2 b = c(a2 +b2 )+b(a2 +c2 ) 2 bc(a2 + b2 )(c2 + a2 ) . . .
Pn
61, ai > 0, i=1 ai = 1. Prove:
n
X
i=1

If f (x) =

x
1x

a
i

1 ai

then, f 00 (x) = 4x
4

(1x)5

n
n1

, which is positive for 0 < x < 1.

But arqady,We only prove by cauchy,


X

X
X
1
ai

1 ai
1 ai
1 ai

X
n2
twitht =
1 ai
P
1 ai
2

We can find minimum nt t very easy.


If youre going to use Cauchy, the following is much easier Smile
X
X
X
X
a
a
i )(
i )(
(
ai (1 ai )) (
ai )3 = 1
1 ai
1 ai

24

This, combined with

ai = 1, yields
X
a
1
P
i )2
(
1 ai 2
1 ai
n
n1

That the right side is not less than


P
P
(Use n( a2i ) ( ai )2 = 1.)

is trivial.

62, Let a, b, c are three positive real numbers. Prove that


2

(a + b c)
c2

+ (b + a)

(b + c a)
a2

+ (b + c)

(c + a b)

b + (c + a)

3
5

I have three keys for this ineq


because the ineq is homogenuous, we can assume that a + b + c = 1 ineq

(1 2a)

a2 + (1 a)

X (3a 1)(3a 2)
1
0
0
2
5
a2 + (1 a)

we can apply Chebyshev inequality ,we have


LHS

X
3a 2
1
(3a + 3b + 3c 3)(
2) 0
2
3
a + (1 a)

(a + b + c = 1).
63, Let a, b, c are real numbers. Prove that
(a + b)4 + (b + c)4 + (c + a)4

4
(a + b + c)4
7

by holder inequality, we get:



X
4
4
(a + b) (1 + 1 + 1)(1 + 1 + 1)(1 + 1 + 1) (2(a + b + c))
so

16
4
(a + b + c)4 (a + b + c)4
27
7
64, Given the reals a, b, c. Prove that:
X

(a + b)

(a + b)4 + (b + c)4 + (c + a)4

4 4
[a + b4 + c4 + (a + b + c)4 ]
7

When does equality occurs? Smile


65, Given the reals a, b, c. Prove that:
X a2
(a + b + c)(a2 + b2 + c2 )

b
ab + bc + ca
25

P
2
X a2
X (a b)2
a2 + b2 + c2
(a b)

( 2a + b) (a+b+c)(
1)
(a + b + c)(
)
b
ab + bc + ca
b
2(ab + bc + ca)
X
a+b+c
2 1

(a b) (
)0
b 2(ab + bc + ca)
use SOS method we have.
After expand, this ineq is only:
x2 + y 2 + z 2 xy + yz + zx
with x = ab2 ; ...
66, Let a, b, c be real numbers. Prove that
(b + c)4 + (c + a)4 + (a + b)4

4 4
(a + b4 + c4 + (a + b + c)4 )
7

Equality holds if and only if a = b = c or a = 3b = 3c or any cyclic permutations.


Yes, its equivlent to:
X
(a + b + 2c)2 (a b)2 0
67, Given a, b, c > 0 and a + b + c = 3. Prove that:

b2
c2
a2
+
+ 2 + 3 abc 6
b
c
a
With this problem, we only use cauchy-schwarz, AM-GM.
After use Cauchy-Schwarts and Am-Gm, we only obtain an old (and easy) result:
a2 + b2 + c2
8abc
+
2
ab + bc + ca (a + b)(b + c)(c + a)
The similar and harder problem: (with the same condition)
Prove that:
r

a2
b2
c2
3( +
+ ) + ( 2 1). abc 2 + 2
b
c
a
My full proof: (it seems not use Cauchy-Schwarts Embarassed )
Using a well-known result:
a2
b2
c2
(a + b + c)(a2 + b2 + c2 )
+
+

b
c
a
ab + bc + ca
(we can prove it easily by Am-Gm)
So,
a2
b2
c2
3(a2 + b2 + c2 )
+
+

b
c
a
ab + bc + ca
Besides, we have:

8abc
abc
(a + b)(b + c)(c + a)
26

<=> (a + b)2 (b + c)2 (c + a)2

64
abc(a + b + c)3
27

(trues again by Am-Gm)


So, we obtain:
8abc
a2 + b2 + c2
+
2
ab + bc + ca (a + b)(b + c)(c + a)
68, Given a, b, c > 0. Prove that:
(b c)2
(c a)2
(a b)2 (b c)2 (c a)2
(a b)2
+
+

c2
a2
b2
3a2 b2 c2
Its an old problem.
Its equivalent to:
X
X
abc(
a3 + 3abc
ab(a + b)) 0 (obviously trues)
if abc(a b)(b c)(c a) 0. Prove that
(b c)3
(c a)3
(a b)3
3abc
+
+

3
3
3
a
b
c
(a b)(b c)(c a)
if abc(a b)(b c)(c a) 0. Prove that:
(b c)3
(c a)3
(a b)3
3abc
+
+

a3
b3
c3
(a b)(b c)(c a)
And in fact, I proved the stronger:
(b c)2
(c a)2
(a b)2 (b c)2 (c a)2
(a b)2
+
+

c2
a2
b2
a2 b2 c2
(indeed, this stronger is also my old result)
Here is my full hint:
Let x = ab(a b); ... The first ineq becomes: x2 + y 2 + z 2 (x + y + z)2
X
X
<=> 2(xy + yz + zx) 0, <=> 2abc(
a3 + 3abc
ab(a + b)) 0
PS: Besides, I think this ineq is only trues with a, b, c 0
But, setting x =

bc
a ,y

ca
b ,z

ca
b .

We get x + y + z + xyz = 0 We have

x2 + y 2 + z 2 = (x + y + z)2 2(xy + yz + zx) = x2 y 2 z 2 2(xy + yz + zx)


We have:
a, b, c 0, wehavexy + yz + zx 0
and x2 + y 2 + z 2 x2 y 2 z 2 or
(b c)2
(c a)2
(a b)2
(a b)2 (b c)2 (c a)2
+
+

2
2
2
a
b
c
a2 b2 c2
27

and the inequalities is true if and only xy + yz + zx 0, and then


(b c)2
(c a)2
(a b)2
(a b)2 (b c)2 (c a)2
+
+

2
2
2
a
b
c
a2 b2 c2
We have xy + yz + zx k(x + y + z)2 , k =

1
3

is the best. And

x2 + y 2 + z 2 2(xy + yz + zx) x2 y 2 z 2

r2
2(x + y + z)2
=
3
3

So in inequalities
(c a)2
(a b)2
(a b)2 (b c)2 (c a)2
(b c)2
+
+

2
2
2
a
b
c
ka2 b2 c2
and k =

1
3

is best. 69, Finally, if (a b)(b c)(c a)0. Prove that


(

bc ca ab 2
27
+
+
) >
a
b
c
17

70, Prove that:


2(a2 + b2 ) 2(b2 + c2 ) 2(c2 + a2 )
(a c)2
+
+

3
+
(a + b)2
(b + c)2
(c + a)2
a2 + b2 + c2
We can write it into:

X (a b)2
(a c)2

(a + b)2
a2 + b2 + c2

Using CS, we have:


LHS

(a c)2
4(a c)2
(a c)2
+
P
RHS
2
2
2
(a + b) + (b + c)
(a + c)
(a + b)2

(By AM-GM).
71, Give a, b, c > 0 and ab + bc + ca = 3. Prove that:
1
p

1 + (a +

b)3

1
1
3
+p
+p

3
3
2
1
1 + (b + c) 1
1 + (a + c) 1

I thinks this is easy problem.


1 + x3 = (1 + x)(1 x + x2 ) (1 +

x2 2
)
2

let x = a + b, x = b + c, x = c + a ,we get:


p
(a + b)2
(a + b)2
1 + (a + b)3 1 1 +
1=
2
2

28

And finally,we need to prove :


1
1
1
3
+
+

(a + b)2
(b + c)2
(c + a)2
4
or

1
1
9
1
+
+

2
2
2
(a + b)
(b + c)
(c + a)
4(ab + bc + ca)

This is Iran TST96.


72, Let a, b, c be positive numbers such that: ab + bc + ca + abc = 4. Prove that:

a
b
c
a+b+c

+
+

c+a
b+c
a+b
2

By Horder we have :
X a
X

(
)2 .(
a(b + c)) (a + b + c)3
b+c
We have to prove that :
a + b + c ab + bc + ca
With a, b, c be positive numbers such that: ab + bc + ca + abc = 4 It is VMO 1996.
We have done 73, Given a, b, c 0. Prove that:
b2
c2
ab + bc + ca
a2
+
+
+ 2
2
2
2
2
2
2
2
a + ab + b
b + bc + c
c + ca + a
a + b2 + c2
P
ab + ac + bc X
(ab + ac + bc) (a5 c a3 c2 b)
a2
Q
2 2

= 2
.
a + b2 + c2
a2 + ab + c2
(a + b2 + c2 ) (a2 + ab + b2 )
cyc
74, Given a, b, c 0 and a + b + c = 3. Prove that
a2 b + b2 c + c2 a 4
Let x = a/3, y = b/3, z = c/3 x + y + z = 1
a2 b + b2 c + c2 a 4 x2 y + y 2 z + z 2 x

4
27

75, Let a, b, c > 0 such that ab + bc + ca = 3. Prove that


p

3 (a + b)(b + c)(c + a) >= 2( a + b + b + c + c + a) 6 2


F First , we prove the left ineq . Setting :x = ab; y = bc; z = ca => x + y + z = 3
By cauchy-schwarz , we need to prove :
9(x + y)(y + z)(z + x) 8(x + y + z)(xy + yz + zx)(Right)
F, Next, we prove the right ineq : By Am-Gm , we need to prove :
(a + b)(b + c)(c + a) 8
29

<=> 3(a + b + c) abc 8(Right because ab + bc + ca = 3)


76, Let x, y, z > 0. Prove that :
p
p
p
3( x(x + y)(x + z) + y(y + z)(y + x) + z(z + x)(z + y))2 4(x + y + z)3
By Cauchy-Schwarz ineq , we have :
X
LHS 3(x + y + z)(
x2 + yz + zx + xy)
Then we prove that :
4(x + y + z)2 3[(x + y + z)2 + xy + yz + zx]
X

(x y)2 0
77, Let a, b, c 0 such that a + b + c = 3. Prove that:
(a2 b + b2 c + c2 a) + 2(ab2 + bc2 + ca2 ) + 3abc 12
I use this ineq :
a, b, c 0; a + b + c = 3
a2 b + b2 c + c2 a + abc 4
Make the similar ineq and we have done .
Maybe this ineq can be stronger .
78, Let a, b, c be positive real numbers such that a + b + c = abc. Prove that:
p
p
p
p
(1 + a2 )(1 + b2 )+ (1 + b2 )(1 + c2 )+ (1 + c2 )(1 + a2 ) 4+ (1 + a2 )(1 + b2 )(1 + c2 )
Setting :a =

1
x2 ; b

1
y2 ; c

1
z2

By Am-Gm we have :

(x2 + z 2 )(y 2 + z 2 ) (xy + z 2 )2


X
<=>
a2 b + b2 a 6abc
79, Let a, b, c 0. Prove that
(a + b + c)3
+
abc

r
3

ab + bc + ca
28
a2 + b2 + c2

By AM-GM Inequality, we get


3abc

(ab + bc + ca)2
a+b+c

It suffices to prove that


3(a + b + c)4
+
(ab + bc + ca)2

r
3

30

ab + bc + ca
28
a2 + b2 + c2

Notice that (by AM-GM Inequality for two numbers)


r
2(a + b + c)
(a + b + c)2
ab + bc + ca
p
+ 3 2
6
3(ab + bc + ca)
a + b2 + c2
3 (ab + bc + ca)2 (a2 + b2 + c2 )
and (AM-GM Inequality for three numbers)
(ab + bc + ca) (ab + bc + ca) (a2 + b2 + c2 )
Thus

(a + b + c)2
+
3(ab + bc + ca)

1
(a + b + c)6
27

2(a + b + c)
ab + bc + ca
=2
q
2
2
2
a +b +c
1
(a + b + c)6
3 6 27

It suffices to prove that


3(a + b + c)4
(a + b + c)2

+ 26
(ab + bc + ca)2
3(ab + bc + ca)
It is true because
9(a + b + c)2
(a + b + c)2
26(a + b + c)2
(a + b + c)2
3(a + b + c)4

=
+

+26
(ab + bc + ca)2
ab + bc + ca
3(ab + bc + ca) 3(ab + bc + ca)
3(ab + bc + ca)
We have done.
80, Let a, b, c 0. Prove that
a
b
c
3
+
+

b(a2 + 2b2 ) c(b2 + 2c2 ) a(c2 + 2a2 )


ab + bc + ca
I will change the problem into
a :=

1
1
1
, b := , c := , the inequality becomes
a
b
c
X

b2
3

2
2
c(2a + b )
a+b+c

By the Cauchy Schwarz Inequality, we get


P
( a2 )2
LHS P 2
b c(2a2 + b2 )
It suffices to prove that
X
X
X
(
a2 )2 (
a) 3
b2 c(2a2 + b2 )

a5 +

ab4 + 2

a3 b2 + 2

31

a2 b3 2

a4 b + 4

a2 b2 c

By the AM-GM Inequality, we get


X
X
X
X
a5 +
a3 b2 =
a3 (a2 + b2 ) 2
a4 b
X

X
X
X
ab4 +
a2 b3 =
(ab4 + c2 a3 ) 2
a2 b2 c
X
X
X
X
X
a3 b2 +
a2 b3 =
a3 (b2 + c2 ) 2
a3 bc 2
a2 b2 c
Adding up these inequalities, we get the result.
81, Let a, b, c > 0. Prove that
1 1 1
3a
3b
3c
+ + 2
+
+
a b
c
a + 2bc b2 + 2ca c2 + 2ab
the inequality is equivalent to :
X

(a b)2 (

c(a b)2 + ab(a + b + c)


)0
(a3 + 2abc)(b3 + 2abc)

The inequality is equalivent with





(ab + bc + ca) a2 + 2bc b2 + 2ca c2 + 2ab
X


abc
3a b2 + 2ca c2 + 2ab
By setting q = ab + bc + ca, r = abc and assume a + b + c = 1 we have
X


3a b2 + 2ca c2 + 2ab = 6q 2 27rq



a2 + 2bc b2 + 2ca c2 + 2ab = 2q 3 + 27r2 18rq + 4r
So the inequality becomes


q 2q 3 + 27r2 18rq + 4r r 6q 2 27rq



3
27a2 b2 c2 + 2abc + (ab + bc + ca) 12abc (ab + bc + ca) 0

So we need to prove that


3

27a2 b2 c2 + 2abc + (ab + bc + ca) 12abc (ab + bc + ca)


But its easy, by AM-GM we have
2

abc = abc (a + b + c) 3abc (ab + bc + ca)


3

27a2 b2 c2 + abc + (ab + bc + ca) 9abc (ab + bc + ca)


Setting
a :=

1
1
1
, b := , c := , the inequality is equivalent to
a
b
c
32

a 3abc

X
2a2

X a(a2 bc)
2a2 + bc

1
+ bc

X
a3

a
2a2 + bc
By the Cauchy Schwarz Inequality, we get
P
X
( a2 )2
a3
P 3
2a2 + bc
2 a + 3abc
3

It suffices to prove that


X
X
X
3(
a2 )2 (
a)(2
a3 + 3abc)
Assume a + b + c = 1, setting q = ab + bc + ca, r = abc, the inequality becomes
3(1 2q)2 2 6q + 9r
Since q 2 3r, it suffices to show
3(1 2q)2 2 6q + 3q 2
3 12q + 12q 2 2 6q + 3q 2
(1 3q)2 0
82, Let a, b, c 0 such that a + b + c = 3. Prove that
r
r
r
a3
b3
c3
+
+
1
a2 + 8b2
b2 + 8c2
c2 + 8a2
By Am-Gm ,we have :
LHS

P
6( a2 )2
6a2
P

3(b + c)a3 + 4a4 + 8b2 a2


9a + a2 + 8b2

We need to prove :
X
X
X
6(
a2 )2 3
(b + c)a3 + 4(
a2 )2
X
X
<=> 2(
a2 )2 3
(b + c)a3
X
<=>
(a b)2 (a2 ab + b2 ) 0
83, Let a, b, c 0. Prove that
(a + b + c)3
+
abc

r
3

ab + bc + ca
28
a2 + b2 + c2

33

by AM-GM we have :
r
3

a2 + b2 + c2
a2 + b2 + c2
2

+
ab + ac + bc
3(ab + ac + bc) 3
=

so :

r
3

3(a + b + c)2
ab + ac + bc

ab + ac + bc
ab + ac + bc

2
2
2
a +b +c
3(a + b + c)2

now we have to prove :


(a + b + c)3
ab + ac + bc
28
+
abc
3(a + b + c)2
since the inequality is homogenous assume that : a + b + c = 1
so we have to prove :
1
1
+ (ab + ac + bc) 28
abc 3
by schur we have :
1
9

abc
ab + ac + bc
so we have to prove :
(ab + ac + bc)2 + 27 84(ab + ac + bc)
wich is true because ab + ac + bc 13 .
WLOG assume a + b + c = 3, put t = ab + bc + ca => a2 + b2 + c2 = 9 2t
LSH =

27
t
27
t
27
3
+ p

+
3
a2 b2 c2 (by AM-GM)
3
abc
abc
3
abc
t.t.(9 2t)

1
2 1
5
2
1
3
(3 a2 b2 c2 + 2
) + (27 )
+ 27 = 28
3
abc
3 abc
3
3
(by AM-GM) =>q.e.d!
*A nother result,same solution:
r
(a + b + c)3
ab + bc + ca
+
28 while a, b, c > 0
abc
a2 + b2 + c2
=

*Some general problems: Let a,b,c>0. Find the best constant of k for inequalitis:
(a + b + c)3
ab + bc + ca k
+( 2
) 28
abc
a + b2 + c2
r
(a + b + c)3
ab + bc + ca
+k
2/
27 + k
abc
a2 + b2 + c2
1/

34

84, Let a, b, c 0. Prove that


b3
c3
1
a3
+
+

3
3
3
3
3
3
a + (a + b)
b + (b + c)
c + (c + a)
3
Let a + b + c = 3, we get a < 3, note that
a3

a3
1 4
1 (a 3)(a 1)2
(a 1) =
0
3
+ (3 a)
9 9
3 a2 3a + 3

Another solution :

a2
a3

(
)2
a3 + (b + c)3
a2 + b2 + c2

85, Let a, b, c > 0 such that abc = 1. Prove that:


X a
1
a3 + 2
By AM-GM we have:
a3 + 1 + 1 3a =>

X
a3

a
1
3. = 1
+2
3

And here is my proof:


by AM-GM we have:
r
3

ab + bc + ca
ab + bc + ca
2
,
2
2
2
a +b +c
a + b2 + c2

and:
abc

(ab + bc + ca)2
,
3(a + b + c)

so we have to prove that:


3(a + b + c)4
ab + bc + ca
+ 2
28,
2
(ab + bc + ca)
a + b2 + c2
we put a + b + c = x and ab + ac + bc = y with x y, so we find that we have to
prove that:
3x3 + 12x2 y 16xy 2 + y 3 0,
which is only AM-GM.
86, A nother result,same solution:
r
(a + b + c)3
ab + bc + ca
+
28 while a, b, c > 0
a2 + b2 + c2
abc
Manlio, we have
ab + bc + ca
1
a2 + b2 + c2
35

then

ab + bc + ca
ab + bc + ca
2
a2 + b2 + c2
a + b2 + c2
So, we only need to prove that
(a + b + c)3
ab + bc + ca
28
+ 2
abc
a + b2 + c2
To prove this inequality, we can use the known (ab + bc + ca)2 3abc(a + b + c). and
so we only need to prove
ab + bc + ca
3(a + b + c)4
+ 2
28
2
(ab + bc + ca)
a + b2 + c2
To prove this inequality, we just setting x =

3

ab+bc+ca
a2 +b2 +c2

1, then it becomes

2
1
+ 2 + x 28
x

3(1 + 2x)2 + x3 28x2


x3 16x2 + 12x + 3 0
(x 1)(x2 15x 3) 0
which is true since x 1.
Dear manlio,
My solution same to above my proof,also two lines:
WLOG assume a + b + c = 3, put

3
t = ab + bc + ca 3 a2 b2 c2 => a2 + b2 + c2 = 9 2t
s
r
27
t3
t3
27
27
26
1
+

+ abc =
+
+ abc 28
LSH =
abc
t.t.(9 2t)
abc
27
abc
abc abc
87, Let a, b, c > 0. Prove that
a2
b2
c2
+
+

b
c
a

a3 + b3
+
a+b

b3 + c3
+
b+c

c3 + a3
c+a

Using the lemma


X a2
b

15(a2 + b2 + c2 ) 3
(a + b + c)
2(a + b + c)
2

and the Cauchy Schwarz Inequality, we get


r
q
X a3 + b3
Xp
X
X
a2 ab + b2 3(2
a2
a)
=
a+b
36

It suffices to prove that




2
X
X
15(a2 + b2 + c2 ) 3
(a + b + c) 3(2
a2
ab)
2(a + b + c)
2

Assume a + b + c = 1 and setting x = ab + bc + ca 31 , we have to prove


(

15
3
(1 2x) )2 3(2(1 2x) x)
2
2
(6 15x)2 3(2 5x)
36 180x + 225x2 6 15x
30 165x + 225x2 0
10 55x + 75x2 0
2 11x + 15x2 0
(1 3x)(2 5x) 0
2

88, Let a, b, c 0 such that a + b2 + c2 = 3. Prove that


b
c
a
+
+
1
4a 4b 4c

X a
X 1
X (a 1)2 (2 a)
a
2(a2 1)
1

+
0
0.
4a
3 4a
9
4a
cyc
cyc
cyc
89, Let a, b, c 0. Prove that
a3

b3
c3
1
a3
+ 3
+ 3

3
3
+ (a + b)
b + (b + c)
c + (c + a)3
3

Here is my solution to this one :


It is equivalent to
X

1
1
with klm = 1
3
1 + (1 + k)
3

Now make the subtitution k =

yz
x2

and now we have to prove that


x6
1

6
2
3
x + (x + yz)
3

By Cauchy Swartz we have


X
x6

x6
(x3 + y 3 + z 3 )2
P 6
2
3
+ (x + yz)
x + (x2 + yz)3

So we have to prove that


(x3 + y 3 + z 3 )2
1
P 6

x + (x2 + yz)3
3
37

which is equivalent to
x6 + y 6 + z 6 + 5(x3 y 3 + y 3 z 3 + z 3 x3 ) 3xyz(x3 + y 3 + z 3 ) + 9x2 y 2 z 2
By AM-GM we have
x6 + x3 y 3 + x3 z 3 3x4 yz
and similar for the others and also by AM-GM
3(x3 y 3 + y 3 z 3 + z 3 x3 ) 9x2 y 2 z 2
90, Let a, b, c 0 such that a + b + c = 3. Prove that
a2
b2
c2
9
+
+
p
b+c c+a a+b
2 3(ab + bc + ca)
Of cause!

a2
b2
c2
9
+
+
p

b+c c+a a+b


2 3(ab + bc + ca)

X  a2
b+c
(a + b + c)2
a+b+c

b+c
4
2
2 3(ab + bc + ca)
cyc


p
2(a + b + c) a + b + c 3(ab + ac + bc)
X (2a b c)(2a + b + c)
p

b+c
3(ab + ac + bc)
cyc
X  (a b)(2a + b + c) (c a)(2a + b + c) 

b+c
b+c
cyc
P
2(a + b + c) (a b)2
p

3(ab + ac + bc) + (a + b + c) 3(ab + ac + bc)


X  (a b)(2a + b + c) (a b)(2b + a + c) 

b+c
a+c
cyc
P
X
2(a + b + c) (a b)2
p

(a b)2 Sc 0,
3(ab + ac + bc) + (a + b + c) 3(ab + ac + bc)
cyc
Sc =

1
1
p
.

(a + c)(b + c) 3(ab + ac + bc) + (a + b + c) 3(ab + ac + bc)

Let
a b c.T henSb 0, Sc 0and(a c)2 (b c)2 .
Thus,
X

(a b)2 Sc (a c)2 Sb + (b c)2 Sa (b c)2 (Sb + Sa ) 0,

cyc

38

which is true because Sb + Sa 0

a + b + 2c
2
p
, which obviously.

(a + b)(a + c)(b + c)
3(ab + ac + bc) + (a + b + c) 3(ab + ac + bc)

91, Let a, b, c > 0. Prove that


a
b
c
1
1
1
+ 2+ 2
+
+
2
2
2
2
2
b2
c
a
a ab + b
b bc + c
c ca + a2
Thanks, Lam. My solution is the following
Notice that by the AM-GM, we have
X1
X a

2
b
a
Hence, it suffices to prove that
X1 X
1

2
a
a ab + b2
By the Cauchy Schwarz Inequality, we have
X

2
a ab + b2

2

X

1
ab

 X

ab
2
a ab + b2

By AM-GM, we have
ab
3
ab + b2

2
X 1
1 X1

ab
3
a
X

a2

Multiplying these inequalities, we can get the result.


My solution. First by Holder we have


a
b
c
2
3
+
+
(ab + bc + ca) (a + b + c)
b2
c2
a2
And by Cauchy Schwarz we have
X

2
a ab + b2

s 
X
3

1
2
a ab + b2

It suffices us to show that


3

(a + b + c)

(ab + bc + ca)

s 
X
3

39

1
2
a ab + b2

Assume a + b + c = 1 and q = ab + bc + ca, r = abc then we have


Y

a2 ab + b2 = 3q 3 + q 2 + 10rq 8r2 3r
X

a2 ab + b2


b2 bc + c2 = 7q 2 5q 2r + 1

the inequality equalivents to



f (r) = 8r2 + 6q 4 + 10q 3 r 21q 6 + 15q 5 3q 4 3q 3 + q 2 0
We see that if
q 0, 2954then6q 4 + 10q 3 0
with the lemma
r

q 2 (1 q)
2 (2 3q)

we have

f (r) f

q 2 (1 q)
2 (2 3q)


=

360q 8 + 744q 7 574q 6 + 176q 5 + 3q 4 13q 3 + 2q 2


2

2 (2 3q)

Note that q 31 so the inequality has proved.


If q 0, 294 then we see that

 
1
f (r) min f (0) , f
0
27
Note that when r =

1
27

we have q 2 3pr q 31 . So q must be 31 .

92, Let a, b, c 0. Prove that


b2
c2
9
a2
+
+
p
b+c c+a a+b
2 3 (ab + bc + ca)
By Cauchy Schwarz we have
X

a2
b+c

 X

a2
b+c

 X

X

a2

b+c

2

X

a2

b+c

2

And by Holder we have


X

So it suffices us to show that


P 2 3
a
9
P 2
P 2 p
( a (b + c)) ( a )
2 3 (ab + bc + ca)
40

Setting q = ab + bc + ca, r = abc we rewrite the inequality


p
2
2 3q (9 2q) 9 (3q 3r)


By Schur inequality we have r max 0, 4q9
3
If q 49 r 0 then we need to show
p
2
2 3q (9 2q) 27q
(q 6, 75) (q 2, 701) 0
The inequality has proved. In the other way if q 49 r
show that
p
2
2 3q (9 2q) 9 (9 q)

4q9
3

then we need to

(q 3) (q 0, 0885) (q 5, 487) 0
which is true so we have done. The equality holds if a = b = c.
93, Let a, b, c 0 and ab + bc + ca = 3. Prove that
r
a2 b2 + b2 c2 + c2 a2
a + b + c abc + 2
3
The first I think you have a mistake. Setting p = a + b + c and r = abc then the
inequality becomes
r
9 2pr
pr
3
3p2 4pr + 3r2 9 0

0 = 4p2 3 3p2 9 = 27 5p2 0
And the equality does not hold.
94, Let a, b, c be positive real numbers such that abc = 1. Prove that
1
1
1
2
+
+
+
1
2
2
2
(1 + a)
(1 + b)
(1 + c)
(1 + a)(1 + b)(1 + c)
we use the fact that if a, b 1 or a, b 1 hence we have :(a 1)(b 1) 0 this mean
that : ab + 1 a + b hence:
2
2
c
=

(1 + a)(1 + b)(1 + c)
(1 + c)(1 + ab + a + b)
(1 + c)2
but we also have :
1
1
ab(a b)2 + (1 + ab)2
1
c
+
=
+

(a + 1)2
(1 + b)2
(ab + 1)(a + 1)2 (b + 1)2
ab + 1
c+1

41

thus we have :
LHS

c
c
1
+
= 1 = RHS
+
2
c + 1 (c + 1)
(c + 1)2

Setting :
xy =

1
1
1
; yz =
; zx =
(1 + a)2
(1 + b)2
(1 + c)2

Next, use contractdition. We need to prove :


Y r
mn
(4
1) 1
(n + p)(m + p)
Which is true by Am-Gm.
95, Let a, b, c 0; ab + bc + ca = 3. Prove that:
a2

1
1
1
3
+ 2
+ 2

+1 b +1 c +1
2

I cant prove it by cauchy-schwarz Smile .But I can prove it by pqr


<=> p2 12 + 3r2 2pr
If p2 12(1), easy to prove that :
RHS 12 LHS
If p2 12 ,we have :
r

p(12 p2 )
9

<=> (12 p2 )(p4 + 24p2 27) 0


Let p = a + b + c, q = ab + bc + c = 3, r = abc
LHS =

q 2 2pr + 2p2 4q + 3
(q 1)2 + (p r)2

3
2p2 2pr
3
q 2 2pr + 2p2 4q + 3

2
2
2
(q 1) + (p r)
2
4 + (p r)
2
4p2 4pr 12 + 3(p r)2 = 12 + 3p2 6pr + 3r2
p2 + 2pr 12 + 3r2
from Am-GM ;
(a + b + c)(ab + bc + ca) 9abc p 3r
pr 3r2
from Am-Gm ;
p2 q + 3pr 4q 2 p2 + pr 12
42


p2 + 2pr 12 + 3r2
3
2
96, If x, y, z are non-negative numbers such that xy + yz + zx = 3 then
LHS

x2 + y 2 + z 2 + 3xyz 6.
Note that from
xy + yz + zx = 3 => x + y + z 3
then
3xyz

9xyz
(x + y + z)

We will show that


x2 + y 2 + z 2 +

9xyz
6 = 2(xy + yz + zx)
(x + y + z)

97, If x, y, z are non-negative numbers such that x2 + y 2 + z 2 = 3, then


12 + 9xyz 7(xy + yz + zx).
With plesure:
12+9xyz 7(xy +yz +zx) =

x(x y)(x z)+(x+y +z 3)2 (x+y +z +5/2)

98, Let a, b, c 0; a2 + b2 + c2 = 3. Prove that:


12 + 9abc 7(ab + bc + ca)
Use schur , we need to prove :
2p3 6p 7p2 45
<=> (2p + 5)(p 3)2 0
99, Let a, b, c 0; abc = 1. Prove that :
(

a+b+c 5
a2 + b2 + c2
)
3
3

3abc(a + b + c)(a2 + b2 + c2 )
(ab + bc + ca)2 (a2 + b2 + c2 )

(a+b+c)5 /27
a+b+c
a+b+c
100, Let a, b, c, d > 0; 4 a + b + c + d. Prove that:

3abc(a2 +b2 +c2 ) =

1
1
1
1
+
+
+
1.
(a + 1)2
(b + 1)2
(c + 1)2
(d + 1)2
43

By Am-Gm .we have:


64
1
(a + b + c + d + 4)2

LHS
101, Let a, b, c > 0. Prove that:

(a2 + b2 + c2 )(a4 + b4 + c4 ) ((a b)(b c)(c a))2


Cauchy-Schwarz?
X
X
LHS = (
c2 )( (a2 )2 ) (a2 c+ab2 +bc2 )2 (a2 c+ab2 +bc2 (b2 c+a2 b+ac2 ))2 = RHS
cyc

cyc

a2 c + b2 a + c2 b a2 b b2 c c2 a = (b a)(c a)(c b)
102, For a, b, c real numbers such that a + b + c = 1. Prove that
X

1
27

1 + a2
10

I have seen the solution use targent line .But I can prove it by cauchy-schwarz Mr. Green
<=>

X
a2

3
a2

2
+ (a + b + c)
10

We have :(a + b + c)2 (|a| + |b| + |c|)2 Setting :x = |a|; y = |b|; z = |c| Assume
that :x + y + z = 1 By cauchy-schwarz ,we need to prove :
X
10(x2 + y 2 + z 2 )2 3(x2 + y 2 + z 2 ) + 3
x4
<=> 17q 2 11q 6r + 2 0
By Am-Gm we have :r

q
9

LHS 17q 2 11q

2q
2
+ 2 = (3q 1)(17q 6) 0
3
3

103, Find the maximum k = const the inequality is right


 a
2
b
c
a2 + b2 + c2
25
+
+
+1 k
+
k
b+c a+c b+a
ab + bc + ac
4
Try a = b; c = 0, we have : 11
4 k
To prove : if a, b, c 0 then :
X
(

11 a2 + b2 + c2
11 25
a
+ 1)2
.

+
b+c
4 ab + bc + ca
4
4

a, b, c 0 and no two of which are zero . Prove that :


(

a
b
c
11(a2 + b2 + c2 ) 7
+
+
+ 1)2
+
b+c c+a a+b
4(ab + bc + ca)
2
44

My solution Assume that :a + b + c = 1


<=> (

1 q + 2r 2
11
)
2
qr
4q
F.4q 1
W eprove :

11
(1 q)2

2
2
q
4q
(4q 1)(3q 4) 0(Rightbecauseq

1
)
4

F.4q 1
Use schur, we have :
r

4q 1
q7 2
LHS (
)
9
5q + 1

We prove that :
(

q7 2
11
)
2
5q + 1
4q

(3q 1)(17q 11)(4q 1) 0


Which is obvious true because

1
1
q
3
4
104, Let a, b, c are positive numbers such that a + b + c = 3. Prove that
X
a
1
2 + 2c
b
cyc
X
cyc

1
a
(a + b + c)2 2
1 <=>
b2 + 2c
ab + bc2 + ca2 + 2(ab + bc + ca)
a2 + b2 + c2 ab2 + bc2 + ca2

which is obviously true.


Yes, very simple
The last is equivalent to:
(a + b + c)(a2 + b2 + c2 ) 3(ab2 + bc2 + ca2 )
X
X
X

a3 +
a2 b 2
ab2
which is always true

F.Lemma : a + b + c = 3; a, b, c 0
45

=> ab2 + bc2 + ca2 + abc 4


9
P
LHS P 2
ab + 2 ab
I need to prove that :
X
X
9(
ab2 + abc) + 2
ab abc
Use this lemma and schur
(abc

4(ab + bc + ca) 9
)
3

Setting :x = ab + bc + ca
4(ab + bc + ca) 9
3

5 + abc 5 +

P
We have to prove: 3 ( ab). Which is obvious true.
105, Let a, b, c > 0 and a2 + b2 + c2 = 3. Prove that
X

1
1
a3 + 2

My old method works:


X

X
1

a3 + 2
cyc

1
1 a2 1

+
a3 + 2 3
6

X a2 (a + 2)(a 1)2
a3 + 2

cyc


0

0.

I prove it by contractdition The ineq equivalent to :


X
4 a3 b3 c3 +
a3 b3
By contractdition, we must prove this ineq: if a, b, c 0 safity that:
X
2
4 = abc + ab + bc + ca.then :
(ab) 3 3
Exist m, p, n 0 safity that :
a=

2m
2n
2p
;b =
;c =
.
n+p
m+p
m+n

So we must prove :
X

r
(3

2a 2
) 3
b+c
46

106, Problem If a, b, c and d are positive real numbers such that a + b + c + d = 4.


Prove that
a
b
c
d
+
+
+
2
2
2
2
1 + b c 1 + c d 1 + d a 1 + a2 b
Solution From Cauchy,
X

(a + b + c + d)2
a
P

2
1+b c
a + b + c + d + ab2 c

Hence it remains to show that


ab2 c + bc2 d + cd2 a + da2 b 4
We have
ab2 c + bc2 d + cd2 a + da2 b = (ab + cd)(ad + bc)

=

(a + c)(b + d)
2

2

ab + bc + cd + da
2

a+b+c+d
2

2

4
=4

A similar problem posted in the same topic, proven in a similar way as well; but the
proof isnt quite obvious at first glance: 107, Problem Let a, b, c and d be non-negative
numbers such that a + b + c + d = 4. Prove that
a2 bc + b2 cd + c2 da + d2 ab 4
Solution The left side of this inequality cannot be factorized as we did in the previous
one. But we do see that it can be written as
ac(ab + cd) + bd(ad + bc) 4
Wed be done if we could make ab + cd appear on the left instead of ad + bc. So lets
assume that ad + bc ab + cd. Then we have

2
ac + bd + ab + cd
ac(ab + cd) + bd(ad + bc) (ac + bd)(ab + cd)
=
2


(a + d)(b + c)
2

2

1
4

a+b+c+d
2

4
=4

and were done! Now it remains to deal with the case ab + cd ad + bc. But due to
the symmetry in the expression this case is easily dealt with in exactly the same way:

ac(ab + cd) + bd(ad + bc) (ac + bd)(ad + bc)


(a + b)(c + d)
2

2

47

ac + bd + ad + bc
2

a+b+c+d
2

4
=4

2
=

Thus we are done! Some harder problems: 108, Problem (ISL2004, A5) If a, b, c are
three positive real numbers such that ab + bc + ca = 1, prove that
r
r
r
1
3 1
3 1
3 1
+ 6b +
+ 6c +
+ 6a
a
b
c
abc
Solution Note that

1
7ab + bc + ca
+ 6b =
a
a

Hence our inequality becomes


Xp
3
bc(7ab + bc + ca)

1
2

(abc) 3

From Holders inequality we have


Xp
3
bc(7ab + bc + ca)

r
3

X 2  X 
a
9
bc

Hence it remains to show that


9(a + b + c)2 (ab + bc + ca)

1
[3abc(a + b + c)]2 (ab + bc + ca)4
(abc)2

Which is obviously true since


(ab + bc + ca)2 3abc(a + b + c)

a2 (b c)2 0

109, Problem Let a, b, c > 0 such that a + b + c = 1. Prove that

a2 + abc
b2 + abc
c2 + abc
1
+
+

c + ab
a + bc
b + ca
2 abc
Solution Note that

a2 + abc X
=
c + ab

p
a(c + a)(a + b)
(b + c)(c + a)

Therefore our inequality is equivalent to


p

X a2 + abc X a(c + a)(a + b)


=
c + ab
(b + c)(c + a)
By AM-GM,
p
a(c + a)(a + b)
a+b+c

(b + c)(c + a)
2 abc
X
p
1
a(a + b) bc(c + a)(a + b) (a + b + c)(a + b)(b + c)(c + a)
2
X

48

Now

p
a(c + a)(a + b)
a+b+c

(b + c)(c + a)
2 abc
X
p
1
a(a + b) bc(c + a)(a + b) (a + b + c)(a + b)(b + c)(c + a)
2
which was what we wanted.  Another one with square-roots and fractions: 110. Problem Let a, b, c > 0. Prove that
s 
r
r
r

a b
c
2a
2b
2c
+
+
3
+ +
b+c
c+a
a+b
b
c a
X

Solution From Cauchy-Schwarz inequality we have


s P
Pp
P
P
Xr a
a(a + b)(c + a)
( a) ( a2 + 3 bc)

=p
b+c
(a + b)(b + c)(c + a)
(a + b)(b + c)(c + a)
Therefore it remains to show that

X
 X
 X  X
X
X 
ab2
a2 b +
ab2 + 2abc
a2 + 3
bc 3
a
2abc
Let p =

a2 b, q =

ab2 . Then this is equivalent to

2abc(a+b+c)3 +2abc(p+q+3abc) 3q(p+q+2abc) 2abc

a3 +6abc(p+q+2abc)+2abc(p+q+3abc) 3q(

X
2abc
a3 + 8abcp + 8abcq + 18(abc)2 3pq + 3q 2 + 6abcq
X
X
X
2abc
a3 +8abcp+2abcq+18(abc)2 3
a3 b3 +3abc
a3 +9(abc)2 +3q 2
X
X
8abcp + 2abcq + 9(abc)2 3
a3 b3 + abc
a3 + 3q 2
Now verify that
q 2 3abcp

a2 b4 abc

a2 b

b2 (ab c2 )2 0

which is obviously true. Thus q 2 3abcp and q 2 3abcq (the latter follows directly
from AM-GM), which imply 3q 2 8abcp + abcq. Therefore it remains to show that
X
X
abcq + 9(abc)2 3
a3 b3 + abc
a3
which follows from adding the following inequalities, of which the former follows from
AM-GM and the latter from Rearrangement:
X
X
3
a3 b3 9(abc)2 abc
a3 abcq
Hence we are done.  In the solutions to the last few problems, one may rise the
question: why do we break up the square-roots in that specific way? For example in
49

the fourth problem one could apply AM-GM for bc and (c + a)(a + b) instead of
b(c + a)andc(a + b). Here are my thoughts on this: while trying to get a stronger
bound, its always worth it to end up with a form which is much less, as less as possible, than the upper bound of the problem (especially in these sort of cases while using
AM-GM or Cauchy-Schwarz). Hence in accordance with the majorization inequality,
we try to derive an expression where the degrees of the terms minorize as much as
possible. For example, if we used AM-GM for 4bc and (c + a)(a + b) wed get [2, 0, 0]
and [1, 1, 0] terms. But if I use it on b(c + a) and c(a + b) I get all [1, 1, 0] terms, which
in the long run could possibly be useful. The same idea goes for the other problems as
well. If you want to see more examples of this, try looking at some older blog posts and
hopefully youll find some more illustrations.
111, If a, b, c > 0. show that :
X
b2

3
a

2
+ bc + c
a+b+c

good prob: now by cs :


X
X
a(b2 + bc + c2 ).

X 2
a
a
>
b2 + bc + c2

its enough to show that:


P 2
X
( a)
3
P
>P
a3 > 3abc
2
2
a(b + bc + c )
a
112, If a, b, c > 0 show that :
X
b2

a
3

2
+ bc + c
a+b+c

Its trival by Cauchy-Schwarz or AM-GM,...


The following Inequality is stronger
X

a
2

2
2
2b + bc + 2c
5

X

1
b+c

113, This one is more easier and i can say it comes from dduclams :
X
c
1 X 1

2
2
7a + 6ab + 7b
10 cyc a + b
cyc
114,
a
2

2b2 + bc + 2c2
5
Its obviously trues by CS and Schur Smile.
X

X

1
b+c

115, Let a, b and c ate non-negative numbers such that ab + ac + bc 6= 0. Prove that:
4b2

a
b
c
1
+ 2
+ 2

2
2
2
+ bc + 4c
4a + ac + 4c
4a + ab + 4b
a+b+c
50

Using Cauchy Schwarts, we need prove:


(a2 + b2 + c2 )2 (a + b + c) 4
<=>

a5 +

ab(a3 + b3 ) + 2abc

a2 b2 (a + b) + abc

ab 2

a2

a2 b2 (a + b) + abc

a2

Using Schur which degree 5, we have:


X
X
X
a5 + abc
a2
ab(a3 + b3 )
So, we only need to prove:
X
X
X
X
ab(a3 + b3 ) + abc
ab
a2 b2 (a + b) + abc
a2
abc
](a b)2 0
2
Easily to see that Sb, Sc, Sa + Sb 0. We have done
<=> [ab(a + b)

nice solution,nguoivn
my solution:
2

LHS

(a3 + b3 + c3 )
P
3
a b3 (a + b) + abc a4

we need to prove that:


2

(a3 + b3 + c3 ) (a + b + c) 4

a3 b3 (a + b) + abc

a4

it equivalent to:
X
X
X
X
X
a7 +
ab(a5 + b5 ) + 2abc
a2 b2 2
a3 b3 (a + b) + abc
a4
<=>

(a b)


2
a5 + b5 c5 2(a + b) abc + 4ab(a + b)(a2 + ab + b2 )

its very easily to prove that


Sc ; Sb ; Sb + Sa > 0if a b c
we have done.
116, If a, b, c be nonnegative real numbers such that a + b + c = 3, then
1
1
1
3
+
+

(a + b)2 + 6 (b + c)2 + 6 (c + a)2 + 6


10

X
X
1
3
1
1
1

(a

1)
0
(a + b)2 + 6
10
(3 a)2 + 6 10 25
cyc
cyc
51

X (a 1)2 (5 2a)
a2 6a + 15

cyc

Thus, our inequality is proven for max{a, b, c} 2.5.


Let 2.5 < a 3. Hence, 0 b + c < 0.5. But
00

6(x2 6x + 7)
1
> 0f or0 x < 0.5.
=
x2 6x + 15
(x2 6x + 15)3
Hence,
X
cyc

a2 6a + 15
2


3a 2
2

3a
2

+ 15

2

b+c 2
2
a2

b+c
2

+
+ 15

1
=
a2 6a + 15

1
8
1
= 2
+ 2
6a + 15
a + 6a + 33 a 6a + 15

Id est, it remains to prove that


1
3
8
+

a2 + 6a + 33 a2 6a + 15
10
which is equivalent to (a1)2 (3a)(a+5) 0, which is true for 2.5 < a 3. Done!
117, This is the strongest of this form
X

a
1
2
4b3 + abc + 4c3
a + b2 + c2

By the way, the following reasoning


X
cyc

X
a
a2 (ka + b + c)2
=

4b3 + abc + 4c3


(4b3 a + a2 bc + 4c3 a)(ka + b + c)2
cyc
2
P 2
(ka + 2ab)
P

(4b3 a + a2 bc + 4c3 a)(ka + b + c)2

gives a wrong inequality


2
P 2
(ka + 2ab)
1
P 3
2
(4b a + a2 bc + 4c3 a)(ka + b + c)2
a + b2 + c2
for all real k.
You can try to use Cauchy Schwarz like this, arqady.
X

a
4b3 + abc + 4c3

 X

a(4b3 + abc + 4c3 )


(b + c)2

52

X

a
b+c

2
.

118, The following inequality is true too. Let a, b and c are non-negative numbers such
that ab + ac + bc 6= 0. Prove that
2b3

b3
c3
a3
+ 3
+ 3
1
3
3
abc + 2c
2a abc + 2c
2a abc + 2b3

where all denominators are positive.


my solution:
2

LHS

(a3 + b3 + c3 )
3
a b3 abc(a3 + b3 + c3 )

we need to prove that:


X

a6 + abc(a3 + b3 + c3 ) 2

a3 b3

by Schur and AM-GM ineq,we get:


X
X
ab(a4 + b4 ) 2
a3 b3
LHS
we have done!
Also by
X

a6 + abc(a3 + b3 + c3 ) 2

a3 b3

we have :
a3 + b3 + c3 3abc
So we have to prove that:
X

a6 + 3a2 b2 c2 2

a3 b3

Puting a2 = x and cyclic we have Schur for n = 3 So


X
X
a6 + 3a2 b2 c2
a4 b2
sym

So we now have to prove that:


X

a4 b2 2

a3 b3

sym

which is Muirhead for the triples (4, 2, 0)  (3, 3, 0)......


119, Now how do you think about following Inequalities
X
cyc

8b3

a2
1

+ 11abc + 8c3
3(a + b + c)

53

and
X
cyc

1
a

7b3 + 13abc + 7c3


(a + b + c)2

for all a, b, c be positive real numbers.


With the first, my way above still solve it easily. With the second, using CS again,
we obtain:
X
7
ab(a2 + b2 ) + 13abc(a + b + c) (a + b + c)4
Its easy to prove (of course, with some calculus)
119, Besides, we also have a nice result
(a2 + b2 + c2 )(ab + bc + ca)2 (a2 + 2bc)(b2 + 2ca)(c2 + 2ab)
Its equivalent to (a b)2 (b c)2 (c a)2 0, obvious.
120, Given a, b, c 0 and a + b + c = 3. Prove that:
p
p
p
p
1
3
3
3
(ab + bc + ca) a2 + b2 + c2
a2 + 2bc + b2 + 2ca + c2 + 2ab
6
3
Besides, we have a similar result (with the same condition)
p
p
p
p

5( a2 + 2bc + b2 + 2ca + c2 + 2ab) 4(ab + bc + ca) a2 + b2 + c2 + 3 3


Both of them can be solved by Am-Gm
121, Given a, b, c 0. Prove that:
a2

b
c
(a + b + c)3
a
+ 2
+ 2

+ 3bc b + 3ca c + 3ab


4(ab + bc + ca)2

122, Nice solution. But why


X
a2

a
a+b+c

+ 2bc
ab + bc + ca

Its true by voirnc hur


<=>

X a(a b)(a c)
a2 + 2bc

123, For all a, b, c be nonnegative real numbers, we have


b
c
2(a2 + b2 + c2 )
a
+ 2
+ 2

2
a + 2bc b + 2ca c + 2ab
(a + b + c)2

54

1
1
1
+
+
a+b b+c c+a

We have
X
a2
and
X

a
a+b+c

(well-known result)
+ 2bc
ab + bc + ca

( a + b + c)2
( a + b + c)2
1
P

=
b+c
a(b + c)
2(ab + bc + ca)

(Cauchy Schwarz) Hence, it suffices to prove that

a+b+c
(a2 + b2 + c2 )( a + b + c)2

,
ab + bc + ca
(a + b + c)2 (ab + bc + ca)
or

( a + b + c)2 (a2 + b2 + c2 ) (a + b + c)3 .

Luckily, this is Holder Inequality, and so it is valid


We need prove that:
2(a2 + b2 + c2 )
(a + b + c)2

1
1
1
+
+
a+b b+c c+a

a+b+c
ab + bc + ac

Using Cauchy Schwarts, we only need to prove:


2(a2 + b2 + c2 )(ab + bc + ca) (a + b + c).
<=>

ab(a + b)

ab(a b)2 0

(obviously trues)
We have:
LHS

a+b+c
ab + bc + ac

We need prove that:


2(a2 + b2 + c2 )
(a + b + c)2

1
1
1
+
+
a+b b+c c+a

a+b+c
()
ab + bc + ac

Let: p = a + b + c = 1; q = ab + bc + ac 31 ; r = abc So
() <=>

2(1 2q)(1 + q)
1

qr
q

<=> q 2q 2 4q 3 + r 0
Case 1:
q
So:

1
4

1
1
q
q 2q 2 4q 3 + r 2q( q) + 4q( q 2 ) + 0
4
16
4
55

We have done! Case 2:

1
1
q
3
4

We have:
r

4q 1
bychur
9

So:
9(q 2q 2 4q 3 + r) 9q + 4q 1 18q 2 4q 3 = (1 12q 2 10q)(3q 1) 0
Its true because

1
1
q
3
4
124, Let a, b, c be nonnegative real numbers, prove that
s
X
bc
9abc
1+
(a
+
b)(a
+
c)
2(a
+
b
+
c)(ab
+ bc + ca)
cyc
125, Let a, b, c 0. Prove that:
X

3 X a(b + c)
a

b+c
4
b2 + bc + c2

3 X a(b + c)
a

b+c
4
b2 + bc + c2
3 a(b + c)
a

2
2
4 b + bc + c
b+c

a(b c)2 0...


126, Let a, b, c 0. Prove that:
X

a
3(a3 + b3 + c3 )
1+
b+c
2(a + b + c)(a2 + b2 + c2 )

127, If a, b, c be nonnegative real numbers, then


s
a
b
c
3(a3 + b3 + c3 )

+
+

+a+b+c
2(a2 + b2 + c2 )
c+a
b+c
a+b
128, If a, b, c be sidelengths of a triangle, then
r
b
c
3(a3 + b3 + c3 ) a + b + c
a

+
+

+
a2 + b2 + c2
2
c+a
b+c
a+b
By cauchy-swarchz.We have:
LHS 2 (a + b + c)(

a
b
c
+
+
)
b+c c+a a+b

56

We need prove that:


3(a3 + b3 + c3 ) a + b + c
a
b
c
+
(a + b + c)(
+
+
2
2
2
a +b +c
2
b+c c+a a+b
We have:
a
b
c
1
3(a3 + b3 + c3 )
+
+
+
b+c c+a a+b
2 (a + b + c)(a2 + b2 + c2 )
129, If a, b, c be sidelengths of a triangle, then
b
c
1
3(a3 + b3 + c3 )
a
+
+
+
b+c c+a a+b
2 (a + b + c)(a2 + b2 + c2 )
130, If a, b, c be sidelengths of a triangle, then
a
b
c
7
a3 + b3 + c3
+
+
+
b+c c+a a+b
6 (a + b + c)(ab + bc + ca)
131, If a, b, c be nonnegative real numbers, then
b
c
5
3(a3 + b3 + c3 )
a
+
+
+
b+c c+a a+b
4 4(a + b + c)(ab + bc + ca)
When does equality hold? 132, Let a, b, c 0; a + b + c = 1. Prove that:
X a + bc
b+c

9abc
9

4(ab + bc + ac)
4

Its very easy.Use Iran 96


Solution (mitdac123)
We have:
X
a + bc X (a + b)(a + c)
9 (a + b)(b + c)(c + a)
1
=
= (a+b)(b+c)(c+a)(
) .
=
b+c
b+c
(b + c)2
4
ab + bc + ca
9 (a + b)(b + c)(c + a)
.
4 (ab + bc + ca)(a + b + c)
and:

9
abc
9
abc
.
= .
4 ab + bc + ca
4 (ab + bc + ca)(a + b + c)

Done!
My solution Very Happy
Setting:
ab + bc + ac = x;

1
1
1
+
+
=y
2
2
(a + b)
(b + c)
(c + a)2

So:
xy
57

9
4

Because:
a + b + c = 1 => abc

x
9

Inequality
9
9
y)
4x
4
x 9
9
LHS xy + (
y)
9 4x
4
8
1
9
<=> xy +
9
4
4
Its true Very Happy. Done!
<=> xy + abc(

133, Let a, b, c be nonnegative real numbers such that max(a, b, c) 4min(a, b, c).
Prove that
2(a + b + c)(ab + bc + ca)2 9abc(a2 + b2 + c2 + ab + bc + ca)
Equality holds for a = b = c or a = 4b = 4c
Nice: Inequality
<=> Sa (b c)2 + Sb (c a)2 + Sc (a b)2 0
With:
Sa =

4ab + 4bc 5ac


4bc + 4ac 5ab
4ab + 4ac 5bc
; Sb =
; Sc =
36bc(ab + bc + ac)
36ac(ab + bc + ac)
36ab(ab + bc + ac)

* a b c => a 4c
=> Sa 0; Sb 0; Sa + Sc =

4b(c a)2 + 2ca(2c + 2a b)


0
36abc(ab + bc + ac)

Your solution is corect. In my solution, we have (assuma b c):


Sa Sb Sc andSb + Sc = 4a(b c)2 + 2bc(2b + 2c a) 0
134, Given that a, b, c 0. Prove that;


a
b+c

2


+

b
c+a

2


+

c
a+b

2

3(a2 + b2 + c2 )
4(ab + bc + ca)

suppose that a b c, by arrangement inequality we have


(

a 2
b 2
c 2
) +(
) +(
)
b+c
c+a
a+b

1 2
1 2
1 2
1 2
(a + b2 + c2 )[(
) +(
) +(
) ]
3
b+c
c+a
a+b

58

then its only to prove that


(ab + bc + ca)[(

1 2
1 2
1 2
9
) +(
) +(
) ]
b+c
c+a
a+b
4

which is obvious now


It equivalent to
4abc

X
cyc

X a3
a
+
4
3(a2 + b2 + c2 )
(b + c)2
b+c

By AM-GM
X

9
a

2
(b + c)
4(a + b + c)

We only need to prove


X a3
9abc
+4
3(a2 + b2 + c2 )
a+b+c
b+c
9abc + 4

X a4
X
+ a3 + b3 + c3 3
bc(b + c)
b+c

By AM-GM
X a4
a3 + b3 + c3

b+c
2
It suffices to show that
a3 + b3 + c3 + 3abc ab(a + b) + bc(b + c) + ca(c + a)
Its Schur!
135, Let a, b, c be nonnegative real number. Prove that


a
b+c

2


+

b
c+a

2


+

c
a+b

2
+

5(a2 + b2 + c2 )
1

2
4(ab + bc + ca)

136, Let a, b, c be nonnegative real number, no two of which are zero. Prove that
(a)

a(2a b c) b(2b c a) c(2c a b)


2(a2 + b2 + c2 )
+
+
+2
2
2
2
(b + c)
(c + a)
(a + b)
ab + bc + ca

(b)

a(a b c) b(b c a) c(c a b) 3


3(a2 + b2 + c2 )
+
+
+
2
2
2
(b + c)
(c + a)
(a + b)
2
4(ab + bc + ca)

137, Let be a, b, c > 0 such that ab + bc + ca = 1. Show that :


b
c
a

+
+
1
2
2
a + b + 1 b + c + 1 c + a2 + 1
59

Because

a2 + 1 = a2 + bc + ab + ca 2a bc + ab + ca = a( b + c)2
hence

X
a
a


=1
a + b2 + 1
a + a( b + c)

138, Let a, b, c > 0. Prove that


X (ab)2
9abc

2
(a + b)
4(a + b + c)
The first Inequality is Iran TST 1996 Inequality (af tersettingx = ab, y = bc, z =
ca).
139, Prove that for all a, b, c be nonnegative real numbers, we have


b2
c2
2 (a + b + c)2
a2
b2
c2
a2
+
+

+
+
b+c c+a a+b
3 ab + bc + ca 2a + b + c 2b + c + a 2c + a + b
140, For all a, b, c be nonnegative real numbers. Prove that
a4
b4
c4
ab + bc + ca
a3
b3
c3
+
+
+

+
+
2
2
2
(b + c)
(c + a)
(a + b)
4
b+c c+a a+b
141, Let a, b, c be sidelengths of a triangle. Prove that


X
1
3 2
1
1
2
2
(a + b + c )
+
a2 + bc
4
a3 b + b3 c + c3 a ab3 + bc3 + ca3
cyc
Very nice inequality, Dduclam. My solution proved that
X

12(a2 + b2 + c2 )
3
1
P
P 2 2 (a2 +b2 +c2 )

2
2
2
a + bc
3 ab(a + b ) + 2 a b
4

1
1
+ 3
3
3
3
a b + b c + c a ab + bc3 + ca3

But I think it is not nice. I think you have a better proof, may I see it, my friend
142, If a, b, c be sidelengths of a triangle. Prove that


1
1
1
2(a + b + c)2
1
1
1
+
+

+
+
a2 + bc b2 + ca c2 + ab
3(a2 + b2 + c2 ) (a + b)2
(b + c)2
(c + a)2
143, Let a, b, c be nonnegative real numbers. Prove that
a(a + b)(a + c) b(b + c)(b + a) c(c + a)(c + b)
(a + b + c)4
+
+

(b + c)3
(c + a)3
(a + b)3
6(ab + bc + ca)2
144, Let a, b, c be nonnegative real numbers. Prove that
X
cyc

4
1

(b + c)(b2 + bc + c2 )
(a + b)(b + c)(c + a)
60


.

145, Let a, b, c be non-negative real numbers such that a + b + c = 3. Prove that


a
b
c
+
+
1
a + 2bc b + 2ca c + 2ab
X
b
c
a2
(a + b + c)2
a
+
+
=

a + 2bc b + 2ca c + 2ab


a2 + 2abc
a2 + b2 + c2 + 6abc
cyclic

from am-gm
(a + b + c)(ab + bc + ca) 9abc, soab + bc + ca 3abc
(a + b + c)2
a2 + b2 + c2 + 6abc
2
=1
2
2
+ b + c + 6abc
a + b2 + c2 + 6abc
146, Let a, b, c be non-negative real numbers such that a + b + c = 3. Prove that

a2

a
b
c
+
+
1
2a + bc 2b + ca 2c + ab
X
X
2a
a
1
2
2a + bc
2a + bc
cyclic

cyclic

(1

cyclic

2a
)1
2a + bc

by Cauchy-Schwarz ;
X
cyclic

X
(ab + bc + ca)2
bc
(bc)2
=

2a + bc
2abc + b2 c2
6abc + a2 b2 + b2 c2 + c2 a2
cyclic

a2 b2 + b2 c2 + c2 a2 + 2abc(a + b + c)
=1
6abc + a2 b2 + b2 c2 + c2 a2
147, Let a, b, c be sidelengths of a triangle which perimeter 3. Prove that
=

1
1
1
9
+
+

3
3
ab + bc + ca
b+ca
c+ab
a+bc

The following Inequality is true or false for the same condition?

1
1
1
9
+
+

4
4
ab
+
bc
+ ca
b+ca
c+ab
a+bc

148, Let a, b, c 0 and ab + bc + ca = 3. Prove that


1
1
3
1
+
+

1 + a2
1 + b2
1 + c2
2
The expanding gives:
X
cyc

1
3
(a b)2 (a c)2 (b c)2 + 4abc(a3 + b3 + c3 3abc) 0.
1 + a2
2
61

149, Let a, b, c > 0,

1
a2 +1

= 2. Prove:
ab + bc + ac

3
2

simple solution:
1
1
1
a2
b2
c2
+
+
=2
+
+
=1
2
2
2
2
2
1+a
1+b
1+c
1+a
1+b
1 + c2
by cauchy:
(a2 +1+b2 +1+c2 +1)(

b2
c2
a2
3
+
+
) (a+b+c)2 (ab+bc+ca)
2
2
1+a
1+b
1 + c2
2

150, Let be a, b, c (0, ) such that ab + bc + ca = 1. Show that :


p
p
p
9a2 b2 c2 + abc( 1 + a2 + 1 + b2 + 1 + c2 ) 1
Since 1 + a2 = (a + b)(a + c), by AM-GM we have
LSH 9ab.bc.ca+abc(a+b+b+c+c+a)

2
1
(ab+bc+ca)3 + (ab+bc+ca)2 = 1
3
3

151, Let a, b, c be the side lengths of a triangle. Prove that


r
r
r
2
2
2
2
2
2
2a
2b
2c
3 a + b
3 b + c
3 c + a
+
+

+
+
2
2
2
2c
2a
2b
b+c c+a a+b
152, for all a, b, c 0 we have following Inequality


a
b
c
2 2
1
1
1
2
2
+
+
(a + b + c )
+
+
b+c c+a a+b
3
(a + b)2
(b + c)2
(c + a)2
153, Let a, b, c 0. Prove the following inequality:


a
b
c
2
1
1
1
+
+
(ab + bc + ca)
+
+
b+c c+a a+b
3
(a + b)2
(b + c)2
(c + a)2
X 3a
X 2ab
2c

[
+
]
b+c
(a + b)2
a+b
X a
X 2ab

b+c
(a + b)2
X ab + ac 2bc

0
(b + c)2
a b c; ab + ac 2bc bc + ba 2ca ca + cb 2ab
1
1
1

(b + c)2
(c + a)2
(a + b)2
62

Applying Chebyshev Inequality. Done 154, Let a, b, c 0. Prove the following inequality:
(

a
1
1
b
c 2
1
+
+
]
+
+
) (ab + bc + ca)[
b+c c+a a+b
(a + b)2
(b + c)2
(c + a)2

155, Let be a, b, c > 0 such that a2 + b2 + c2 = 3. Show that :


a2 + b2
b2 + c2
c2 + a2
+
+
3
ab(ab + 1) bc(bc + 1) ca(ca + 1)
Of course, by AM-GM
X a2 + b2
X 2
X
18

3
ab(ab + 1)
ab + 1
ab + bc + ca + 3
156, Prove that for all a, b, c be nonnegative real numbers, we have


a
b
c
a3 +b3 +c3 +6abc a2 (b+c)+b2 (c+a)+c2 (a+b)+2abc
+
+
b+c c+a a+b
Its equivalent to
(a b)2 (a c)2 (b c)2 +

ab(a2 + ab + b2 c2 )(a b)2 0

cyc

which is obviously true. 157, Prove that for all a, b, c be nonnegative real numbers, we
have
c+ab
a+bc
1
b+ca
+ 2
+ 2

2
5a + 4bc 5b + 4ca 5c + 4ab
a+b+c
158, For a, b, c be nonnegative real numbers, we have

2
1
1
1
2

+
+

+
2
2
2
2
2
2
ab + bc + ca
a + bc
b + ca
c + ab
a +b +c
159,

X 2
1

a+b
a2 + bc
cyc
cyc

For along time, we havent got solution for this problem yet. Now, I will post my solution
Problem (Vo Quoc Ba Can) Let a,b,c be nonnegative real numbers. Prove that

X
X 2
1

.
a+b
a2 + bc
cyc
cyc
Solution.
By the Cauchy schwarz Inequality, we have
X
2 X
 X

1
1
(a + b)(a + c)

(a + b)(a + c)
a2 + bc
a2 + bc
63

P
X

2 a
a(b + c)
+
3
(a + b)(b + c)(c + a)
a2 + bc

It suffices to show that


P
X

X
2
2 a
a(b + c)
1
+3 2
(a + b)(b + c)(c + a)
a2 + bc
a+b
P
P
X a(b + c)
( a2 + 3 ab)2
P

+
3

a2 + bc
(a + b)(b + c)(c + a) a
P 4 P 2 2


X
X a(b + c)
a a b
1
1
P
3

(ab)(ac)
+
0

a2 + bc
(a + b)(b + c)(c + a) a
a2 + bc (b + c)(a + b + c)
Due to symmetry, we may assume a b c, since a c ab (b c). It suffices to
show that




1
1
1
1
a
+

b
+
a2 + bc (b + c)(a + b + c)
b2 + ca (a + c)(a + b + c)
c(a2 b2 )[(a b)2 + ab + bc + ca] 0
which is trivial. Equality holds if and only if a = b = c.
160, Let a, b, c be non-negative real numbers. Prove that
b
c
a
b
c
a
+
+

+
+
2
2
2
b+c c+a a+b
a + 3bc
b + 3ca
c + 3ab
After using Cauchy Schwarz, we can see that the inequality follows from
X
that is

X a(b + c)
a

,
b+c
a2 + 3bc

X a(a2 + bc b2 c2 )
(b + c)(a2 + 3bc)

or

X a3 (b + c) a(b3 + c3 )
(b + c)2 (a2 + 3bc)

0,

0.

Without loss of generality, we can assume that a b c, then


a3 (b + c) a(b3 + c3 ) 0 c3 (a + b) c(a3 + b3 ),
and

1
1
1

.
(b + c)2 (a2 + 3bc)
(c + a)2 (b2 + 3ca)
(a + b)2 (c2 + 3ab)

It follows that
a3 (b + c) a(b3 + c3 )
a3 (b + c) a(b3 + c3 )

,
(b + c)2 (a2 + 3bc)
(c + a)2 (b2 + 3ca)
64

and

c3 (a + b) c(a3 + b3 )
c3 (a + b) c(a3 + b3 )

.
2
2
(a + b) (c + 3ab)
(c + a)2 (b2 + 3ca)

Therefore
X a3 (b + c) a(b3 + c3 )
(b + c)2 (a2 + 3bc)

P 3
(a (b + c) a(b3 + c3 ))
= 0.
(c + a)2 (b2 + 3ca)

Our proof is completed.


Infact, we have the following Inequality for all a, b, c 0
a
b
c
(a + b + c)2

+
+
2(ab + bc + ca)
a2 + 3bc
b2 + 3ca
c2 + 3ab
161, Let equation:
(x + 1).lnx x.ln(x + 1) = 0.
Prove that this equation have only one root
I can prove it.
x0

f(x) = (x + 1)lnx - xln(x + 1)


f 0 (x) = ln(
Setting :

x+1
x
x
)+

x+1
x
x+1

x
<1
x+1
1
g(x) = lnt + t
t
1
1
g 0 (x) = 2 1 0t R
t
t
=> g(x) g(1) = 0
t=

=> f 0 (x) 0
=> ...
But we have :
f (...) = ... > 0
162, For positive a, b and c such that a + b + c = 3. Prove that:
b2
c2
a2
+
+
1
2a + b2
2b + c2
2c + a2

65

P
X
a2
a4
( a2 )2
P
P
=

2a + b2
2a3 + a2 b2
2 a3 + a2 b2

So we must prove that


X
X
X
X
X
X
a4 +
a2 b2 2
a3 or
a4 + 3
a2 b2 2
a3 b + a3 c
and i think the last inequality is true
Of course the last is was true Wink .Because :
X
<=>
(a b)4 0
X

X
a4
(a2 + b2 + c2 )2
a2
=
3
1
2
3
2
2
3
a + 2b
a + 2a b
(a + b + c3 ) + 2(a2 b2 + b2 c2 + c2 a2 )

So its enough to prove that :


a4 + b4 + c4 a3 + b3 + c3 3(a4 + b4 + c4 ) (a3 + b3 + c3 )(a + b + c)
which reduces to :
2

a4

(a3 b + ab3 )

which is just Muirhead.Or you can also prove this last inequality by AM-GM.
163, Let a, b, c > 0 and abc = 1. Prove that
b2
c2
a
b
c
a2
+
+

+
+
1 + 2ab 1 + 2bc 1 + 2ca
ab + b + 1 bc + c + 1 ca + a + 1
a=

x
y
z
b= c=
y
z
x

we have to prove that:


X
cyc

X ( xz
x2 z
y )

2
y (2x + z)
x+y+z
cyc

(x + y + z)

X
cyc

X xz
x2 z

2
y (2x + z)
y
cyc

X x z
X xz
2x z
x2 z 2
( 2 +
+ 2
)2
y
y(z + 2x) y (z + 2x)
y
cyc
cyc
By Cauchy
x2 z
(2x y)z
2xz

=
z
y y
y
y
9x2
( y(2x+z)
)
z

6x

y(2x + z)
2xy
= 6x y
z
z

66

z
9x2
y(2x + z)
6xz
z
)=
2x z
(6x
y(2x+z)
y(
y
z
y
)
z

So
X 20xz
X xz
X x2 z
2x2 z
x2 z 2
2x
+ 2
)
(

)2
( 2 +
y
y(z + 2x) y (z + 2x)
9y
9
y
cyc
cyc
cyc
Let me try another solution MSetting :
x
y
z
a = ;b = ;c =
y
z
x
LHS =

RHS =

x2 y 2
z2

2xz + z 2
P xy
z

x+y+z

By cauchy-schwarz, we can prove


X
X
(x + y + z) LHS = (
2xz + z 2 )(

x2 y 2
z2

2xz +

z2

X xy
)(
)2
z

Now, we need to prove :


X xy
X xy
)2 (x + y + z)(
)
(
z
z
X xy
<=>
x+y+z
z
Which is obvious true by Am-Gm
164, Let a, b, c be positive real numbers such that a + b + c = 3. Prove that
b
c
1
a
+
+

2b + 1 2c + 1 2a + 1
abc
Very nice inequality, my brother I have solved it and I hope that I have not made mistake
in solving ... Here is my solution. Note that we have a well-known inequality:
FLemma :
Let a, b, c 0 such that a + b + c = 3 we have:
a2 c + c2 b + b2 c 4 abc
It is easy to prove if we assume that
a(a b)(c b) 0.
Now we are coming back above inequality:
F Let a, b, c be positive real numbers such that a + b + c = 3. Prove that
a
b
c
1
+
+

2b + 1 2c + 1 2a + 1
abc
67

Expanding and numeratoring we have the inequality is equivalent to:


X
X
X
7 + 4abc
ab + 4
ab 2
a2 b2 + 13abc + 4abc(a2 c + c2 b + b2 c)
Applying above lemma, we only need prove that:
X
X
X
7 + 4abc
ab + 4
ab 2
a2 b2 + 13abc + 4abc(4 abc)
Now, I think we can solve it easily by p, q, r technique. In fact, we put q = ab + bc +
ca, r = abc and note that q 3. The inequality is equivalent to:
f (r) = 4r2 + (4q 17)r 2q 2 + 4q + 7
Using Schurs inequality we have:
12q 27
q
r
9
3
Now, we will use derivative method to prove that f (r) 0.
T hef irstcase : 4q + 8r 17 0then


12q 27
94(3 q)2
f 0 (r) = 4q + 8r 17 0 f (r) f
0
=
9
9
The second case: 4q + 8r 17 0 then
f 0 (r) = 4q + 8r 17 0 f (r) f

q
3

(3 q)(2q + 21)
0, 0 q 3
9

We are done
Let me try. My solution used Am-Gm .
F Lemma :
a + b + c = 3, a, b, c 0
then :
a2 b + b2 c + c2 a + abc 4 (It can prove by Am-Gm)
Expanding the ineq :
X
X
X
4(
a2 c)abc + 4abc + (
a2 )abc 4
ab + 7
X
LHS 20abc + (
a2 )abc 4a2 b2 c2
We need to prove:
X
X
20abc + (
a2 )abc 4(a2 b2 c2 + 1) + 4
ab + 3
Easy to prove :
a2 b2 c2 + 1 2abcand :
68

ab 3abc

The ineq become:


X
3(
a2 )abc
But :
9LHS = (

X
a2 )[3(a + b + c)abc] (
a2 )(ab + bc + ca)2 27

Another solution(but similar) Wink


<=>

X 2a2 bc
2
2b + 1

X a2 c
X

a2 c 2
2b + 1
By cauchy-schwarz , we can prove:
P
( a2 c)2
P
LHS P
2( a)abc + a2 c
<=>

So ,we need to prove :


X
X
X
(
a2 c)2 (
a2 c 2)(6abc +
a2 c)
X
a2 c + 6abc 3abc(
a2 c)
X
<=> 6abc (3abc 1)(
a2 c)
X
<=> 5abc + 3a2 b2 c2 (3abc 1)(
a2 c + abc)
<=>

Why the above lemma , we can prove :


RHS 12abc 4
LHS = 5abc + 3(a2 b2 c2 + 1) + 1 4 12abc 4
165, Let a, b, c be positive real numbers such that a + b + c = 3. Prove that
a
b
c
1
+
+

2b + 1 2c + 1 2a + 1
abc
F My similar one,also nice:
1
1
1
1
+
+
2 2 2
a(2ab + 1) b(2bc + 1) c(2ca + 1)
a b c
PS: Two problems were made (stronger) from my following two ones:
F

b
c
1
1
a

+
+

3
(b + 2)2
(c + 2)2
(a + 2)2
3abc

69

1
1
1
1
1
+
2 2 2

+
2
2
2
3
a(ab + 2) ) b(bc + 2)
c(ca + 2)
3a b c

166, Let a, b, c 0 such that a + b + c = 1. Prove that


r
r
r
a + 2b
b + 2c
c + 2a
+
+
3
a + 2c
b + 2a
c + 2b
Setting :x = a + 2b; y = b + 2c; z = c + 2a; x, y, z [0; 23 ]
Assume that :a + b + c = 1 => x + y + z = 3
Xr x
<=>
3
2x
X
[
x2 (2 x)]LHS 2 (x + y + z)3
We need to prove:
(x + y + z)3 3[2(x + y + z)(x2 + y 2 + z 2 ) 3x3 3y 3 3z 3 ]
X
X
<=> 4
x3 3
xy(x + y) + 6xyz 0
Its easier than schur.
yes, your solution is correct but not good. I dont know what is Schurs inequality.
We only need prove that:
x3 + y 3 + z 3 + 3 2(x2 + y 2 + z 2 )
(1 + x3 2x2 + x 1) + (1 + y 3 2y 2 + y 1) + (1 + z 3 2z 2 + z 1) 0
x(x 1)2 + y(y 1)2 + z(z 1)2 0
167,
r

r
r
a + 2b
b + 2c
c + 2a
+
+
3; a, b, c 0
a + 2c
b + 2a
c + 2b
168, F A stronger and harder one:
r
r
r
a + 2b
b + 2c
c + 2a
+
+
3; a, b, c 0
c + 2b
a + 2c
b + 2a
169, Let a, b, c be nonnegative real numbers. Prove that
a + 2b + 3
b + 2c + 3
c + 2a + 3
+
+
3
c + 2b + 3
a + 2c + 3 b + 2a + 3
My solution assume c = min(a, b, c) and set c + 1 = z, a + 1 = z + m, b + 1 =
z + nwithm, n 0
Setting x = a + 1, y = b + 1, z = c + 1, then the inequality becomes
X x + 2y
3
z + 2y
70

(x + 2y)(2z + x)(2x + y) 3(2x + y)(2y + z)(2z + x)


X
X
X

(x + 2z)(2x2 + 5xy + 2y 2 ) 3(9xyz + 4


x2 y + 2
xy 2 )
X
X
X
X
X
2
x3 + 9
x2 y + 6
xy 2 + 30xyz 3(9xyz + 4
x2 y + 2
xy 2 )
X
X
2
x3 + 3xyz 3
x2 y
By Schurs Inequality, we get
X
X
X
x3 + 3xyz
x2 y +
xy 2
It suffices to prove that
X

x3 +

xy 2 2

x2 y

x(x y)2 0

which is true.
170, And general problems:
1,
2,

b + 2c + k
c + 2a + k
a + 2b + k
+
+
3f ork 0
c + 2b + k
a + 2c + k b + 2a + k

a + mb + n
b + mc + n
c + ma + n
+
+
3f orm, n 0
c + mb + n
a + mc + n b + ma + n

1/ Setting x = a + k3 , y = b + k3 , z = c + k3 , the inequality becomes


X x + 2y
z + 2y
It is the previous.
2/ Setting x = a +

n
m+1 , y

=b+

n
m+1 , z

=c+

X x + my
z + my

n
m+1 ,

the inequality beomces

This inequality is not always true.


171, Prove if a, b, c > 0; abc = 1 then
X a2
a3 + b3 + c3

>=
>=
b+c
3abc

abc(a + b + c)

Assume that :abc = 1


F:

X a2
X 2a2
X
X
3
3
3
2 4

a +b +c
a a=

4
b+c
bc
71

Done
F
3

X
X 1
a2
3 3

(
a2 )(
) (a + b + c)2 p
..
b+c
b+c
2(a + b + c)

172, Let a, b, c > 0 and abc = 1. Prove that


a3 + b3 + c3 3

a
3

b+c 2

ts easy to prove by use SOS method. the inequality is equivalent to


X
Sc (a b)2 0
with
Sa = a + b + c

1
>0
(a + b)(a + c)

similar with Sb , Sc we get the same result.so the inequality has been proved.
X a
X 2a
X a

+
a3 + b3 + c3 a a + b b + c c =
b
+
c
b
+
c
2
bc
By the hypothesis, we have
a3 + b3 + c3 =

X a2
bc

4.

a 2
4 X a 2 X a
3
) .(
)
+
b+c
3
b+c
b+c 2

173, This inequality is true ,but very easy:


X a
3
3
2
b+c
X
X
X a
2RHS
2LHS 3
a2 3
a a=3
bc
=>done By schur
X
a3 + b3 + c3 + 3abc
ab(a + b)
a3 + b3 + c3 +

abc = 1 > a3 + b3 + c3 + 3

Xa+b
c

174, Let a, b, c > 0 such that abc = 1. Prove that


X

1
1
+
1
(1 + a)2
1+b+c+a

Easy expand
<=> q 2 2qp + p3 5p 3 0
0 = p2 + 3 + 5p p3 = (p 3)(1 + p)2 0
72

a
b+c

175, Let a, b, c 0. Prove that:


1
1
1
1
+
+
2
5(a2 + b2 ) ab 5(b2 + c2 ) bc 5(c2 + a2 ) ca
a + b2 + c2
Its very easy. We need prove
X

5c2 + ab
2
+ b2 ) ab

5(a2
Using Cauchy Schwarz,we have:
X

P
P
5c2 + ab
(5 a2 + ab)2
P

(1)
5(a2 + b2 ) ab
(5(a2 + b2 ) ab)(5c2 + ab)

Setting q = ab + bc + ca, r = abc, p = a + b + c = 1. We have


X
X
X
(1) (5
a2 +
ab)2
(5(a2 + b2 ) ab)(5c2 + ab)
(5 9q)2 2(108r + 39q 2 + 5q)
(q 1)(3q 1) 0
176, Let a, b, c > 0 such that ab + bc + ca = 3. Prove that
p

3 (a + b)(b + c)(c + a) 2( a + b + b + c + c + a) 6 2
F First, we prove the left ineq. Setting :x = ab; y = bc; z = ca => x + y + z = 3
By cauchy-schwarz , we need to prove :
9(x + y)(y + z)(z + x) 8(x + y + z)(xy + yz + zx)(Right)
F, Next, we prove the right ineq :
By Am-Gm, we need to prove
(a + b)(b + c)(c + a) 8
<=> 3(a + b + c) abc 8(Rightbecauseab + bc + ca = 3)
177, Let a, b, c > 0 such that a2 + b2 + c2 = 3. Prove that:
(2 a)(2 b)(2 c)

25
27

Let be c = M in(a, b, c) then c [0, 1]. We have:


2(2a)(2b) = 84(a+b)+2ab = 84(a+b)+(a+b)2 a2 b2 = (a+b2)2 +4a2 b2 c2 +1
Thus


f (c) = (2 c)

73

c2 + 1
2

f 0 (c) = 0 c =


 
1
25
1
, c = 1Hence, M inf (c) = M in f (0), f (1), f
=
3
3
27

The equality holds if and only if a = 35 , b = 13 , c =

1
3

or any cyclic permutations.

178, Solve
log42 (x2 + 4) = 2log 12

p
8x2 + 32 + 6

log2 (x2 + 4) = a
log42 (x2 + 4) = (log4 (x2 + 4))2 =
log 12

8x2 + 32 = log2

a2
4

1
1
8x2 + 32 = (log2 8 + log2 (x2 + 4)) = (3 + a)
2
2

1
a2
= 2( (3+a))+6 = 3a+6 = a+3 > a2 +4a12 = 0 > (a+6)(a2) = 0 >
4
2
a1 = 2 > log2 (x2 + 4) = 2 > x2 + 4 = 22 > x = 0
a2 = 6 > log2 (x2 + 4) = 6 > x2 + 4 = 26 >
179, Let a, b, c > 0 such that a2 + b2 + c2 = 3. Prove that
X a
9

2(a + b + c)
b+c
its very easy for you. we have
X

X a2
a
(a + b + c)2
=

b+c
ab + ac
2(ab + bc + ac)

then in view the problem :


a2 + b2 + c2 = 3 > s2 = 2p + 3
we obsever
s3 + s3 + 27 9s2
<=> (3 s)(9 + 3s 2s2 ) 0
we have

Xa+b
c
2

18
a+b+c

180, Let a, b, c > 0 such that a + b + c2 + d2 = 1. Prove that


(1 a)(1 b)(1 c)(1 d)

(ab + cd)(ac + bd)


4

By CBS
p
(a2 + b2 )(c2 + d2 ) ac + bd

74

we have
2(1 a)(1 d) = (a + d 1)2 + b2 + c2 b2 + c2
2(1 a)(1 d) b2 + c2
2(1 b)(1 c) a2 + d2
p
(a2 + d2 )(b2 + c2 ) (ab + cd)
p
(a2 + d2 )(b2 + c2 ) (ac + bd)
181, Let triangle ABC. Prove that

tan2 A + 3 tan2 B + 15 tan2 C 3 13


I think this problem is: Let AB is a triangle have A; B; C < 90 Prove that
tan2 A + 3tan2 B + 15tan2 C 36
Let tanA = 3x; tanB = 2y; tanC = z(x; y; z > 0) by AM - GM:
p
3x2 + 4y 2 + 5c2 12 12 x6 y 8 z 10
p
3x + 2y + z 6 6 x3 y 2 z
> (3x2 + 4y 2 + 5z 2 )(3x + 2y + z) 72xyz
> 3(3x2 + 4y 2 + 5z 2 ) 36
182, Prove if a, b, c > 0 and ab + bc + ca = 3 then
Xp
a2 + 3 a + b + c + 3
183, If x, y, z are three nonnegative reals, then prove that
Xp

(z + x) (x + y) x + y + z + 3 yz + zx + xy,
cyc

where the cyc sign means cyclic summation.


Applying the Conway substitution theorem (http://www.mathlinks.ro/Forum/viewtopic.php?t=2958
post 3) to the reals x, y, z (in the role of u, v, w), we see that, since the numbers
y + z, z + x, x + y and yz + zx + xy are all nonnegative, we can conclude
that there

exists a triangle ABC with sidelengths a = BC = y + z, b = CA = z + x, c =

AB = x + y and area S = 21 yz + zx + xy.


Now,
Xp
X
X

(z + x) (x + y) =
z+x x+y =
b c = bc + ca + ab;
1
((y + z) + (z + x) + (x + y))
2

2
2
2 

1
=
y+z +
z+x +
x+y
2
x+y+z =

75


1 2
a + b2 + c2 ;
2

1

3 yz + zx + xy = 2 3
yz + zx + xy = 2 3 S.
2
Hence, the inequality in question,
Xp

(z + x) (x + y) x + y + z + 3 yz + zx + xy,
=

becomes


1 2
a + b2 + c2 + 2 3 S.
2
Multiplication by 2 transforms this into


2 (bc + ca + ab) a2 + b2 + c2 + 4 3 S,
bc + ca + ab

or, equivalently,


2 (bc + ca + ab) a2 + b2 + c2 4 3 S.
Using the notation

Q = (b c) + (c a) + (a b)
this rewrites as


a2 + b2 + c2 Q 4 3 S,

what is equivalent to

a2 + b2 + c2 4 3 S + Q.

But this is the well-known Hadwiger-Finsler inequality


Xp
p
(z + x)(z + y) x + y + z + 3(xy + xz + yz)
cyc

2 x+y+z

 X
2

3(xy + xz + yz)
z + x z + y 0.
cyc

But

 X
p
2

2 x + y + z 3(xy + xz + yz)
z+x z+y =
cyc

X
cyc

(x y)2

p

2

x + y + z + 3(xy + xz + yz)
z+x+ z+y


p
p
X (x y)2 z + 2 (z + x)(z + y) 3(xy + xz + yz)


=
p
2 =

3(xy + xz + yz)
z+x+ z+y
cyc x + y + z +

76

!
=


(x y)2 z +
=

X

cyc

x+y+z+

4z 2 +xy+xz+yz

(z+x)(z+y)+ 3(xy+xz+yz)


2 0.

3(xy + xz + yz)
z+x+ z+y

184, Prove if a, b, c > 0 such that ab + bc + ca = 3 then


2b
3c
49
a
+
+

2
2
2
12
3+a
3+b
3+c
X
X
2xycosC
x2 > OK
P
185, Prove if a, b, c > 14 such that
a 3 then

Xp
a2 + 3 a + b + c + 3
let prove
p
1
a2 + 3 (a + 1) f(x) denotea >
4
p

1
a2 + 3 a a + 2witha.
4
p

a2 + 3 a a + 2
< > (4a 1)(a 1)2 0
186, Let a, b, c > 0 such that a+b+c=1 Prove that
1
1

X (a + b)b
cyc

c+a

X (a + b)b
c+a

(a4 a2 b2 + a3 c a2 bc) 0, which obviously true.

cyc

X b(a + b)
c+a
<>

X a(a + b + c) + b(a + b + c a c)
<>

c+a
X a+b
a+c

187, Let ABC is a triangle. Prove that


X
1
>=
B
C
A
sin 2 sin 2 sin 2
() < >

cos BC
2
+2
sin B2 sin C2

X
a

pa

77

b+c
pa

because
X

Xc+b
a

pa
a

BY CBS
Let x = p a > 0 : y = p b > 0; z = p c > 0 we have
Xx+y
z

>
X
> (

x
2x + y + z

y+z
y+z

Xb+c
a

pa
a

X a
Xb+c
X
a 2
) (
)(
)(
pa
oa
a

b+c 2
)
pa

188, Let a, b, c > 0 such that a4 + b4 + c4 = 3. Prove that


X

a
3

b + c3
2

By Trebusep
188, Let a, b, c > 0 such that abc = 1. Prove that
X

1
1
+
1
2
(1 + a)
1+b+c+a

Easy. Expand
<=> q 2 2qp + p3 5p 3 0
0 = p2 + 3 + 5p p3 = (p 3)(1 + p)2 0
189, Let a, b, c, x, y, z > 0. Find the minimal value of the expression
P =

(a3 + x3 + 1)(b3 + y 3 + 1)(c3 + z 3 + 1)


(a + b + c)(x + y + z)

By Holder
190, Prove that the sides a, b, c of any triangle suck that a2 + b2 + c2 = 3 satisfy
the inequality
X
a
1
2
a +b+c
P
a + b + c + 2ab + 2bc + 2ca
cyc a(1 + b + c)
LHS
=
1
(a + b + c)2
3 + 2ab + 2bc + 2ca
191, Prove if a, b, c > 0 then
X
p

(a + b)(b + c) a b + c 4(a + b + c) (a + b + c)(a b + c)(a + b c)

78

b + c a = x2 , a + c b = y 2 , a + b c = z 2
We want to prove
X
x(2y 2 + z 2 + x2 )(2z 2 + y 2 + x2 ) 16xyz(x2 + y 2 + z 2 )
X

X
x(2y 2 +z 2 +x2 )(2z 2 +y 2 +x2 ) =
x((x4 +y 2 z 2 )+(2y 4 +2y 2 z 2 )+(2z 4 +2y 2 z 2 )+(3x2 y 2 +3x2 z 2 ))
X
(8x3 yz + 4y 3 zx + 4z 3 xy) = 16xyz(x2 + y 2 + z 2 )

The inequality is equivalent to


(a + b)(b + c)

X
p

(b + c a)(c + a b)

4(a + b + c)

From AM-GM we get


(a + b)(b + c)
p

(b + c a)(c + a b)

2(a + b)(b + c)
(a + b)(b + c)
=
b+ca+c+ab
c

Therefore it remains to show that


X (a + b)(b + c)
c

4(a + b + c)

(1)

Since the sequences { a1 , 1b , 1c } and{(c + a)(a + b), (a + b)(b + c), (b + c)(c + a)} are
oppositely sorted, from Rearrangement we get
X (a + b)(b + c)
c

X (a + b)(b + c)
b

=a+b+c+

ca
b

Therefore it remains to show that


X ca
b
which follows from Rearrangement
X ca
b

a+b+c

X ca
c

=a+b+c

192, Let a, b, c > 0 such that a2 + b2 + c2 = 3. Prove that


X

a+b
6
a+bc

P2: If a, b, c > 0 then


r
4
4
4
b2
c2
a2
4 a + b + c
+
+
3
b
c
a
3
By Holder ,we have
(

a2
b2
c2
+
+ )2 (a2 b2 + b2 c2 + c2 a2 ) (a2 + b2 + c2 )3
b
c
a
79

and

(a2 + b2 + c2 )6
27(a4 + b4 + c4 )
+ b2 c2 + c2 a2 )2

(a2 b2

Let x2 = a + b + c; ....... then a =

y 2 +z 2
2 ; ....

By P2 we have done

193, Prove If a, b, c > 0 then

a2
b2
c2
323
+ 3
+ 3
p
b3 + c3
c + a3
a + b3
2 2 (a2 + b2 + c2
We assume a2 + b2 + c2 = 3 then the inequality becomes
b2
c2
3
a2
+
+

b3 + c3
c3 + a3
a3 + b3
2
Note that for a, b, c 0 and a + b + c = 3 then
3

a2 b + b2 c + c2 a 3
By the Cauchy Schwarz we get
2
a2 + b2 + c2
9
a2
b2
c2
3
+ 3
+ 3
P 2 3 P 3 2 =
a b + a b
b3 + c3
c + a3
a + b3
6
2
Let a2 + b2 + c2 = 3. Then we need to prove that
X
cyc

But
X
cyc

3
a2
.
3
3
b +c
2

X
a2
a4
9
.
=
P 3 2
3 a2 + c3 a2
(a
b
+ a3 c2 )
b3 + c3
b
cyc

Id est, it remains to prove that


X

(a3 b2 + a3 c2 ) 6.

cyc

But

(a3 b2 + a3 c2 ) 6

cyc

a3 (3 a2 ) 6

cyc

(a5 3a3 + 2) 0

cyc

(a5 3a3 + 2 + 2(a2 1)) 0

cyc

a2 (a + 2)(a 1)2 0.

cyc

80

194, Let a, b, c > 0 such that ab + bc + ca = 1. Find min:


M=

a2
b2
c2
+
+
2(a2 + b2 + c2 )
b
c
a

Its an old problem of canh ang. We can solve it by the lenma:


b2
c2
(a2 + b2 + c2 )(a + b + c)
a2
+
+

b
c
a
ab + bc + ca
195, SOLVE

1)x2 + x 1 = xex

+ (x2 1)ex

2)2x2 6x + 1 = log2

2x + 1
2(x 1)2

196, Let a, b, c > 0 abc = 1. Prove that


(a + b)(b + c)(c + a) + 7 5(a + b + c)
(a + b + c)(ab + bc + ca) + 6 5(a + b + c)
ab + bc + ca +

6
5
a+b+c

setting
F (a, b, c) = ab + bc + ca +

6
a+b+c

F (a, b, c) F (a, bc, bc) = ( b c)2 (a f rac6(a + b + c)(a + 2 bc))


assume


a = max(a, b, c) F (a, b, c) F (a, bc, bc)

thus, we need prove


F (1/t2 , t, t) 5
(a + b)(b + c)(c + a) + 7 5(a + b + c)
(a + b + c)(5 ab bc ca) 6.
oh, after an hour for it, I have an interesting solution Very Happy with:
(a + b)(b + c)(c + a) = (a + b + c)(ab + bc + ca) abc
and continue with AM-GM
Similar to it, we have:
(a + b)(b + c)(c + a) + 3n 8 n(a + b + c)(n 3)
197, Let a, b > 0. Prove that

b
a
+
1
a2 + 3b2
b2 + 3a2
81

By Holder
2

b
a

+
(a(a2 + 3b2 ) + b(b2 + 3a2 )) (a + b)3 =
a2 + 3b2
b2 + 3a2
= a(a2 + 3b2 ) + b(b2 + 3a2 ).
198, Let a, b, c > 0 such that a2 + b2 + c2 = 3. Prove that
a3 (a + b) + b3 (b + c) + c3 (c + a) 6
We have
(a + b + c)(a3 + b3 + c3 ) (a2 + b2 + c2 )2
and
3(a3 c + b3 a + c3 c) (a2 + b2 + c2 )2
199, Let a, b, c > 0. Prove that
X

2a

b+c

Xa
3(
)
b

Well, you can easily prove by AM-GM that:


r
Xa
3(
) 3.
b
So it suffices to show that:
X

2a
3,
b+c

which is a known Vasc inequality.


200, The following inequality is true too
X 2a
Xa

a, b, cbethesidelengthsof atriangle
b+c
b
201, Let a, b, c be positive number such that a2 + b2 + c2 = 34 . Prove that
ab
bc
ca
+
+
1
ab + 1 a bc + 1 b ca + 1 c
Its easy by CS! Razz 202, Find the best positive constant k such that the following
inequalitys right
ab
bc
ca
+
+
1
ab + k(k a) bc + k(k b) ca + k(k c)
for all positive numbers a, b, c such that a2 + b2 + c2 = 1. Is k =

2 3
3

203, Let a, b, c be positive number such that a + b + c = 1. Prove that


1)

b(b + c 2a) c(c + a 2b) a(a + b 2c)


+
+
0
3ab + 2b + c
3bc + 2c + a
3ca + 2a + b
82

ab
bc
ca
1
+
+

3ab + 2b + c 3bc + 2c + a 3ca + 2a + b


4
problem 2,you dedicated to quykhtn-qa1?Am I right?
2)

204, Let a, b, c be positive numbers such that a + b + c = 1. Prove that


ab
bc
ca
1
+
+

3ab + 2b + c 3bc + 2c + a 3ca + 2a + b


4
205, Let x, y, z be positive numbers such that x2 + y 2 + z 2 = 6 and A, B, C are three
angles of an acute triangle. Prove that
X
cyc

1
1
1 + yzcosA + xyz 2 cosAcosB

Use:

P 2
q X
X
x
3
xyz 2 cosAcosB
yzcosA
2
and Cauchy-Schwarz inequality,we have Q.E.D
206, Let a, b, c be positive number such that

a
1+a

b
1+b

c
1+c

= 1. Prove that

2a 1
2b 1
2c 1
+
+
0
1 + 2a + 4ab 1 + 2b + 4bc 1 + 2c + 4ca
207, Let a, b, c be positive numbers. Prove that
1 1 1
(a + 2b)2
(b + 2c)2
(c + 2a)2
(a + b + c)( + + ) 2
+ 2
+ 2
a b
c
a + b(c + a) b + c(a + b) c + a(b + c)
LHS =

X
cyc

(1 +

X a2
X a + 2b2
b
b
4b2
+ )=
+

= RHS
c a
a2
bc + ba
a2 + bc + ba
cyc
cyc

208, Let a, b, c be positive numbers. Prove that


a3
b3
c3
b+c
ca2
+
+

+
a2 + b2
b2 + c2
c2 + a2
2
c2 + a2
Althought this ineq may be seem ugly, but the equality holds when a = b = c.
It is weaker than:
a3
b3
c3
a+b+c
+
+

a2 + b2
b2 + c2
c2 + a2
2
and an inequality very strong
a3
b3
c3
+ 2
+ 2

2
2
2
a +b
b +c
c + a2

83

3(a2 + b2 + c2 )
2

X
cyc

a2

X
X
a3
ab2
ab2
1X
=
a 2
>
a
=
a
2
2
+b
a +b
2ab
2 cyc
cyc
cyc

It remains to prove
b+c
ca2
a+b+c
>
+ 2
2
2
c + a2
which is equivalent to ac2 + a3 > 2ca2 which is true by AM-GM
209, Let x, y, z > 1. Prove that
1 + x2
1 + y2
1 + z2
+
+
2.
2
2
1+y+z
1+z+x
1 + x + y2
Ill try to clarify. As x
X

1+x2
2

we have

X
2(1 + x2 )
1 + x2

2
1+y+z
(1 + y 2 ) + 2(1 + z 2 )

Denoting 1 + x2 = a and so on we have to prove that


X a
1
b + 2c
but Cauchy tells us
X
and as

X 2
a X
a
a(2b + c)
b + 2c

X 2
X
a(2b + c)
a 3(ab + bc + ca) =

we have the result.


210, Let a, b, c be positive number such that a2 + b2 + c2 = 32 . Prove that
b2

p
b3
c3
a3
+ 2
+ 2
+ 2 3(a6 + b6 + c6 ) 3(a3 + b3 + c3 )
2
2
2
+c
c +a
a +b

211, Solve the equation:


p
p
p
13x2 + 8x + 5 + 29x2 24x + 5 = 2 12x2 + 4x 1
squaring both sides of your equation and simplifying we get
p
p
2 13x2 + 8x + 5 29x2 24x + 5 = 6x2 + 32x 14
squaring again and factoring we have
16(23x2 + 12x 6)(2x 1)2 = 0

84

The only solution which fulfills our equation is x = 21


p
p
p
13x2 + 8x + 5 + 29x2 24x + 5 = 2 12x2 + 4x 1
p
p
p
(3x + 2)2 + (2x 1)2 + (5x 2)2 + (2x 1)2 = (8x)2 (4x 2)2
Note that:
p
(3x + 2)2 + (2x 1)2 |3x + 2|
p
(5x 2)2 + (2x 1)2 |5x 2|
p
(8x)2 (4x 2)2 |8x|

Therefore:
p
p
p
(3x + 2)2 + (2x 1)2 + (5x 2)2 + (2x 1)2 |3x+2|+|5x2| |(3x+2)+(5x2)| = |8x| (8x)2
with equality occuring when
(2x 1)2 = (4x 2)2 = 0 x =

1
.
2

212, Let ABC be an acute triangle. Prove that


cos4 A2
cos4 B2
cos4 C2
B
C
A
B
C
A
+
+
4 cos cos cos (cos + cos + cos )
A
B
C
2
2
2
2
2
2
2
2
2
sin 2
sin 2
sin 2
Here is my proof. Let b + c a = x, a + c b = yanda + b c = z. Hence,
A
2
2 A
sin
2
cyc

X cos4


X 1+
cyc

b2 +c2 a2
2bc

1 b

cos2

cyc

B
C
A
cos cos
2
2
2

2

2 +c2 a2
2bc

X
cyc

b2 + c2 a2
1+
2bc

 s

a2 + c2 b2
1+
2ac



a2 + b2 c2
1+

2ab

p
X (b + c a) (a + b c)(a + c b)
(b + c a)2

bc(a + b c)(a + c b)
abc bc
cyc
cyc
p
X (b + c a) (a + b c)(a + c b)
X
(b + c a)2

bc(a + b c)(a + c b)
abc bc
cyc
cyc
s
X
X (y + z)x2
4x2 yz

2yz
(x + y)(x + z)
cyc
cyc
s
X
X
4x2 yz
3
3

(x y + x z) 2xyz
(x + y)(x + z)
cyc
cyc
X

85

which is true because


X

(x3 y + x3 z) 2xyz(x + y + z)

cyc

and
X
cyc

1
x+y+z

s
X
cyc

X
4x2 yz
y+z
=
(x + y)(x + z)
2(x
+ y + z)
cyc

8x2 yz
=
(x + y + z)(x + y)(x + z)(y + z)

16x2 yz

(x + y)(x + z)(y + z)2

8xyz
1
(x + y)(x + z)(y + z)

Id est, your inequality is proven.


213, Let a, b, c be positive number such that a + b + c = abc. Prove that

bc
ca
ab
a+b+c
3 3

+
+

4
a(1 + bc) b(1 + ca) c(1 + ab)
4
use the inequality:
4
1
1
+
x+y
x y
we obtain:

4bc
bc
bc

+
2a + b + c
a+b a+c
ca
ac
4ac

+
2b + a + c
a+b b+c
4ab
ab
ab

+
2c + a + b
b+c a+c
From Titus lemma, we have:

P 2
P 2
X ab
ab)
ab)
(
(
3 3abc
3 3

=
c + abc
a + b + c + 3abc
4abc
4abc
4

214, Let a, b, c be positive number. Prove that


X

3
a2

4
3
3
5
a + 24b c

a2
3

4
2
2
5
a + 24b c

cyc

I suppose you mean:


X
cyc

or equivalently:
P =

X
cyc

a
3

5
a2 + 24bc
86

Then Holder gives us:


!3

!
X

a + 24abc P

cyc

cyc

so it suffices to prove
!3
X

25

!
X

cyc

a + 24abc

cyc

which is obvious upon expanding


Another solution is(dedicated to quykhtn-qa1):
r
X
X
X
a2
1
a4

q
=
=
4 + 24b2 c2
4
2
2
2 2 2
a
a + 24b c
1 + 24a b c
a6

We consider the function

f (x) = q

1+

24a2 b2 c2
x6

f 00 (x) 0
So
f (a) + f (b) + f (c) 3f (
So the
LHS 3

a+b+c
)
3

X
q

1+

36 24a2 b2 c2
(a+b+c)6

So we have to prove that:


1

X
q

1+

36 24a2 b2 c2
(a+b+c)6

1
(a + b + c)6 36 a2 b2 c2
5

which is true .
214, Let a, b, c be positive number such that a + b + c = 1. Find the minimum of
P =

b
c
a
+
+
2
2
2
2
1+b +c
1+c +a
1 + a2 + b2

bu Cauchy-Schwarz,


X
S =P a+b+c+
ab(a + b) (a + b + c)2 = 1

87

by Schur we can prove that,


X
ab(a + b) a3 + b3 + c3 + 3abc a3 + b3 + c3 + 6abc
4

ab(a + b) a3 + b3 + c3 + 3

then

ab(a + b) + 6abc = (a + b + c)3 = 1

ab(a + b)


P

1+

1
4

1
4


S1

and finally
4
5
< a, b, c < 1 and
Pmin =

216, Let a, b, c be satisfying

1
2

a4 + b4 + c4 + 4a2 b2 c2 = 2(a2 b2 + b2 c2 + c2 a2 )
Prove that
X
cyc
2 2

2 2

a4
4abc(a + b + c)
1 a2

2 2

As a b + b c + c a abc(a + b + c) we can even prove the more stronger inequality


still holds:
If 21 < a, b, c < 1 such that
4abc = 2(ab + bc + ca) a2 b2 c2 .
Then

a2
b2
c2
+
+
4(ab + bc + ca).
1a 1b 1c
This inequality follows from the following inequality
8(a2 + b2 + c2 ) (ab + bc + ca)

21abc(a + b + c)
2(ab + bc + ca) a2 b2 c2

if a, b, c are the sidelengths of a triangle.


216,Let a, b, c R+ and

X a3 + 2a
cyc

Prove that

we have

a2 + 1

9
2

X1
3
a
cyc
X a
X
9 X
=
a+
sowehave
a3
2
2
a +1
88

but

X
9 X 1 X a4 + a2 1 X
=
+
=
a+
f (a)
3
2
a
a +a

where
f (a) =
but
f 00 (a) =

a3 + a

2(6a4 + 3a2 + 1)
< 0f oreverya > 0
a3 (a2 + 1)3

so we have
X

X a4 + a2 1

f (a) 3f (

a+b+c
3
) 3f (1) =
3
2

so we have

X1
3
a
217, Let a, b and c are non-negative numbers such that ab + ac + bc = 3. Prove that:
1) a2 + b2 + c2 + 3abc 6
Using Schurs inequality
4(ab + bc + ca)(a + b + c) 9abc (a + b + c)3
Then

9abc
and
a+b+c
9abc
+ 3abc RHS
LHS = (a + b + c)2 6 + 3abc 6
a+b+c
(a + b + c)2 12

because
(a + b + c)2 3(ab + bc + ca)
2) a4 + b4 + c4 + 15abc 18
assume that: a + b + c = p, ab + bc + ca = q, abc = r so q = 3 and we have to prove
that:
p2 (12 p2 )
r
15 + 4p
case1:if p2 > 12 the ineq is true
case2: if p2 12
remember this schur ineq
r

(p2 3)(12 p2 )
6p

we will prove that


(p2 3)(12 p2 )
p2 (12 p2 )

6p
15 + 4p

89

2
which is
equivalent to (p 3)(2p 9p 15) 0 (which is obvious true for all
p 12).

218, Let ABC be an acute triangle. Prove that:


p
cos A + cos B + cos C sin2 A + sin2 B + sin2 C
Let

s
cos A =

bc
(b + a)(c + a)

So we have to prove that:


q X
Xp
2
ab(a + b) + 6abc
ab(a + b)
which is equivalent to:
r

r
Xa+b
a+b
2
+6
c
c
p
cosA + cosB + cosC 3 (cos2 A + cos2 B + cos2 C)
X

(cosA + cosB + cosC)2 + (cos2 A + cos2 B + cos2 C) 3


By Cauchy,we have
(cosA + cosB + cosC)2 + (cos2 A + cos2 B + cos2 C)

4
(cos2 A + cos2 B + cos2 C)
3

It remains to prove
cos2 A + cos2 B + cos2 C

9
4

Its obvious.....
219, Let a, b, c 0 satisfy a + b + c = 1. Prove that
(a2 + b2 )(b2 + c2 )(c2 + a2 )

1
32

let
f (a, b, c) = (a2 + b2 )(b2 + c2 )(c2 + a2 )
letc = max(a, b, c); we have
f (a, b, c) f (a + b, 0, c)
(which is equivalent)
ab(4abc2 + a3 b + ab3 4a2 c2 4b2 c2 2c4 ) 0

90

we will prove that


f (a + b, 0, c) = f (1 c, 0, c)

1
2

which is equivalent to
1
(16c4 32c3 + 20c2 4c 1))(1 + 2c)2 0
32
remember that

1 5
1+ 5
2
16c 32c + 20c 4c 1 = 4(2c 2c +
)(2c 2c +
)0
4
4
4

for every c [0, 1].


220, Let 0 < x, y, z < 1 and xy + yz + zx + 2xyz = 1. Prove that:
p
X
x(1 x)
p
p
x+y+z
y(1 y) + z(1 z)
221, Let a, b, c > 0 such that 4abc = a + b + c + 1. Prove that
b2 + c2
c2 + a2
a2 + b2
+
+
2(ab + bc + ca)
a
b
c
Another approach: By AM - GM, we obtain:
LHS

2
2bc 2ca 2ab
+
+
=
((ab)2 + (bc)2 + (ca)2 )
a
b
c
abc

But
(ab)2 + (bc)2 + (ca)2
= LHS

1
(ab + bc + ca)2
3

2
(ab + bc + ca)2
3abc

Thus, it is enough to prove


2
(ab + bc + ca)2 2(ab + bc + ca)
3abc
ab + bc + ca 3abc.
Indeed, from the condition, by AM - GM, we obtain:

4
4abc = a + b + c + 1 4 abc = abc 1
= a + b + c 4abc 1 3abc
But
(ab + bc + ca)2 3abc(a + b + c) 9a2 b2 c2 = ab + bc + ca 3abc.
91

The result is lead as follow.


222, Let a, b, c be positive real numbers,prove that
X a + b
c
X a + b
c

X a 
X
4
+4
b+c

a(b c)

(b + c)

X  a  (x + y)(y + z)(z + x) 8xyz


4
+
b+c
(x + y)(y + z)(z + x)

My soulution
<=>

a(b c)2 (

1
4

)0
bc(b + c (b + c)

We have: (b + c)3 4bc(b + c).


LHS RHS =

P
(a + b + c) ( a3 (b c)2 )
abc(a + b)(b + c)(c + a)

223, Let a, b, c > 0. Prove that:


3(a + b + c)
a2 + 2bc b2 + 2ca c2 + 2ab
+
+
>
b+c
c+a
a+b
2
We can prove it easily by CS and Voni Schur Smile
Besides, the stronger of Nguyen Van Thach is very hard:
r
X
a2 + 2bc
3
a+b+c
b+c
2
224, Let a, b, c > 0. Prove that:
r
X

(b + c)

(b + a)(c + a)
4(a + b + c)
bc

Its very easy but nice Very Happy


By cauchy-swarchz anh Am-Gm we have:
LHS

X a(b + c)
a

(b + c)(1 + ) = 2(a + b + c) +
4(a + b + c)
bc
bc

225, Let 0 < a, b, c < 1. Prove that


1a
1b
1c
+
+
3(1 a)(1 b)(1 c)
1+b+c 1+c+a 1+a+b
we have
(1 + b + c)(1 b)(1 c) 1(cauchy)
so

1a
(1 a)(1 b)(1 c)
1+b+c
92

226, This one is the correct one Very Happy


a4 + b4 + c4
3abc
2(a2 + b2 + c2 )
+

ab + bc + ca a + b + c
3
227,

a4 + b4 + c4
3abc
(a2 + b2 + c2 )
+9
14
ab + bc + ca
a+b+c
3
But the slightly stronger is wrong:
5

3abc
(a2 + b2 + c2 )
a4 + b4 + c4
+ 10
15
ab + bc + ca
a+b+c
3

228, Problem. For all positive real numbers a,b,c we have the following inequality
r
6
6
6
a2
b2
c2
6 a + b + c
+
+
3
b
c
a
3
Use the following
X a2
b

a4 + 13

P
P
a3 (b + c) a2 b2 a2 bc
P P
3( a)( ab)

With k= 6, we can use this lenma:


X a2
6(a2 + b2 + c2 ) 3(ab + bc + ca)

b
a+b+c
With this lenma, I have had a nice proof by Am-Gm.
229, Let a, b, c > 0. Prove that:
2

a2 + b2 + c2
(a b)2 + (b c)2 + (c a)2
a3 + b3 + c3 + abc
(
) +
4abc
ab + bc + ca
8(ab + bc + ca)
230, Let be x, y, z R+ . Show that :




p
3
3
3
2
2
2
x +
y +
z +
(x + y)(y + z)(z + x)
4
4
4
Because

3
x +
4
2



3
2
y +
x+y
4

We have
3
x + =
4
2

1
x +
4
2

1
+ 2
2

s 

1
1
x2 +
2
4

Similarly we obtains


3
x +
4
2

s 






3
3
1
1
1
1
2
2
y +
z +
8
x2 +
y2 +
z2 +
4
4
8
4
4
4
93

So ineq at first equivalent to:


Y

Y

4x2 + 1 8
(x + y)

Then apply Cauchy-Shwar we have




2
4x2 + 1 1 + 4y 2 4 (x + y)
similarly and multiply we have finished. 231, Let x, y, z be the non-negative real number satisfying (x + y + z)2 + xy + yz + zx = 2. Prove that

x+y
y+z
z+x
3 2
p
+p
+p

2
z 2 + xy + 1
x2 + yz + 1
y 2 + zx + 1
we can let a = x + y, b = y + z, c = z + x then we have ab + bc + ca = 2 and we have
to prove that:
X
3
a

(1)
2 + 3bc
2
a
cyc
Just use Holder, let the LHS be S, then by Holder
X
S2(
a(a2 + 3bc)) (a + b + c)3
So we have to show
4(a + b + c)3 9(a3 + b3 + c3 + 9abc)
which is obvious by Muirhead. I will must prove that:
2(ab(a + b) + bc(b + c) + ca(c + a)) a3 + b3 + c3 + 9abc
<=> (3c a b)(a b)2 + (3a b c)(b c)2 + (3b a c)(c a)2 0()
Suppose a b c We have:
3b a c 3b c (b + c) = 2(b c) 0, , (c a)2 (a b)2 + (b c)2
then
LHS (3c a b + 3b a c)(a b)2 + (3b a c + 3a b c)(b c)2 0
x + y + z X x2 y 2
9
(
)
2
xyz
(x + y)
4
but:

P
x+y+z
( xy)2
LHS
P
xyz
(x + y)2

remember that:
X

(x + y)2

4
(x + y + z)2
3

94

LHS

P
P
x + y + z 3 ( xy)2
3
( xy)2
9
P 2 =

xyz
4 ( x)
4 xyz(x + y + z)
4

232, For any triangle with sides a, b, c. Prove that


a2 b(a b) + b2 c(b c) + c2 a(c a) 0
Let a = x + y, b = y + z, c = z + x; after expanding,we need to prove that:
X
x3 y (x + y + z)xyz
cyc

233, For all positive real numbers a, b and c


s
s
s
2a(b + c)
2b(c + a)
2c(a + b)
a+b+c
+
+

3
(a + b)(a + c)
(b + c)(b + a)
(c + a)(c + b)
abc
LHS 3 RHS
234, Given a, b, c 0 satisfy a + b + c = 6. Prove that:
(11 + a2 )(11 + b2 )(11 + c2 ) + 120abc 4320
Equality occurs when (a, b, c) = (1, 2, 3).
LHSRHS = 11(ab+bc+ca11)2 +(abc6)2 +121(a+b+c)2 36 435636 = 4320
Equality occurs if f a+b +c = 6, ab +bc +ca = 11 and abc = 6ora = 1, b = 2, c = 3
or any symmetric permutation.
ab + bc + ac
a3 + b3 + c3
+9 2
12
abc
a + b2 + c2
X
X
X 1
9
LHS RHS = (
a2
ab)(
P )0
ab
ab
It is instantly solved by SOS
Sa = Sb = Sc =

(a + b + c)(a2 + b2 + c2 ) 9abc
0
2abc(a2 + b2 + c2 )

235, For all positive real numbers a, b and c. Prove that


P 2 2
a3
1
a
+ (P )
4abc
4
ab

236, let a, b, c be positive numbers such that: abc = a + b + c + 2. Prove that


1 1 1
abc 2( + + ) 5
a b
c
95

we need to prove that:


P

ab
5
abc
X
X
X
(
a + 2)2 2(
ab) 5(
a + 2)
X
X
a2
a+6
abc

remember that:

X
abc = a + b + c + 2then
a6
X
X
(a2 + 4)
4a
X
3
a 18
P 2
237, Let x, y, z > 0 and x + 2xyz = 1. Prove that:
X
3xyz 2
x2 y 2
q
ab
... This ineq becomes:
Let x = (a+c)(b+c)
X

3
a

b+c
2

238, Prove that in any triangle ABC exists the relation


r
X a
b+ca
+2
6
a
b+c
the inequality
r
<=>
Remember that:

Xb+ca
b+ca
2
a
b+c

(b + c) 2 b + c a a

so we get:
r

b+ca
b+ca
2
a
b+c
239, Let a, b, c be the length of the sides of triangle ABC. S is the area of ABC and
0 < a < b < c. Prove that:
X

ab(a b) 2
) 4S
(b c)(c a)

use Cauchy, and this inequality:


abc

4 p 2
a + b2 + c2 S
3
96

And I will prove this inequality:


s
27abc
4 p
3
a2 + b2 + c2 1
2
2
2
(a b) (b c) (c a)
3
240, Let a, b, c be the lengths of the sides of triangle ABC and R is the circumradius
ang r is the inradius of triangle ABC. Prove this Inq
(a + b + c)2
2r
+
4
2(ab + bc + ca) (a2 + b2 + c2 )
R
t is really a complicated solution and of course it is not necessary for this one Razz

(a + b + c)2
(a + b c)(a + c b)(b + c a)
+
4
2(ab + bc + ca) (a2 + b2 + c2 )
abc

Let a + b c = x, b + c a = y, c + a b = z the inequality is equivalent to:


8xyz
(x + y + z)2
+
4
xy + yz + zx (x + y)(y + z)(z + x)

x2 + y 2 + z 2
8xyz
+
2
xy + yz + zx (x + y)(y + z)(z + x)

241, Let be a, b, c > 0. Prove that :




a b
c
2ab + 2bc + 2ca
3
+ +
+
11
b
c a
a2 + b2 + c2


a b
c
a2 + b2 + c2
(a + b + c)2
+ +
=2+

b
c a
ab + bc + ca
ab + bc + ca
LHS 6 +

a2 + b2 + c2
a2 + b2 + c2
ab + bc + ca
+ 2(
+
) 11
ab + bc + ca
ab + bc + ca a2 + b2 + c2

From Cauchy
a b
c
(a + b + c)2
+ +
.
b
c a
ab + bc + ca
Let x = a2 + b2 + c2 , y = ab + bc + ca. Then it remains to show that
3(x + 2y) 2y
+
11
y
x
which in turn is equivalent to
(x y)(3x 2y) 0
which is obviously true since x y.
For your ineq,we can prove easily. For example:
c
2(ab + bc + ca)
2(a2 + b2 + c2 ) 2(ab + bc + ca)
a b

+
+46
2( + + ) +
b
c a
a2 + b2 + c2
ab + bc + ca
a2 + b2 + c2
97

a b
c
8(ab + bc + ca)
17
+ + +

b
c a 3(a2 + b2 + c2 )
3
242, Let a, b, c > 0. Prove that

a2 + bc
3 2

b+c
2

With this one, use Holder ineq, we need to prove:


X
X
9X 2
(
a2 +
ab)3
(a + bc)2 (b + c)2 (1)
2
And pqr works here, of course, not so nice (notice that the equality of (1) occurs when
a = b = c or a = b; c = 0.
243, Let a, b, c [0, 1]. Prove that
C=

5
a
+ abc
1 + bc
2

Assume a b c
=> C

a
b+c
a
+
+ abc
+ abc + 1 = V
1 + bc 1 + bc
1 + bc
V =

a(bc2 + bc + 1)
+ 1 5/2
1 + bc

(Because a, b, c [0, 1].


244, Let a, b, c R such that abc = 1. Find max :
1
a+b+4

P =

1
1
a+b+1

it is very easy! we have

with a, b, c > 0 such that abc = 1 and


4
1
1

+
a+b+4
a+b+1 3
245, Let x, y, z be positive real numbers such that x2 +y 2 +z 2 12. Find the minimum
of:
y6
z6
x6

p
+
+
S=
xy + 2 1 + z 3
yz + 2 1 + x3
zx + 2 1 + y 3
Its very easy!
We have

p
2 1 + x3 x2 + 2
98

then, use CS
=> min = 96/5
246, Let a, b, c 0. Prove that
Y
(a4 + 7a2 + 10) 216(a + b + c)3
We have :
Y
(a4 + 7a2 + 10) 729(a2 + 1)(b2 + 1)(c2 + 1) 216(a + b + c)3
your solution is not true! We have
(a2 + 2)(b2 + 2)(c2 + 2) 3(a + b + c)2
and my ineq:
(a2 + 5)(b2 + 5)(c2 + 5) 72(a + b + c)
C-S lemma
a4 + 7a2 + 10 6(a3 + 2)
(a3 + 2)(b3 + 2)(c3 + 2) (a + b + c)3
Its very easy C-S.
247, Let a, b, c be positive numbers such that a + b + c = 3. Prove that
abc +

12
5
ab + bc + ac

abc (b + c a)(c + a b)(a + b c)


and then we have

4X
bc 3
3
12
ab + bc + ca
3
abc +
4(
+
) 3 4.2 3 = 5
ab + bc + ca
3
ab + bc + ca
248, If a, b, c are non-negative numbers, prove that
abc

(a2 + ab + b2 )(b2 + bc + c2 )(c2 + ca + a2 ) (ab + bc + ca)3 .


Because we have

a2 + ab + b2

3 (a + b)
4

so
a2 + ab + b2

 27 (a + b)2 (b + c)2 (c + a)2


c2 + ca + a2
64

b2 + bc + c2

1
2
2
(a + b + c) (ab + bc + ca) (ab + bc + ca)
3

99

249, Let a, b, c 0 and a + b + c = 3. Prove that


4 a2 b + b2 c + c2 a + abc
a+b+c=3
If {p, q, r} = {a, b, c}, p q r
then as
pq pr qr,
a2 b+b2 c+c2 a+abc = a(ab)+b(bc)+c(ca)+b(ac) p(pq)+2q(pr)+r(qr) =

1
(2q)(p+r)(p+r)
2

23
1 (2q) + (p + r) + (p + r) 3
(
) =
= 4.
2
3
2
250, Prove that for all positive real numbres a, b, c
=

(a2 + 2)(b2 + 2)(c2 + 2) 9(ab + ac + bc)


WLOG (a 1)(b 1) 0 We have
(a2 + 1 + 1)(1 + b2 + 1) (1 + 1 + 1)(a2 + b2 + 1)(byT chebychef 0 sInequality.)
and
(a2 + b2 + 1)(1 + 1 + c2 ) (a + b + c)2 3(ab + bc + ca)
251, Let a, b, c > 0 and abc 1. Find minimum of
A=
[

ca
ab
bc
+
+
a2 b + a2 c b2 a + b2 c c2 a + c2 b

bc
ac
ab
b+c a+b a+c
1 1 1
+
+
][
+
+
] ( + + )2
a2 (b + c) b2 (a + c) c2 (a + b)
bc
ab
ac
a b
c
252, Let x, y, z > 0 and xyz = 1. Prove that
x2
y2
z2
3
+
+

1+y 1+z
1+x
2

by CS we have
x2
y2
z2
(x + y + z)2
3
+
+

1+y 1+z
1+x
3+x+y+z
2
since for X = x + y + z we have 2X 2 3X 9 0f orX 3.
We can use Cauchy- Schwartz to solve this problem: We have
1+y
x2
+
x
1+y
4

100

253, a, b, c, d > 0 and a, b, c, d R and a2 + b2 + c2 + d2 = 1. Prove that


(1 a)(1 b)(1 c)(1 d) abcd
Note that
(1 a)(1 b) cd
Since

c +d
2

cd, it suffices to prove that


(1 a)(1 b)

1 (a2 + b2 )
c2 + d2
=
2
2

1 (a + b) + ab

1 a2 + b2

2
2

2 2(a + b) + 2ab 1 (a2 + b2 )


1 2(a + b) + 2ab + (a2 + b2 ) 0
1 2(a + b) + (a + b)2 0
[(a + b) 1]2 0
Similarly, (1 c)(1 d) ab.
254, Let a, b, c be positive real number. Prove that
s
s
s
a3
b3
c3
3
3
++
+ 3 3
1
3
3
3
3
a + (b + c)
b + (b + c)
c + (b + c)3
s
3

a3

a3
a2
2
3
+ (b + c)
a + b2 + c2

We could use the same tehnic as in here. So , from Holder we have :


!3

X
X
a
4
3
3
p
a(a
+
(b
+
c)
)
(a + b + c)
3
3
3
a + (b + c)
Itisenoughtoprovethat(a+b+c)4

X
a(a3 +(b+c)3 ) = (a+b+c)(
a3 +6abc)

255, Let a, b, c and x, y, z be non-negative numbers such that a + b + c = x + y + z.


Prove that
ax(a + x) + by(b + y) + cz(c + z) 3(abc + xyz).
Apply CBS
(a2 x + b2 y + c2 z)(yz + zx + xy) xyz(a + b + c)2
But
(a + b + c)2 = (x + y + z)2 3(xy + yz + zx)
101

therefore
a2 x + b2 y + c2 z 3xyz
Similarly
ax2 + by 2 + cz 2 3abc
Adding up these inequalities yields the desired rezult.
256, Your inequality is

b
c
a
+
+
1
3 3
3 3
a3 + 26abc
b + 26abc
c + 26abc

By Holder we have
X

3
3
a + 26abc

3  X

  X 4
a a3 + 26abc
a

So it is enough to show that


X 4 X

a
a a3 + 26abc
4

X

ab a2 + b2



+6

X


a2 b2 14abc (a + b + c)

which is true.
257, Let a, b, c be positive real numbers such that a + b + c = 1. Prove that
p
p
p
a(a + bc)
b(b + ca)
c(c + ab)
1
+
+

b + ca
c + ab
a + bc
2 abc
p
X a(a + bc)

LHS
2 abc
but
Xp
p
a(a + bc) (a + b + c)(a + b + c + ab + bc + ca) 2(B C s)
so i get
p

a(a + bc)
1

2 abc
abc

by AM-GM we have :
X
and we have :
X a + a + bc
b + ca

p
a(a + bc)
1 X a + a + bc

b + ca
2
b + ca

X
2a + bc
2a + bc
=
b(a + b + c) + ca
(a + b)(b + c)
102

now the inequality become :


X

(2a + bc)(a + c)

(a + b)(a + c)(b + c)

2 abc

258, Prove that for all non-negative real numbers a, b, c we have


a4 + b4 + c4
2(ab + bc + ca)
+
3.
2
2
2
2
2
2
a b +b c +c a
a2 + b2 + c2
Standard Vornicu-Schur will do: using
x2 + y 2 + z 2 xy yz zx =

(x y)(x z)

cycl

the inequality is equivalent to


x(a b)(a c) + y(b c)(b a) + z(c a)(c b) 0
where

2
(a + b)(a + c)
2
2
2
2
2
+b c +c a
a + b2 + c2
(and similarly for y and z). Note that
x=

a2 b2

(a2 + ab + bc + ca)(a2 + b2 + c2 ) (ab + bc + ca)(a2 + b2 + c2 )


a3 (b + c) + a(b3 + c3 ) + bc(b2 + c2 ) 2(a2 b2 + b2 c2 + c2 a2 )
by the AM-GM inequality so that x, y, z 0. And clearly, x y if we assume
a b c.
Suppose that c = min{a; b; c} We can rewrite this inequality :
(ab)2 [

(a + b)2
2
(a + c)(b + c)
2

]+(ac)(bc)[ 2 2

]0
a2 b2 + b2 c2 + c2 a2 a2 + b2 + c2
a b + b2 c2 + c2 a2 a2 + b2 + c2

From c = min{a; b; c},we have

a2 b2

2
(a + b)2
2
0
+ b2 c2 + c2 a2
a + b2 + c2

and

2
(a + c)(b + c)
2
0
a2 b2 + b2 c2 + c2 a2
a + b2 + c2
Here is another solution with AM-GM
We have that
s
ab + bc + ca
(a4 + b4 + c4 )(ab + bc + ca)2
a4 + b4 + c4
+2 2
33 2 2
2
2
2
2
2
2
2
2
a b +b c +c a
a +b +c
(a b + b2 c2 + c2 a2 )(a2 + b2 + c2 )2
103

So we have to show that


(a4 + b4 + c4 )(ab + bc + ca)2 (a2 b2 + b2 c2 + c2 a2 )(a2 + b2 + c2 )2
which is equivalent to
X
(abc2 (a b)(a3 b3 )) + abc(a5 + b5 + c5 abc(a2 + b2 + c2 )) 0
259, Let a, b, c be the positive numbers such that a + b + c = 3. Prove that

a b + ca b c + ab c a + bc
3 2
+
+

c+a
a+b
b+c
2
260, Let a, b, c be real numbers. Prove that:
a2 + b2 + c2 ab bc ca 3(a b)(a c)
the inequality becomes
X

(a2 ab ac + bc) 3(a b)(b c)

(a b)(a c) 3(a b)(b c)

(a b)(a c) + (c a)(c b) 4(a b)(b c)


(a c)(a b c + b) 4(a b)(b c)
(a b + b c)2 4(a b)(b c)
its quivalent to (x + y)2 4xy which is true for all reals number x, y.
261, F Let a, b, c, d are real number such that ad bc =

3. Prove that:

a2 + b2 + c2 + d2 + ac + bd 3
Okie we will prove

a2 + b2 + c2 + d2 + ac + bd (ad bc) 3

d c 3 2
c d 3 2
) + (b + +
) 0
<=> (a +
2
2
2
2
262, For any three positive reals a, b, c. Prove the inequality
a2 + bc b2 + ca c2 + ab
+
+
a+b+c
b+c
c+a
a+b
Its equivalent to;
a2 + ab + ac + bc b2 + ab + ac + bc c2 + ab + ac + bc
+
+
2(a + b + c)
b+c
a+c
a+b
104

(a + b)(a + c) (a + b)(b + c) (a + c)(b + c)


+
+
(a + b) + (a + c) + (b + c)
(b + c)
(a + c)
(a + b)

Let a + b = x, a + c = y, b + c = z we have to show that


xy yz
xz
+
+
x+y+z
z
x
y
equivalent to;
(xy)2 + (yz)2 + (xz)2 xyz(x + y + z)
With true by Am-Gm
(b2 c2 )2 + (c2 a2 )2 + (a2 b2 )2
a2 + bc b2 + ca c2 + ab
+
+
(a + b + c) =
.
b+c
c+a
a+b
2(b + c)(c + a)(a + b)
263, Let a + b + c = 1, a, b, c 0. Prove that

c
a
b
+
+

a+c
b+c
a+b

3
2

Another way:
X

X
a
(a + b + c)2
1
a2

=

=

b+c
(a b + c
(a b + c + b a + c + c b + a
(a b + c + b a + c + c b + a

So now we try to solve that:


1

a b+c+b a+c+c b+a

3
2

(a b + c + b a + c + c b + a

2
3

Equivalent to

By Cauchy Schwarz we have:


p



(a b + c+b a + c+c b + a = a ab + ac+ b bc + ab+ c ac + bc (2((a + b + c)((ab + bc + ca) =
Since

(a + b + c)2
1
=
3
3
So we have:
r

2
(a b + c + b a + c + c b + a
3
r

X a
1X X 1
1
3 3
3

a
p
=
3
3 2(a + b + c)
2
b+c
b+c
(ab + bc + ca

264, Assume that a, b, c are positive reals satisfying a + b + c


a3 c
b3 a
c3 b
3
+
+

b(a + c) c(a + b) a(b + c)


2
105

a
b

+ cb + ac . Prove that

irst use holder or the general of cauchy we have:




a3 c
b3 a
c3 b
a b
c
+
+
(2a + 2b + 2c)( + + ) (a + b + c)3
b(a + c) c(a + b) a(b + c)
b
c a
so:

a3 c
b3 a
c3 b
a+b+c
+
+

b(a + c) c(a + b) a(b + c)


2

but we also have:

a b
c
+ + 3
b
c a
265, Let a, b, c be positive real numbers. Prove the inequality
a+b+c

1
1
3
1
+
+

a (b + 1) b (c + 1) c (a + 1)
1 + abc
He showed that:

(1 + abc)

1
1
1
+
+
a (1 + b) b (1 + c) c (1 + a)


+3

1 + abc + a + ab 1 + abc + b + bc 1 + abc + c + ca


+
+
a + ab
b + bc
c + ca
1+a
b+1
c+1
b(c + 1) c(a + 1) a(b + 1)
=
+
+
+
+
+
ab + a bc + b ca + c
b+1
c+1
a+1

3
3

+ 3 abcwhichisof course 6.
3
abc
The inequality is equivalent to
=

X abc + 1
3
a(1 + b)
cyc
X
cyc

abc + ab
1+a
+
)6
a(1 + b)
a(1 + b)

This ineq is right .( usingAMG M 3times).


Nice and easy. In fact, this is too easy! Wink We have
1 + abc
1 + a + ab + abc
1+a
b(1 + c)
=
1=
+
1.
a + ab
a + ab
a(1 + b)
1+b
Hence, rewrite the inequality in the form:
1+a
b(1 + c)
1+b
c(1 + a)
1+c
a(1 + b)
+
+
+
+
+
6
a(1 + b)
1+b
b(1 + c)
1+c
c(1 + a)
1+a
ab(b + 1)(ca 1)2 + bc(c + 1)(ab 1)2 + ca(a + 1)(bc 1)2 0whichistrue
106

X
1
1
3
1
+
+

(bc2 + bc)(ab 1)2 0.


a(b + 1) b(c + 1) c(a + 1)
1 + abc
cyc
1 + a + ab + abc
1+a
b(1 + c)
1 + abc
+1=
=
+
a(b + 1)
a(b + 1)
a(1 + b)
1+b
So

X 1+a
b(1 + c)
+
6
a(1 + b)
1+b

true by Am - Gm

266, Let a, b, c be non-negative real numbers and let 0 r 2. Prove that


p
p
p
a4 + b4 + c4 + r a2 b2 + b2 c2 + c2 a2 (r + 1) a3 b + b3 c + c3 a
267,
2xy
+
x+y

x2 + y 2
x+y

xy +
2
2

I saw that somebody else posted a solution while I was typing mine Embarassed . they
are basically the same but so I wouldnt have wrote it for nothing I will post it .
r

x2 + y 2
xy
2

!2

r
x2 + y 2
x2 + y 2
0
+ xy 2 xy

2
2

!2
r
r
x2 + y 2
x2 + y 2
x2 + y 2
(x + y)
+ xy + 2 xy
=
+ xy

2
2
2
r
(x y)2
x2 + y 2
(x y)2
x+y
+ xy
q

2
x+y
x2 +y 2
xy
+
2
r
x+y
2xy
x2 + y 2
+ xy
+
2
x+y
2
r
2xy
x+y
x2 + y 2
(x y)2
(x y)2

+
xy +
q

0
2
2

x+y
2
2
2(x + y)
2( x +y
+
xy)
2
r
r

x2 + y 2
x2 + y 2
2
x+y
xy 0 ( x y) (
xy) 0
2
2

( x y)2 ( x + y)2

q
( x y)2
0 2(x2 + y 2 ) x y 0
2
2

2( x +y
+ xy)
2
2

(x y)2 0

107

268, find the minimal of expression P


P (a, b) =

(a +

a2 + b2 )3
ab2

a2 + b2 = a2 + 8.(b2 /8) 9(a2 .b16 /88 )1/9


and
a + 3(a.b8 /84 )1/9 4(a.b2 /8)1/3
269, For all nonnegative real numbers a, b and c, no two of which are zero,
1
1
(a + b + c)4
1
+
+

2
2
2
2
2
(a + b)
(b + c)
(c + a)
4(a + b + c2 )(ab + bc + ca)2
This inequality follows from

 
2
1
1
1
a
b
c
(a2 + b2 + c2 )
+
+

+
+
(b + c)2
(c + a)2
(a + b)2
b+c c+a a+b
and

b
c
(a + b + c)2
a
+
+

.
b+c c+a a+b
2(ab + bc + ca)

270, For all nonnegative real numbers a, b and c, no two of which are zero,
X

(a + b + c)4
1

.
2a2 + 3ab + 2b2
7(a2 + b2 + c2 )(ab + bc + ca)2

271,
X
cyc

a2

a(b + c)

cyc

a(b + c)

cyc

X
cyc

1
(a + b + c)4
(b + c)2

But what about the following:


1
1
1
3(a + b + c)2
+
+

(a + b)2 (b + c)2 (c + a)2


8(ab + bc + ca)

1
1
+ 2
ab + bc + ca a + b2 + c2

272, For all nonnegative real numbers a,b and c, no two of which are zero,
p
3 3abc(a + b + c)(a + b + c)2
1
1
1
+
+

(a + b)2
(b + c)2
(c + a)2
4(ab + bc + ca)3
Its obviously trues because of Am-Gm, we have:
X
X
X
(
a2 +
ab)2 .
ab 108witha + b + c = 3
Replacing a, b, cby a1 , 1b , 1c respectively, we have to prove that
p
X a2 b2
3 3(ab + bc + ca)(ab + bc + ca)2

.
(a + b)2
4(a + b + c)3
108

Now, using Cauchy Schwarz Inequality, we hav


X

a2 b2
(ab + bc + ca)2
(ab + bc + ca)2

=
2
2
2
2
2
(a + b)
(a + b) + (b + c) + (c + a)
2(a + b2 + c2 + ab + bc + ca)

It suffices to prove that


p
3 3(ab + bc + ca)(ab + bc + ca)2
(ab + bc + ca)2

2(a2 + b2 + c2 + ab + bc + ca)
4(a + b + c)3
or equivalently,
p
2(a + b + c)3 3 3(ab + bc + ca)(a2 + b2 + c2 + ab + bc + ca)
that is
4(a + b + c)6 27(ab + bc + ca)(a2 + b2 + c2 + ab + bc + ca)2
By AM-GM, we see that
27(ab + bc + ca)(a2 + b2 + c2 + ab + bc + ca)2
3
1
2(ab + bc + ca) + (a2 + b2 + c2 + ab + bc + ca) + (a2 + b2 + c2 + ab + bc + ca) =
2
4(a + b + c)6
273,
2a2

1
1
1
2
1
+ 2
+ 2

+ 2
+ bc 2b + ca 2c + ab
ab + bc + ca a + b2 + c2

1st solution. (also in pvthuans book, page 62)


By Cauchy inequality,
X

(b + c)2 (2a2 + bc)

cyc

X
cyc

2a2

1
4(a + b + c)2
+ bc

It remains to show that


X
(b + c)2 (2a2 + bc) 4(a2 + b2 + c2 )(ab + bc + ca)
cyc

which is easy. 274, Let a, b, c > 0 such that a + b + c = 3abc. Prove that
X

1
3

a+b
2

t :
a=

1
1
1
; b = ; c = ; xy + yz + zx = 3
x
y
z
109

LHS =

X xy
xy
3

x+y
2
2

275, Prove if a, b, c > 0 such that a2 + b2 + c2 = 3 then


X

a4

a3
3
b4 + b 2

The condition of this ineq didnt show a4 b4 + b2 0. But if a4 b4 + b2 0(and


others expression) , we can prove M
(a2 + b2 + c2 )LHS 2 (a2 + b2 + c2 )3 = 27
=> LHS 3
276, Prove that if a, b, c are nonnegative real numbers, we have
s
r
r
r
2abc
3 a(a + b)
3 b(b + c)
3 c(c + a)
+
+
33 3
a2 + 2b2
b2 + 2c2
c2 + 2a2
(a + b3 + c3 )
r
3

a(a + b)
+
a2 + 2b2

r
3

b(b + c)
+
b2 + 2c2

r
3

r
c(c + a)
2abc
3

3
c2 + 2a2
a3 + b3 + c3

By Am-Gm ,we need to prove :


(a3 + b3 + c3 )3 abc(a2 + b2 + c2 )3
It can prove by Am-Gm
s
s
abc(a
+
b)(b
+
c)(c
+
a)
8a2 b2 c2
9

3
LHS 3 9 2
(a + 2b2 )(b2 + 2c2 )(c2 + a2 )
(a2 + 2b2 )(b2 + 2c2 )(c2 + a2 )
So we need to prove:
(a3 + b3 + c3 )3 abc(a2 + b2 + c2 )3
But
a3 + b3 + c3 3abc
and
3(a3 + b3 + c3 ) (a2 + b2 + c2 )3
277, Let a, b, c be positive reall number satisfyin abc = a + b + c. Prove the following
inequality :

b
c
3 3
a
+
+

1 + a2
1 + b2
1 + c2
4
CM:

X a
3 3

1 + a2
4
110

Setting :
1
1
1
; b = ; c = => xy + yz + zx = 1
x
y
z

a=

By Am-Gm ,we can prove :


1
2
2
+x
+ ; x > 0
x
3x
3
So we need to prove:

x
3

4
3+x

1
3 3
<=>

4
3+x
But its true by Am-Gm.
278, Let a, b, c > 0, ab + bc + ca = 3. Prove that :

a
b
c
+
+
1
5c + 4a
5b + 4c
5a + 4b

Cauchy-schwarz ?
27LHS 2 = [

a(5b + 4c)]LHS 2 (a + b + c)3

We need to prove:
(a + b + c)3 9(ab + bc + ca)
But its true by Am-Gm Smile.
279, Given a, b, c > 0. Prove that:
r
3
3
3
a b
c
4 a + b + c
+ + 3.
()
b
c a
3abc
I hope youll like it, my dear friend Smile
This is my solution.
c
a
b
Let = x, = y, = z.
c
a
b
Then we have : xyz = 1 and :
r
1 y
z
x
() x + y + z 3 4 ( + + )
3 z
x y
(x + y + z)4 27(xy 2 + yz 2 + zx2 )
But we have a inequality :
27(xy 2 + yz 2 + zx2 ) 4(x + y + z)3 27
111

Therefore we have only prove that :


(x + y + z)4 4(x + y + z)3 27
(x + y + z 3)2 [(x + y + z)2 + 2(x + y + z) + 3] 0
Its right Wink .
Suppose
abc = 1; P uta =

y
z
x
;b = ;c =
x
y
z

This inequality become:


(x3 + y 3 + z 3 )4 27xyz(x6 y 3 + y 6 z 3 + z 6 x3 )()
Put m = x3 , n = y 3 , p = z 3 , () become:
(m + n + p)1 2 31 2mnp(m2 n + n2 p + p2 m)3 ()
Using yhe well-known result:
27(m2 n + n2 p + p2 m + mnp) 4(m + n + p)3
and AM-GM inequality, we have ().
280, For positive number x, y, z such that x + y + z = 1. Prove that

2
yz
zx
xy

+
+

xy + yz
yz + zx
zx + xy
2
s
X
2(xy + xz + yz)2
xy
1
xy + xz + yz (xy + yz)(x + y + z)2
cyc
But
X
cyc

xy

xy + xz + yz

v
uX
u
2(xy + xz + yz)2
2x(xy + xz + yz)
t

2
(xy + yz)(x + y + z)
(x + z)(x + y + z)2
cyc

we have to prove that


X 2x
(x + y + z)2

x+z
xy + xz + yz
cyc
<=>

<=>

X 2x2 z
x2 + y 2 + z 2
x
+
z
cyc

x2 + y 2 + z 2
x4
y4
z4
2
+ 2
+ 2
2
x + xz
y + xy z + yz
112

but we have that


x4
y4
z4
(x2 + y 2 + z 2 )2
x2 + y 2 + z 2
+
+

x2 + xz
y 2 + xy z 2 + yz
x2 + y 2 + z 2 + xy + xz + yz
2
s
! s
r
X
X
X
X x+y
xy
x2 y
4x2 y
2x2 y

=
=

2
xy + yz
x+z
2
(x + y) (x + z)
(x + y)(x + z)
cyc
cyc
cyc
cyc
Remain to prove that
X
cyc

1
2x2 y

(x + y)(x + z)
2

But
X
cyc

X
2x2 y
1

4x2 y(y + z) (x + y)(x + z)(y + z)(x + y + z)
(x + y)(x + z)
2
cyc

(x3 y + x3 z 2x2 y 2 ) 0.

cyc

281, Let a, b, c > 0 such that a + b + c = 3. Prove that:


abc + a2 b + b2 c + c2 a 4
Because

4
3
(a + b + c) abc
27
282, Let a + b + c + d = 4 and a, b, c, d 0. Prove that
a2 b + b2 c + c2 a

a2 bc + b2 cd + c2 da + d2 ab 4
It is necessary to prove, that
(a + b + c + d)4 64(a2 bc + b2 cd + c2 da + d2 ab)
If a = min{a, b, c, d} and b = a + x, c = a + y, d = a + z then it is killing. But it is
very ugly.
Let p, q, r, s = a, b, c, d and p q r s. Then by rearrangement inequality,
a2 bc + b2 cd + c2 da + d2 ab = a(abc) + b(bcd) + c(cda) + d(dab)
p(pqr) + q(pqs) + r(prs) + s(qrs) = (pq + rs)(pr + qs)
(

1
pq + rs + pr + qs 2
) = ((p + s)(q + r))2
2
4
1 p+q+r+s 2 2
((
) )
4
2
= 4.
113

Equality holds q = r = 1 and p + s = 2. So equality holds if two of


them are equal to 1. Applying this, we can get the equality conditions (a, b, c, d) =
(1, 1, 1, 1), (2, 1, 1, 0) or any cyclic forms.
And by this idea, we can solve that
a2 b + b2 c + c2 a

4
if a + b + c = 1
27

which was from Canada. If {p, q, r} = {a, b, c}, p q r, then as pq pr qr,


a2 b + b2 c + c2 a = a(ab) + b(bc) + c(ca) p(pq) + q(pr) + r(qr)
1
(2q)(p + r)(p + r)
2
1 (2q) + (p + r) + (p + r) 3
(
)
2
3
4
1 2
.
= ( )3 =
2 3
27
282, Let a, b, c > 1. Prove that:
= q(p2 + pr + r2 ) q(p + r)2 =

1 + a2
2
1 + b + c2

We have : a, b, c > 1 therefore


1 + b + c2 , 1 + c + a2 , 1 + a + b2 0
So
b2 + 1 2b
2(1 + b + c2 ) (1 + b2 ) + 2(1 + c2 )
2(1 + c + a2 ) (1 + c2 ) + 2(1 + a2 )
2(1 + a + b2 ) (1 + a2 ) + 2(1 + b2 )
So,
X

X 2x
X 2x2
1 + a2

=
2
1+b+c
y + 2z
xy + 2xz

where x = 1 + a2 , y = 1 + b2 , z = 1 + c2 . Clearly x, y, z > 0. Other, by CauchySchwarzt we have :


X

2x2
2(x + y + z)2

2
xy + 2xz
3(xy + yz + zx)

283, Let x, y, z 0 be such that x2 + y 2 + z 2 = 1. Prove that


1

z
y
z
+
+
2
1 + xy 1 + xz
1 + xy
114

Firstly, we prove that:


(a + b + c)2 2(1 + bc)2 (1)
Indeed, we have :
(1) 2(ab + bc + ca) 1 + 4bc + 2b2 c2 2a(b + c) a2 + (b + c)2 + 2b2 c2
(b + c a)2 + 2b2 c2 0(true)
Therefore,

b
c
2a
2b
2c
a
+
+

+
+
= 2
1 + bc 1 + ca 1 + ab
a+b+c a+b+c a+b+c
Other, we have :
(a 1)2 (a + 2)
a(b2 + c2 )
=1
1
2
2

a + abc a +
therefore,

a
b
c
+
+
a2 + b2 + c2 = 1
1 + bc 1 + ca 1 + ab
We are done
a
b
c
+
+
1 + bc 1 + ca 1 + ab
!2
!
!

X a
X
X
a

a(1 + bc)
a 1 + bc
1 + bc
1 + bc
cyc
cyc
cyc
1

(a + b + c)2 =

X
cyc

a
(a + b + c)2

1
1 + bc
a + b + c + 3abc
p2 p 3r 0

284, Let a, b, c 0 satisfying

a2 = 1. Prove that:
X

a
1
1 + bc

X
X
a
a
a

=
2 +c2 =
b
1a2
1
+
bc
1
+
1
+
cyc
cyc
cyc
2
2


X a(a + 2)(a 1)2
X  2a
1
1
2

=1+
1.
=1+
2
3a
3
3
3(3 a2 )
cyc
cyc
P
P
X a
1X X 1
3 a
6 a
P
P

1 + bc
3
1 + bc
3 + ab
5 + ( a)2
X

115


P
And 1 a 3
P
X
X
X
X
6 a
P 2 1(
a)2 6

a+5 0 (
a1)(
a5) 0, Right
5 + ( a)
by cauchy
X

a
(a2 + b2 + c2 )2
1
P
3
= 3
1 + bc
a + b3 + c3 + a3 bc
a + b3 + c3 + abc

It remains to show that

1
1
a3 + b3 + c3 + abc

which is obviously true from


(a2 + b2 + c2 )3 (a3 + b3 + c3 + abc)2
( by muirhead , AM-GM)
(a2 + 2)3 (a3 + a + 2)2 a(a2 + 1)2
But after cancelling the degree 6 term the right side has larger degree so it is incorrect.
Try again you should look yourself After expanding
L.H.S = a6 + 6a4 + 12a2 + 8 2a5 + 4a4 + 2a3 + 10a2 + 2a + 7byAM GM
and
2a5 + 4a4 + 2a3 + 10a2 + 2a + 7 > a5 + 2a3 + a = a(a2 + 1)2
285, Let a, b, c 0. Prove that:
X
a2

9
1
P 2
2
+ ab + b
( a)

Solution : We have :
a2 + ab + b2 = (a + b + c)2 (ab + bc + ca) (a + b + c)c
b2 + bc + c2 = (a + b + c)2 (ab + bc + ca) (a + b + c)b
c2 + ca + a2 = (a + b + c)2 (ab + bc + ca) (a + b + c)a
Suppose a + b + c = 1. We have :
()

1
9
1 (ab + bc + ca) a

1 4(ab + bc + ca) + 9abc 0


(a + b + c)2 4(ab + bc + ca)(a + b + c) + 9abc 0
a3 + b3 + c3 + 3abc a(b2 + c2 ) + b(c2 + a2 ) + c(a2 + b2 )
116

It is true because it is inequality s Schur. We are done.


Try
X
cyc

X
(3c + a + b)2
1
=

a2 + ab + b2
(3c + a + b)2 (a2 + ab + b2 )
cyc
25(a + b + c)2
P
.
(3c + a + b)2 (a2 + ab + b2 )

286, I know its Cauchy, but how do we show


25(a + b + c)2
P
9
(3c + a + b)2 (a2 + ab + b2 )
P
pqr technique works here. We put a+b+c = 1, ab = q, abc = r. The ineq becomes
q(3q 1)2 + 9r + 1 4q
after expanding Very Happy And it is obivious since 9r + 1 4q 0(schur ineq) The
equality holds when a = 0, b = c = 21 , a = b = c.
287, Solve the equation

q
p

9 2
2
x+ x 1=
(x 1) x 1
4

2
2
Let a = x 1, b = x + 1. Then b a = 2. There fore :
 2
b a2 = 2
2(a + b) = 9a3
 2
b = a2 + 2

2b = 9a3 2a
 2
b = a2 + 2

81a6 36a4 8 = 0
 2
b = a2 + 2

a2 = 32
 2
b = 83

2b = 9a3 2a
5
3
288, Let a, b, c be nonnegative real numbers such that a + b + c = 1. Prove that
X p
a 8b2 + c2 1
x=

X p
3
a 4b2 + c2
4
117

Note that
3b + c

p
3bc
8b2 + c2
2b + c

Hence, it suffices to show that


X
X
3bc
(
a)2
a(3b + c
)
2b + c
X
X
1
+
a2 2
ab 0
2b + c
The Cauchy Schwarz Inequality gives us
X

3abc

1
3

2b + c
a+b+c

It suffices to show that


X
X
9abc
+
a2 2
ab 0
a+b+c
X
X

a3 + 3abc
bc(b + c)
which is true by Schur.
289,
X

a2 +

ab

X p
a 3b2 + c2

We have
X
X
X 1
X
X
X
X p
2bc
9abc
a(2b+c
)=3
ab2abc
3
ab

a2 +
ab
a 3b2 + c2
b+c
b+c
a+b+c
290,

X p
3
a 4b2 + c2
4
I used two lemmas in my solution
1) 2b + c
2) 8

p
2bc(2b + c)
4b2 + c2

4b2 + 3bc + c2

X
4b2

2b + c
27

2
+ 3bc + c
a+b+c

291, Another question, please.


I see that
p
3bc
3b + c
8b2 + c2
2b + c
is AM-GM
p
(2x + 1)2 + (8x2 + 1) 2(2x + 1) 8x2 + 1

118

but for the other inequality how to estabilish the right factor Ax + B to write
p
(Ax + B)2 + (4x2 + 1) 2(Ax + B) 4x2 + 1
Thank you very much.
Well, for the second inequality, I dont establish the inequality (since it reverse...)
p
(Ax + B)2 + (4x2 + 1) 2(Ax + B) 4x2 + 1
292, Let a, b, c, d 0 a2 + b2 + c2 + d2 = 4. Prove that:
a3 + b3 + c3 + d3 8
I just have seen that this inequality is too easy.
Squaring the both sides weneed to prove that:
(a3 + b3 + c3 + d3 )2 (a2 + b2 + c2 + d2 )3 .
Using CBS we infer that:
(a3 + b3 + c3 + d3 )2 (a2 + b2 + c2 + d2 )(a4 + b4 + b4 + d4 )
So we need only to prove that:
a4 + b4 + c4 + d4 (a2 + b2 + c2 + d2 )2
We have :
a2 + b2 + c2 + d2 = 4
0 a, b, c, d 2
3

0 a 2a , 0 b3 2b2 , 0 c3 2c2 , 0 d3 2d2


0 a3 + b3 + c3 + d3 2(a2 + b2 + c2 + d2 ) = 8
293, Let x, y, z > 0. Prove that :
p
p
p
3( x(x + y)(x + z) + y(y + z)(y + x) + z(z + x)(z + y))2 4(x + y + z)3
By Cauchy-Schwarz ineq , we have :
X
LHS 3(x + y + z)(
x2 + yz + zx + xy)
Then we prove that :
4(x + y + z)2 3[(x + y + z)2 + xy + yz + zx]
X

(x y)2 0
294, Prove that for any reals x, y, z which satisfy condition x2 + y 2 + z 2 = 2 we have
x + y + z) xyz + 2
119

We have :
2 = x2 + y 2 + z 2 2yzoryz 1
By Bunhiacopsky we have :
[x(1 yz) + y + z]2 [x2 + (y + z)2 ][(1 yz)2 + 1] (2 + 2yz)(y 2 z 2 2yz + 1) 4
(becauseyz 1) There fore:
|x + y + z xyz| 2
or
x + y + z 2 + xyz
295, If a, b, c > 0. Prove that
b3
c3
a2 + b2 + c2
a3
+
+

b+c a+c a+b


2
although it can be easily solved by rearrangement inequality, i wonder if there is any
other method
Here is a solution using Andreescu
a3
b3
c3
a2 + b2 + c2
+
+

b+c a+c a+b


2

b4
c4
(a2 + b2 + c2 )2
(a2 + b2 + c2 )(ab + bc + ca)
a2 + b2 + c2
a4
+
+

=
a(b + c) b(a + c) c(a + b)
2(ab + bc + ca)
2(ab + bc + ca)
2

Proof 1:
I have :

a3
a(b + c)
+
a2
b+c
4

There fore:
b3
c3
ab + bc + ca
a2 + b2 + c2
a3
+
+
a2 + b2 + c2

b+c c+a a+b


2
2
Proof 2 : By Chebysev inequality and Nesbit inequality we have :
a3
b3
c3
1
a
b
c
a2 + b2 + c2
+
+
(a2 + b2 + c2 )(
+
+
)
b+c c+a a+b
3
b+c c+a a+b
2
296, For all nonnegative real numbers a, b and c, no two of which are zero,
a(b + c)(a2 bc) b(c + a)(b2 ca) c(a + b)(c2 ab)
+
+
0
a2 + bc
b2 + ca
c2 + ab
!
X
1 X 1
1

2
2 cyc ab
c + ab
cyc
120

c4 (a2 b2 )2 + b4 (c2 a2 )2 + c4 (a2 b2 )2


0
(a2 + bc)(b2 + ca)(c2 + ab)
Setting A = LHS, then we see that
A=

X a3 (b + c)
a2 + bc

X a3 (b + c)

abc

X b+c
X a3 (b + c)
X1

abc
=
2
2
a + bc
a + bc
a

1 X a(b + c)(a2 bc)


1
= A
+ bc
2
2
a2 + bc
2
which shows that A 0. 297, For any positive real numbers a, b and c
s
r
r
r




 1
a(b + c)
b(c + a)
c(a + b)
1
1

a
+
b
+
c
+
+
+
+

a2 + bc
b2 + ca
c2 + ab
a
c
b
a2

X a(b + c)

From Cauchy we have


X

a(b + c)
a2 + bc

!2

X X a(b + c) 
(
a)
a2 + bc

Now all we have to prove is


X a(b + c)
a2 + bc

X 1

which is equivalent
X (a b)(a c)
2
0
a(a + bc)
which is Vornicu Schur.
From this idea we should square the inequality and then use that
ab(c + a)(c + b)
1
(a2 + bc)(b2 + ca)
for example .Then you will have to prove that

b
(a + b)(a + c) (b + a)(b + c) (c + a)(c + b) X a
+
+

( + )
2
2
2
a + bc
b + ca
c + ab
a
b
Very nice, Ductrung. Here is my solution for it
We have the inequality is equivalent to
X

X a(b + c)
a2 + bc

a(b + c)
a2 + bc
s

+2

!2

 X  X 1 

a
a

 X  X 1 
ab(a + c)(b + c)

a
(a2 + bc)(b2 + ca)
a
121

We can easily prove that


s
X

ab(a + c)(b + c)
3
(a2 + bc)(b2 + ca)

So, it suffices to prove that

 X  X 1 

+6
a
a2 + bc
a

X a(b + c)

To prove this ineq, we only need to prove that


a + b c(a + b)
2
10
c + ab
ab
But this is trivial, because
a + b c(a + b)
2
1 = (a+b)
ab c + ab

1
c
2
1 2 ab
ab c + ab




1
c
2
1 =
ab c + ab

We are done.
298, For any positive real numbers a, b and c,
s


a(b + c) b(c + a) c(a + b)
1
1 1 1
+ 2
+ 2

27 + (a + b + c)
+ +
a2 + bc
b + ca
c + ab
2
a b
c
299, The Inequality is equivalent to
X a2 (b + c)2



X ab(b + c)(c + a)
15 1 X b + c
+2

+
(a2 + bc)2
(a2 + bc)(b2 + ca)
2
4
a

Notice that
(a2 + bc)(b2 + ca) ab(b + c)(c + a) = c(a + b)(a b)2
then
2

X ab(b + c)(c + a)
6(1)
(a2 + bc)(b2 + ca)

Other hand,
X a2 (b + c)2
(a2 + bc)2

X a2 (b + c)2
4a2 bc

1X
=
4


b c
+ +2
c b

From (1) and (2) we have done! Besides, by the sam ways, we have a nice solution for
an old problem:
s
r
 X  X 1 
X a(b + c)

a
a2 + bc
a
122

2
ab

c2 + ab

300, Let a, b, c > 0 and abc = 1. Prove that


Xa1

0
b+c
X 1
(a + b + c 1)(
)3
b+c
By Am-Gm , we can prove :
LHS (a + b + c 1)

9
2(a + b + c)

So we need to prove this ineq :


3(a + b + c 1) 2(a + b + c)
<=> a + b + c 3
301, Let a, b, c be nonnegative real numbers. Prove that
r
r
r
r
3
ab + bc + ca
a
b
c
3

+
+

a2 + b2 + c2
a+b
b+c
c+a
2
2
X
Xr a
(a + b + c)(ab + bc + ca)
9
a
)2 2(a+b+c)(
)=4

(
a+b
(a + b)(a + c)
(a + b + c)(ab + bc + ca) abc
2
301, Given a, b, c 0 and a + b + c = 8. Prove that:
268 + 12a2 b2 c2 ab(a b)2 + bc(b c)2 + ca(c a)2
When does equality occurs ?
By "pqr"
4
12(r 1)2 + (q 2 40)2 0
9
302, Given a, b, c > 0 satisfy a + b + c = 3. Prove that
ab bc ca 9abc
21
+
+
+

c
a
b
4
4
Below is my first attempt, which is indirect but fairly short: Rewrite the inequality as
1
1
1
9
21
+ 2+ 2+
a2
b
c
4
4abc
Put x = a1 , y = b1 andz = c1 . Then xy + yz + zx = 3xyz and the inequality
becomes
4(x2 + y 2 + z 2 ) + 9 7(xy + yz + zx)
or after homogenizing
4(x + y + z)2 +

81(xyz)2
15(xy + yz + zx)
(xy + yz + zx)2
123

Without loss of generality, assume x + y + z = 1. Put xy + yz + zx = (1 q 2 )/3.


Then as in
(1 + q)2 (1 2q)
xyz
27
It remains to show that



2
2
3
(1 + q)2 (1 2q)
1 q2
(1 q 2 )
+ 81
15
4
3
27
3
or
4(1 q 2 )2 + (1 + q)4 (1 2q)2 5(1 q 2 )3 0
But this is reduced to
q 2 (1 + q)2 (1 3q)2 0
303, Using Cauchy Schwarts and Am-Gm, well need to prove:
X a4 + b4
2(ab + bc + ca)2
+
(a + b + c)2
2
2
a +b
a2 + b2 + c2
First squaring

X a4 + b4
a2

b2

+4

X
a2 b2
p
(
a)2
2
2
2
2
(a + b )(b + c )

Then use
(a2 + b2 )(b2 + c2 ) (a2 + 2b2 + c2 )2
and Cauchy Schwartz. The ineq turns into the form nguoivn gave.
X (a2 + b2 ca cb)2 (a b)2
a2 + b2

304, Let a, b, c be positive numbers such that: a + b + c = 1. Prove that:


a
b
c
36(a2 + b2 + c2 )
+
+

b2 + b c2 + c a2 + a
ab + bc + ca + 5
I can prove your problem by one result of hungkhtn and vacs is if a + b + c = 1, a, b, c
be positive numbers then
a2 b + b2 c + c2 a + abc

4
27

uses cauchuy-schawrs we have:


X
b2
let q =
36

a
(a + b + c)2
36(a2 + b2 + c2 )
P 2
P
+b
ab + b a
ab + bc + ca + 5

ab, r = abc we have

X
X
X
X
X
4
a2 (
ab2 +
ab) 36
a2 ( abc +
ab)
ab + 5
27
124

<=> q + 5 36(1 2q)(


becase
1 2q 1

4
r + q)(1)
27

2
0
3

so uses schur third degree we have


r

1 4q
9

supposing
(1) <=> q + 5 36(1 2q)(

4
4q 1

+ q) <=> (1 3q)(40q + 13) 0


27
9

equality when a = b = c = 31 .
my solution:
b
c
3
a
+
+

b2 + b c2 + c a2 + a
4

a b
c
+ +
b
c a

36(a2 + b2 + c2 )
ab + bc + ca + 5

How about the stronger, Toan Smile


a b
c
30(a2 + b2 + c2 )
+ +
b
c a
3(a + b + c)2 + ab + bc + ca
I think we need to use the lemma:


c
21(a2 + b2 + c2 )
a b
+ +
2
+1
2
b
c a
(a + b + c)
305, Let be a, b, c > 0. Show that :


1 1 1
2((a b)2 + (b c)2 + (c a)2 )
(a + b + c)
+ +
9+
a b
c
ab + bc + ca
<=> Sc (a b)2 + Sa (b c)2 + Sb (a c)2 0
With:
Sc =

1
2

; ...
ab ab + bc + ca

Assume:
a b c.
So : Sa ; Sb 0 easyRazz
We have:
Sa + Sc =

a2 (b + c) + c2 (a + b) 2abc
0
abc(ab + bc + ca)

125

306, If x, y, z are reals and x2 + y 2 + z 2 = 2. Prove that


x + y + z xyz + 2
WLOG x y z xy 1. By Cauchy-Schwartz

(xyz(x+y+z))2 = (z(xy1)xy)2 (z 2 +(x+y)2 )((xy1)2 +1) = (2+2xy)(22xy+(xy)2 ) = 42(xy)2 (


P
307, For any positive reals a, b, c such that a = 1
(ab + bc + ca)(

a
b
c
3
+
+
) holds
b2 + b c2 + c a2 + a
4

by AM-GM;Schwarz;Holder ineqlities,we obtain:



X a
X a
a
=

b2 + b
b
b+1
!

r
4
4
Xa
X a
33 a
33 X 4 a

=

aaa
b
b
4
b
44b

r
4
Xa
3Xa
3
33 4 X a

b
4
b
4
b
4(ab + bc + ca)
308, Let a, b, c 0. Prove that
X

X a
a 3
9abc
) +

b+c
(a + b)(b + c)(c + a)
b+c

q = xy + yz + zx, p = x + y + z, r = xyz
my solution
X

X a
a 3
9abc
) +

b+c
(a + b)(b + c)(c + a)
b+c
<=> p3 + 12r + 3pr 16p
<=> p3 + (3r 16)p + 12r 0
f (p) = p3 + (3r 16)p + 12r

f 0 (p) = 3p2 + 3r 16
p
p
f 0 (p) = 0 <=> p = (16 3r)/3 13/3
=> f 0 (p) > 0withp [3; 4]
=> M inf (p) = minf (3)
If p 4
we have
r

16p p3
4p + 9
126

309, For a, b, c > 0 and ab + bc + ca = 3. Prove that :


3+

(a b)2 + (b c)2 + (c a)2


a + b2 c2
b + c2 a2
c + a2 b2

+
+
3
2
b+c
c+a
a+b

By cauchy-swarchz:
X b2 c2
a + b2 c2 b + c2 a2 c + a2 b2
(a + b + c)2
a + b + c X b2 c2
+
+

+
b+c
c+a
a+b
2(ab + bc + ca)
b+c
2
b+c
By Am-Gm We have:
X a2 b2
X
a+b
(
+
)
ab = 3
a+b
4
By Am-Gm and Cauchy Schwarts, we can prove easily the stronger:
5(a2 + b2 + c2 ) 1
a + b2 c2
b + c2 a2
c + a2 b2
+
+
+
6
2
b+c
c+a
a+b
The first, using my old result:
X a
1
a2 + b2 + c2
+
ab + bc + ca 2
b+c
Besides, by CS and Am-Gm:
s
r
X a2 b2
X
X a2 b2
X ab X a2 + b2 + c2
1 2

a2 b2 .

.(a + b2 + c2 )2 .
=
2
a+b
(a + b)
3
4
2
Add 2 inequalities, we have my stronger
310, Let x, y, z be postive real numbers such that xyz = x + y + z + 2. Prove that:

2( xy + yz + zx) x + y + z + 6
Proof: The inequality is enquivalent to:
X
X
p
(
x)2 (x + y + z) x + y + z + 6or
x 2(x + y + z + 3)
Denote

c+a
a+b
b+c
,y =
,z =
a
b
c
Therefore, we just need to prove:
r
r
r
r
X b+c
Xb+c
X
1 1 1
1 1 1
2(
+ 3) = 2(a + b + c)( + + ) = [
(b + c)]( + + )
a
a
a b
c
a b
c
x=

But this is obviously true due to Cauchy-Schwartz, which ends our proof.
311, Given a, b, c are prositive real numbers a2 + b2 + c2 = 1. Find max of P:
P =

ab
bc
ca
+
+
2
2
1+c
1+a
1 + b2
127

I think it trues by AM-GM:


X
a2

c2

X
ab
ab
1X
b2
a2
p

+ 2
)
( 2
2
2
2
+b +c
4
a +c
c + b2
2 (a2 + c2 )(b2 + c2 )

anh also true by cauchy Schwarz:


2

X
ab
(a + b)
b2
1X
a2

+
)

(
a2 + c2 + b2 + c2
4(a2 + c2 + b2 + c2 )
4
a2 + c2
c2 + b2

312, Given a, b, c > 0 and ab + bc + ca = 1. Prove that:


(a2 + 2b2 + 3)(b2 + 2c2 + 3)(c2 + 2a2 + 3) 64
313, Let a, b, c > 0 and
1
1
1
1
1
1
+
+
3;
+
2;
1.
a 2b 3c
2b 3c
3c
Prove that

1
1
1
+ 2 + 2 14
2
a
b
c

Setting :
1
1
1
1
1
1
= x + 1;
+
= 2 + y; +
+
= z + 3(x, y, z 0)
3c
2b 3c
a 2b 3c
1
1
1
+ 2 + 2 14 = 2(z + 3y + 5x) + (x y)2 + (y z)2 + x2 0
a2
b
c
Then we have done Mr. It holds when x = y = z = 0
=>

=> a = 1; b =

1
1
;c =
2
3

We have
1
1
1 1 1
1
1
1
1
1
1
a + 2b +
+
+
=
(
+
+
)+3(
+
)+5(
)

3(
3
a2 b2 c2
a2 4b2 9c2
4b2 9c2
9c2

1
3c 2

) +6( 2b

+
3

314, Let x, y, z > 0. Prove that :


p
p
p
3( x(x + y)(x + z) + y(y + z)(y + x) + z(z + x)(z + y))2 4(x + y + z)3
By Cauchy-Schwarz ineq , we have :
X
LHS 3(x + y + z)(
x2 + yz + zx + xy)
Then we prove that :
4(x + y + z)2 3[(x + y + z)2 + xy + yz + zx]
128

1
3c 2

) +5(

1 2
) 3+6+5 =
3c

(x y)2 0

315, Let a, b, c be nonegative numbers. Prove that :


a3 + b3 + c3
135abc
1
+
(a + b)(b + c)(c + a) 8(a + b + c)3
316, Let a, b, c 0. Prove that

a3 + b3 + c3
15 3 abc
+
1
(a + b)(b + c)(c + a) 8(a + b + c)
317, Let a, b, c be positive real numbers such that a + b + c = 3. Prove that


1 1 1
8
+ +
+ 9 10(a2 + b2 + c2 ).
a b
c
Equality holds for a = 2, b = c = 12 .
Setting:
a + b + c = p = 3; ab + bc + ca =
8.

p2 t 2
; abc = r
3

9 t2
+ 27 10(2t2 + 9) 0(3 t 0)
r

We have this ineq :


r

(3 t)2 (3 + 2t)
.
27

Then the ineq becomes one varible .


318, Let a, b, c 0 such that a + b + c = 3. Prove that:
(a2 b + b2 c + c2 a) + 2(ab2 + bc2 + ca2 ) + 3abc 12
I use this ineq :
a, b, c 0; a + b + c = 3
a2 b + b2 c + c2 a + abc 4
319, Given that a, b, c 0. Prove that,


a2 + b2 + c2
1 1 1
+ +
4
+5
(a + b + c)
a b
c
ab + bc + ca
Find the maximal k, so that the following inequality holds:


1 1 1
a2 + b2 + c2
(a + b + c)
+ +
k
+9k
a b
c
ab + bc + ca

129

I have a feeling that, k = 4 is best constant. Try to prove or disprove me.


Simple calcultation, we will set
a :=

1
1
1
, b := , c :=
a
b
c

then our inequality becomes




1 1 1
4(a2 b2 + b2 c2 + c2 a2 )
(a + b + c)
+ +

+5
a b
c
abc(a + b + c)
(ab + bc + ca)(a + b + c)2 4(a2 b2 + b2 c2 + c2 a2 ) + 5abc(a + b + c)
ab(a b)2 + bc(b c)2 + ca(c a)2 0
This proof also shows that the best constant is k = 4. So, your statement is valid, shaam.
We can also prove it by Muirhead inequality: Our inequality is equivolent to
(a + b + c)(ab + ac + bc)2 4abc(a2 + b2 + c2 ) + 5abc(ab + ac + bc)
X
X
X
X
X

a3 b2 + 5
a2 b2 c +
a3 bc 2
a3 bc + 5
a2 b2 c
sym

sym

cyclic

a3 b2

sym

sym

cyclic

a3 bc

sym

which is right by Muirhead inequality.


It is only the following indentity Mr. Green
(a b)2 (b c)2 (c a)2 0
which is the strongest 3-variables inequatily
We can also solve it easily by SOS with
Sa =

2
1

...
bc ab + bc + ca

(a + b + c)2
a2 + b2 + c2
+5=4
3
ab + bc + ac
ab + bc + ac
X a
X (a + b)2
b
(a + b + c)2
1 1 1
( + + 2) =
4
(a + b + c)( + + ) + 3 =
a b
c
b
a
ab
ab + bc + ac
cyc
cyc
4

How about
(a + b 2c)2 (b + c 2a)2 (c + a 2b)2 0
Dear Zaizai, i was asking how to transform it. Because:
(a b)2 (b c)2 (c a)2
130

(a4 b2 + a4 c2 + 2a3 b2 c + 2a3 bc2 )

cyc

(2a3 b3 + 2a4 bc + 2a2 b2 c2 )

cyc

320, Given a, b, c 0. Prove that:


(

a 3
b 3
c 3
abc
1 a2 + b2 + c2 2
) +(
) +(
) +
.(
)
b+c
c+a
a+b
(a + b)(b + c)(c + a)
2 ab + bc + ca

Its stronger than a well-known:


(

b 3
c 3
5abc
a2 + b2 + c2
a 3
) +(
) +(
) +

b+c
c+a
a+b
(a + b)(b + c)(c + a)
ab + bc + ca

PS: With this old ineq, we are easily to solve it by Am-Gm and... Notice that:
abc(a2 + b2 + c2 ab bc ca) 0
I havent tried with AM-GM as you said but the inequality is actually weak and we can
prove it using Cauchy Schwarz + Schur as follow: Applying Cauchy Schwarz Inequality, we have
b
c
(a + b + c)2
a
+
+

b+c c+a a+b


2(ab + bc + ca)
It follows that
a2 + b2 + c2
2
ab + bc + ca


a
b
c
+
+
1
b+c c+a a+b

And we can deduce our inequality to


a3
b3
c3
abc
+
+
+
2
(b + c)3 (c + a)3 (a + b)3 (a + b)(b + c)(c + a)
Setting x =
becomes

2a
b+c , y

2b
c+a , z

2c
a+b ,

2
a
b
c
+
+
1
b+c c+a a+b

then xy + yz + zx + xyz = 4 and our inequality

x3 + y 3 + z 3 + xyz 4(x + y + z 2)2


Now, we denote p = x + y + z, q = xy + yz + Zx, r = xyz, then q + r = 4 and our
inequality is equivalent to
p3 3pq + 4r 4(p 2)2
p3 3p(4 r) + 4r 4(p 2)2
(p 4)(p2 + 4) + (3p + 4)r 0
If p 4, it is trivial. If 4 p 3, applying Schurs Inequality, we obtain r
hence
p(4q p2 )
4=q+r q+
9

131

p(4qp2 )
,
9

It follows that
q
and we obtain
r =4q 4

p3 + 36
4p + 9
p3 + 36
p(16 p2 )
=
4p + 9
4p + 9

We have to prove
(p 4)(p2 + 4) + (3p + 4)

p(16 p2 )
0
4p + 9

p(p + 4)(3p + 4) (4p + 9)(p2 + 4) 0


7p2 p3 36 0
(p 3)(12 + 4p p2 ) 0
which is obviously true because 4 p 3.
This completes our proof. Equality holds if and only if a = b = c or a = b, c = 0 and
its cyclic permutations. 321, Given a, b, c 0. Prove that:
(

b 3
c 3
5abc
a2 + b2 + c2
a 3
) +(
) +(
) +

b+c
c+a
a+b
(a + b)(b + c)(c + a)
ab + bc + ca

I havent read Thuans book, so I dont know that solution. Anyways, the following
inequality is stronger than yours Wink
3 
3 
3

a
b
c
b
c
9abc
a

+
+
.
+
+
+
b+c
c+a
a+b
(a + b)(b + c)(c + a)
b+c c+a a+b
Your proof is same to me Embarassed. I also used a nice lenma:
X

a
a2 + b2 + c2
4abc

+
b+c
ab + bc + ca (a + b)(b + c)(c + a)

After expand, its become:


abc(a2 + b2 + c2 ab bc ca) 0
And I think its an useful lenma because notice that its stronger than:
a
(a + b + c)2

b+c
2(ab + bc + ca)

We dont need to expand here, nguoi vn. We rewrite it as follow:


X a[a(b + c) + bc]
b+c
a2 + b2 + c2 + abc

a2 + b2 + c2 +

4abc(ab + bc + ca)
(a + b)(b + c)(c + a)

1
4abc(ab + bc + ca)
a2 + b2 + c2 +
b+c
(a + b)(b + c)(c + a)
132

1
4(ab + bc + ca)

b+c
(a + b)(b + c)(c + a)

which is obviously true by Cauchy Schwarz because


X
and

1
9

b+c
2(a + b + c)

4(ab + bc + ca)
9

(a + b)(b + c)(c + a)
2(a + b + c)

I meant:

a
a2 + b2 + c2
4abc

+
b+c
ab + bc + ca (a + b)(b + c)(c + a)

(a + b + c)(a2 + b2 + c2 ) abc
a2 + b2 + c2

(a + b)(b + c)(c + a)
ab + bc + ca
X
<=> (a+b+c)(a2 +b2 +c2 )(ab+bc+ca)abc
ab [(a+b+c)(ab+bc+ca)abc](a2 +b2 +c2 )
<=>

<=> abc(a2 + b2 + c2 ab bc ca) 0


I think both of ways also short and nice
322, Let a, b, c be nonnegative real numbers, not all are zero. Prove that
b
c
2 ab + bc + ca
a
+
+
+ 2
1
a + b + 7c b + c + 7a c + a + 7b 3 a + b2 + c2
Equality holds if and only if (a, b, c) (1, 1, 1), (2, 1, 0), (1, 0, 0) Now, I will post my
solution for this inequality. Because
a
a
6ca

=
,
a + b + c a + b + 7c
(a + b + c)(a + b + 7c)
it suffices to prove that
X

ca
(a + b + c)(ab + bc + ca)

a + b + 7c
9(a2 + b2 + c2 )

If a = b = 0 or b = c = 0 or c = a = 0, the inequality becomes equality. For


a + b > 0, b + c > 0, c + a > 0, applying the Cauchy Schwarz Inequality, we get
X

ca
(ab + bc + ca)2
P
.
a + b + 7c
ca(a + b + 7c)

The inequality is reduced to


9(ab + bc + ca)(a2 + b2 + c2 ) (a + b + c)(7
or

a3 b + 4

ab3 4

133

a2 b +

a2 b2 abc

a,

ab2 + 3abc),

that is

ab(a 2b)2 abc

a.

Applying the Cauchy Schwarz Inequality again, we get


hX

ab(a 2b)2

i2
i X  h X
abc(a 2b) = abc(a + b + c)2 ,
c

hence

ab(a 2b)2 abc

and our proof is completed.


323, Let a, b, c > 0. Prove that






1
1
1
4
1 1 1
a+
b+
c+

a+b+c+ + +
a
b
c
3
a b
c
Its easy



1
1
1
(a + ) b +
c+
4(c +
a
b
c



1
1
1
(a + ) b +
c+
4(b +
a
b
c



1
1
1
(a + ) b +
c+
4(a +
a
b
c

1
)
c
1
)
b
1
)
a

After summing we will get:








1
1
1
4
1 1 1
a+
b+
c+

a+b+c+ + +
a
b
c
3
a b
c
324, Let a, b, c > 0. Prove that

(1 +

ab)2

(1 +

bc)2

(1 +

ca)2

3+a+b+c
(1 + a)(1 + b)(1 + c)

ab)2 (1 + a)(1 + b)
X
1
1
1
1
3+a+b+c

2
+
+
=
2
2
(1
+
a)(1
+
b)
(1
+
a)(1 + b)(1 + c)
(1
+
ca)
(1 + ab) (1 + bc)
(1 +

134

You might also like